面白い問題おしえてーな

1 名前:132人目の素数さん投稿日:02/07/09 21:38
教えてください

面白い問題教えて 第2版
http://natto.2ch.net/math/kako/1004/10048/1004839697.html
面白い問題教えて
http://cheese.2ch.net/math/kako/970/970737952.html


2 名前:132人目の数列さん投稿日:02/07/09 21:43
次の数列a(n)の一般項を求めてください。
a(n)={1,2,3,2,2,3,3,2,3,4,2,3,5,2,3,6,2,3,7,2,3,8,2,3,......}
体操の掛け声みたいな数列ですよ。

3 名前:投稿日:02/07/09 21:43
じゃあ拙者から
【問題】
点Oを中心とする同一円周上に四点A,B,C,Dがあり、
∠AOB=10°,∠BOC=15°,∠COD=25°,∠AOC=25°,∠BOD=40°である。
点Bから直線ODに下ろした垂線と直線OCの交点をP,
点Dから直線OBに下ろした垂線と直線OCの交点をQとする。
線分OP,OQ,ADを長い順に並べよ

4 名前:投稿日:02/07/09 21:44
次の等式が正しいことを左辺から右辺を導くことによって説明しなさい。
  12+22+32+・・・+19982+19992+20002
    =2000×1+1999×3+1998×5+・・・
     ・・・+3×3995+2×3997+1×3999

5 名前:132人目の素数さん投稿日:02/07/09 21:49
鋭角三角形 ABC の各頂点 A, B, C から対辺に下ろした垂線と対辺との交点をそれぞれH1, H2, H3とする.三角形ABC の内接円は辺 BC, CA, AB とそれぞれ点 T1, T2, T3 で接する.
直線T2T3に対して直線H2H3と対称な直線をL1, 直線T3T1に対して直線H3H1と対称な直線をL2, 直線T1T2に対して直線H1H2と対称な直線をL3 とする.
L1, L2, L3によって作られる三角形の全ての頂点は,三角形 ABCの内接円の周上にあることを示せ.

6 名前:132人目の数列さん投稿日:02/07/09 21:51
>>3 AD>OP=OQ?
図書いたらかうなった。

7 名前:投稿日:02/07/09 21:53
図間違えてた。禿しく逝ってきます。

8 名前:132人目の素数さん投稿日:02/07/09 21:54
>>7
逝ってらっしゃい

9 名前:132人目の素数さん投稿日:02/07/09 21:57
>>2
a(n)={(n+2)/3}*[cos{2(n+2)π/3}]+2*[cos{2(n+1)π/3}]+3*[cos{2nπ/3}]
[x]:ガウス記号

10 名前:9投稿日:02/07/09 22:01
絶対値つけなきゃ駄目か?
a(n)={(n+2)/3}*[|cos{2(n+2)π/3}|]+2*[|cos{2(n+1)π/3}|]+3*[|cos{2nπ/3}|]

11 名前:132人目の数列さん投稿日:02/07/09 22:03
>>9 多分正解。確認は各自でw
答えは何通りもあるっス。

12 名前:132人目の素数さん投稿日:02/07/09 22:05
>>3 並べる。長い順。
│──────────────── OP
│AD(=OP)














─────────────── OQ


13 名前:9投稿日:02/07/09 22:08
ガウスいらなかった。
a(n)={(n+2)/3}*(2/3)*{cos{2(n+2)π/3}+1/2}
       + 2*(2/3)*{cos{2(n+1)π/3}+1/2}
        + 3*(2/3)*{cos{2nπ/3}+1/2}

14 名前:132人目の数列さん投稿日:02/07/09 22:09
>>10 ガウス記号使わなかったら√とか出てきて楽しいよ。
所詮第二工房ですから正直ガウス記号使うのは思いつかなかった。

15 名前:14投稿日:02/07/09 22:11
レス遅し。もっかい逝ってきます。

16 名前:9投稿日:02/07/09 22:21
b(n)={1,2,3,4,2,2,3,4,3,2,3,4,5,2,3,4,6,2,3,4,.....}

ガウスありなら[|cos{2(n-1)π/8}|]などで同様に。
無しだと?・・・・・まいった。

17 名前:132人目の素数さん投稿日:02/07/10 12:30
3以上の整数pが次の条件を満たすという
1以上p-1以下の整数mでm^2≡-1(mod.p)が存在するという
このようなpで50≦p≦100を満たすものは何個あるか

なお解答は持ってません

18 名前:132人目の素数さん投稿日:02/07/10 13:22
無隋YOdめお
面白い

19 名前:132人目の素数さん投稿日:02/07/11 13:18
あげ・・・

神降臨キボンヌ・・・

20 名前:脳内公理投稿日:02/07/12 16:51
http://www.asahi-net.or.jp/~yx3k-nkzw/MApuzzle.html
ここの数理パズルは面白かった。駄作も混じってるけど。

ところで、
3人の博士が帽子をかぶって自分の帽子の色を推理する問題って、
どのスレにあったっけ?

21 名前:132人目の素数さん投稿日:02/07/12 17:48
あいよ。いつの間にか200も下がってますね
http://science.2ch.net/test/read.cgi/math/1025371587/

22 名前:132人目の素数さん投稿日:02/07/12 23:57
三角形の三辺の長さが整数比で、三角の角度が全て整数となるのは正三角形だけである事を証明せよ。

23 名前:132人目の素数さん投稿日:02/07/13 00:10
http://science.2ch.net/test/read.cgi/math/1018115848/611
エレガントな解答を求む

24 名前:(およそ3)/3=およそ1投稿日:02/07/13 07:58
>>22
π/3は整数?

25 名前:132人目の素数さん投稿日:02/07/13 11:31
>>24
60度って事で

26 名前:132人目の素数さん投稿日:02/07/13 13:07
>>20>数理パズル
まだ全部見てないけど、06がわからん。。。
どちらか1人しか道を知らなくて、一人だけにしか質問できないんだったら
もし知らないほうの人に質問しちゃったら絶対に聞き出せないと思うんだが、む〜

27 名前:132人目の素数さん投稿日:02/07/13 14:00
>>26
「あっちの人に「こっちが,X村に続いてますか?」と聞くと「はい」と答えると思いますか?」
と聞いて,「はい」と答えたら,そっちがX、「いいえ」と答えたら,違うほうの道がX。

28 名前:132人目の素数さん投稿日:02/07/13 17:55
>>27
どちらか一人しか道を知らないのに、何でそれでいいんだYO!

29 名前:27投稿日:02/07/13 18:30
>>28
>しかし、その2人はどちらがX村への道かを知っています
って書いてあるだろーが.


30 名前:27投稿日:02/07/13 18:39
そのまえに,01がワカラン

31 名前:132人目の素数さん投稿日:02/07/13 18:42
4回だべ

32 名前:132人目の素数さん投稿日:02/07/13 20:16
A,B,C,の 3 人がいて,
この中には“正直者”“嘘つき”“いい加減な人”が 1 人ずついる.
この 3 人に何度か質問をして,3 人がそれぞれどれであるかを決定したい.
ただし以下の条件に従うものとする.

質問は必ず“YES”か“NO”で答えられるものに限る.

3 人はお互いに誰がどの種類の人か知っている.

3 人のうち一度に 1 人にしか質問できない.

“正直者”は必ず自分の知るかぎり正しい答を答える.
つまり,“YES”か“NO”かのうち,
自分の知識の範囲内で,嘘であるとはわからない方を答える.
どちらを答えても嘘とは決まらない場合には,
どちらかを勝手に択んで答える(下記の例の 3 つ目を参照).

“嘘つき”は必ず自分の知るかぎり間違った答を答える.
つまり,“YES”か“NO”かのうち,
自分の知識の範囲内で,本当であるとはわからない方を答える.
どちらを答えても本当だとは決まらない場合には,
どちらかを勝手に択んで答える.

“いい加減な人”は質問に関わらず,
“YES”か“NO”かのうちどちらかを答える.

( 1 )

最低何回の質問をすれば,
必ず 3 人全員がどれであるか決定できるといえるか.

( 2 )

さらに“YES”と答えても“NO”と答えても
矛盾してしまう質問をしてもよいことにし,
そのようなときにはその人は“答えられない”と答えるものとする.
このときは最低何回の質問で,
必ず 3 人全員がどれであるか決定できるといえるか.

33 名前:132人目の素数さん投稿日:02/07/14 16:35
>>30
まず短いほうに500gやって塩を長いほうにやってつりあわせて
500gをとってそこに塩やればいいんじゃないのかな?

34 名前:132人目の素数さん投稿日:02/07/14 16:40
>>33
それは01じゃなくて00

35 名前:132人目の素数さん投稿日:02/07/14 16:42
>>34
スマソ

36 名前:27投稿日:02/07/14 17:11
>>31
やっぱり4回か,ってそのままじゃん
3回以下を考えようとしたけど,ないみたいね

37 名前:132人目の素数さん投稿日:02/07/15 21:18
http://web2.incl.ne.jp/yaoki/wari7.htm
これやってみて

38 名前:をっさん ◆96jfHsJM 投稿日:02/07/15 21:43
>>37
n(2n-1)=1から2n-1までの自然数の総和
∴数列の総和は、nの倍数であり
しょこうとしゅうこうの和はnの倍数である
隣の数同士の和もnの倍数
nが奇数の場合は、真ん中の数がn
∴任意にn倍の組み合わせを抜き取れる

39 名前:132人目の素数さん投稿日:02/07/15 21:43
名前騙ったままにしておくな。名無しに戻そう

40 名前:132人目の素数さん投稿日:02/07/15 21:51
>>38
ん?問題の意味誤解してないか?

41 名前:132人目の素数さん投稿日:02/07/15 22:40
>>32
>どちらかを勝手に択んで答える.

みんないい加減な人だな

42 名前:もっさん ◆96jfHsJM 投稿日:02/07/15 22:42
最後の一行の書き方が、言葉足らずで勘違いされたのかなあ?
それとも解ってないのか?あほが書き込んでるのか?
一応
∴任意にnの倍数となるn個の自然数の和が、必ず存在する
に訂正しておく

43 名前:132人目の素数さん投稿日:02/07/15 23:01
>>42
俺もなにか問題を勘違いしているように思うぞ


44 名前:132人目の素数さん投稿日:02/07/15 23:24
>>37
しばらく考えてみたが、分からず
反例がみつからないのでたぶん証明できるのだと思う
いい問題だと思うのでコピペ


2n−1個の任意の自然数がある。(nは自然数)
(2n−1個の内に、同じ自然数があってもかまわない)
その中のあるn個の自然数の和で、nで割り切れるものが必ず存在する。

そうであるなら証明を、そうとも限らないなら反例を示してください。


45 名前:132人目の素数さん投稿日:02/07/15 23:29
>>37
自分で考えろ。


46 名前:132人目の素数さん投稿日:02/07/15 23:33
>>44俺も考えたけど、ダメだった。
本当はテスト勉強しなきゃいけないのに、このままじゃ手がつかない。
って、これって逃避か?

47 名前:132人目の素数さん投稿日:02/07/16 00:15

2n−1個の自然数のうち、nで割り切れるものがn個以上あるときは
このn個を取ってくればよい。
よってnで割り切れるものがn-1個以下であるときを考える。
このときnで割り切れないものがn個以上ある。

といった感じで攻めてったらどうだろう(もしかしてミスリードしてたらスマソ)

48 名前:132人目の素数さん投稿日:02/07/16 21:33
age

49 名前:132人目の素数さん投稿日:02/07/17 21:22
ttp://ww2.wt.tiki.ne.jp/~keixx/quiz/quiz.html
面白かった

50 名前:132人目の素数さん投稿日:02/07/17 21:42
i~i=?
iは虚数単位な〜り〜。


51 名前:132人目の素数さん投稿日:02/07/17 21:43
びみょーにちがーう。
i^iでした。

52 名前:132人目の素数さん投稿日:02/07/17 21:57
50の~は某スレのでむぱが主張する累乗の事か?

53 名前:132人目の素数さん投稿日:02/07/17 21:59
>>20の数理パズルの34が全然分からん

54 名前:132人目の素数さん投稿日:02/07/17 22:00
>>52
素で間違えたのよ。
ちなみに、答えは実数になるそうで〜す。

55 名前:132人目の素数さん投稿日:02/07/17 22:20
>>49
30問中19問解けた。
これって普通何問ぐらい解けるの?

56 名前:デムパ投稿日:02/07/17 22:24
x=i^i
x^4=(i^i)^4=i^(4i)=(i^4)^i=1^i
4logx=ilog1=0
logx=0
x=1

i^i=1

57 名前:132人目の素数さん投稿日:02/07/17 22:34
>>56
どうもっ。

58 名前:132人目の素数さん投稿日:02/07/17 22:38
log使っちゃ、ダメダメ。

59 名前:132人目の素数さん投稿日:02/07/17 22:41
>>57
おい、マジかよ

60 名前:132人目の素数さん投稿日:02/07/17 22:42
i^4=1
4logi=log1=0
∴i=1

61 名前:132人目の素数さん投稿日:02/07/17 22:51
>>60
うーん、何かおかしい。

62 名前:132人目の素数さん投稿日:02/07/17 22:52
>>60
ってか結論がおかしい。w

63 名前:132人目の素数さん投稿日:02/07/17 22:53
>>60
だから、log使っちゃいけないって事?

64 名前:132人目の素数さん投稿日:02/07/17 22:54
いや、まって。
何がおかしいのかわからない。

65 名前:132人目の素数さん投稿日:02/07/17 23:15
(-1)^2 = 1
2log -1 = log 1 = 0
-1 = 1

66 名前:132人目の素数さん投稿日:02/07/18 00:01
>>64
logの定義域の問題なんだが(以下略)

67 名前:132人目の素数さん投稿日:02/07/18 10:33
>>64
i={e^(iπ/2+i2nπ); n∈Z}, 1={e^(i2nπ); n∈Z},
i^i={e^(-π/2-2nπ); n∈Z}, 1^i={e^(-2nπ); n∈Z},
∴ (i^i)^4 = 1^i (or 4*log(i^i)=i*log1={-2nπ; n∈Z})

多価関数はムズイ

68 名前:132人目の素数さん投稿日:02/07/18 10:43
>>1
ここなんかどう? 【明日地球が滅亡する確率】
http://science.2ch.net/test/read.cgi/math/1026789565/l50


69 名前:132人目の素数さん投稿日:02/07/19 12:25
2n個の自然数をA_(n)=kn+b_(n)  (k=非負の整数,0≦b_(n)≦n-1)とおく.
b_(n)を小さい順に並べたものをa_(n)とする。
すなわち
0≦a_(1)≦a_(2)≦・・・≦a_(2n-1)≦n-1

つづく

70 名前:132人目の素数さん投稿日:02/07/19 16:08
なぁ、質問ですが
上の三角形を見てください と、英語であって
上の三角形というのは5つぐらいのブロックが組み合わさって出来ていて、
下にも三角形があって、それは上の三角形のブロックをそのまま組み替えた者なのだが
何故か一マスが空いている。こんなことはあるのか!?

みたいな画像どこだっけ。結局あれの答えはウンコ並なわけなんだが
アドレス忘れちゃった・・・。誰かおせーてくれ・・・

71 名前:132人目の素数さん投稿日:02/07/21 19:08
age

72 名前:132人目の素数さん投稿日:02/07/21 19:23
>>70
もっと丁寧に質問しないと誰も分からんと思うぞ。
とかいいつつ、俺は多分分かった。(と思う)
あれだろ、長方形に切れ目を入れて三角形やら台形やらに分割して、
組み合わせ直すと、元の長方形よりも面積が少し大きい長方形に
なるってやつだろ。
んで、タネ明かしすると、後でできた長方形には対角線に沿って狭い
スリットが出来てるってやつだろ。

73 名前:132人目の素数さん投稿日:02/07/21 19:36
>>72
・・・・。

74 名前:132人目の素数さん投稿日:02/07/22 08:55
結局あれの答えはウンコ並なわけなんだが

75 名前:132人目の素数さん投稿日:02/07/25 21:03
http://homepage2.nifty.com/ishikawah/s-j-r.html
わかんないよぅ

76 名前:132人目の素数さん投稿日:02/07/25 21:20
>>75
一応、機関士はスミスっていう答えが出た・・・。
思考経路は面倒だから省くけど。アタマのいい方、答え合わせ
おながいします。

77 名前:132人目の素数さん投稿日:02/07/25 21:35
機関士はスミスであってるよ
かんたん

78 名前:132人目の素数さん投稿日:02/07/25 21:38
「物理学者は数学と無縁でない」は正しいのか?正しいんならスミスが機関士だ

79 名前:132人目の素数さん投稿日:02/07/25 21:42
「8.制動手と物理学者は同じ都市に住んでいる。」
はダミーで関係ないよ

80 名前:132人目の素数さん投稿日:02/07/25 21:50
9よりスミスは火夫でない。スミスが制動手でないことを言えばよい。スミスが制動手だとする。
7よりスミス氏はシカゴに住んでいる。4と合わせるとジョーンズ氏はオマハに住んでいることになる。
すると6よりジョーンズ氏は物理学者ではない。一方5と8より物理学者はオマハに住んでいる。
これは矛盾である。

81 名前:132人目の素数さん投稿日:02/07/25 21:56
9.よりスミスは氏がないので(以下略)

82 名前:132人目の素数さん投稿日:02/07/25 21:58
「7よりスミス氏はシカゴに住んでいる」とはいえない
8の物理学者は乗客かどうか不明

83 名前:132人目の素数さん投稿日:02/07/25 22:04
曖昧な記述が多く問題が悪いね

84 名前:132回目の素因数分解投稿日:02/07/26 23:25
数Uと数Bの知識で解ける、難しめの問題おながいします。

85 名前:132人目の素数さん投稿日:02/07/26 23:35
数2とか数Bって、何やってるかわからないけど、手元の問題週からニ、三問
引っ張ってきてあげようじゃないか。

次のように定義された数列{a_n}が収束することを示し、その極限値を求めよ。
a_{n+1} = 6(1 + a_n) / (7 + a_n)

Sを次の性質を満たす素数からなる集合とする。a,b ∈ S(aとbは異なる必要がない)のとき、
ab + 4 ∈ Sとなる。このとき、Sは空集合であることを示せ。

Qを有理数の集合とする。次の二条件を同時にみたすようなQからQへの関数を全て求めよ。
(i) f(1) = 2
(ii) 全てのx, y ∈ Q に対して、f(xy) = f(x)f(y) - f(x+y) + 1

86 名前:132人目の素数さん投稿日:02/07/26 23:37
>84
一辺が1の正四面体,立方体,正十二面体をそれぞれ平面に正射影した領域面積の最大最小

87 名前:85投稿日:02/07/26 23:45
最初の問題を解いてみたら、少し問題に不備があるような気がするので、a_0 = 0 と、条件をつけてみる。

88 名前:132人目の素数さん投稿日:02/07/27 01:21
>>85
y=6(1+x)/(7+x)のグラフがx≧0で
単調増加であることより、a_nは増加数列。

また、a_n≦2 ⇒ a_{n+1}≦2 なのでa_nは有界。
従ってa_nは収束する。

漸化式より、収束値は-3または2だが、以下略

2番目と3番目は、ちょっと考えたが、わからん。

89 名前:132人目の素数さん投稿日:02/07/27 07:44
>>85
3番目は、数学オリンピックの過去問に似たようなのがあった気がする
もし記憶どおりなら普通の工房には解けない罠


90 名前:132人目の素数さん投稿日:02/07/27 08:10
>>85 3番目
y=1 とすると f(x+1)=f(x)+1 より x が整数のときは f(x)=x+1

91 名前:90投稿日:02/07/27 09:21
続き
y=1/x (x は0でない整数) とすると (中略) f(1/x)=1/x+1
x=m,y=1/n (m, n は整数,n≠0) とすると (中略) f(m/n)=m/n+1

92 名前:132人目の素数さん投稿日:02/07/27 09:38
ある1本の不思議な木がありました。
この木は1年目に1m成長し、次の年は前年の
半分だけ成長します。
さて1000年目、この木は何mになっているでしょうか?

93 名前:132人目の素数さん投稿日:02/07/27 09:49
約L+2m(L+2-0.5^1000)

94 名前:132回目の素因数分解投稿日:02/07/27 21:37
問題どうもです。
>数2とか数B
虹関数、三角比、三角関数、確率、数列、
指数・対数関数、ベクトル、複素数平面、
微分、積分etc.etc...

ただし分数や三角関数の微積、三角形の5心、コンピュータ、数列・数式の極限
とかはダメですね・・・すいません。

95 名前:132人目の素数さん投稿日:02/07/31 14:16


96 名前:132人目の素数さん投稿日:02/08/01 17:24
このきなんのききになるき

97 名前:132人目の素数さん投稿日:02/08/04 07:07
この穴なんの穴きになる穴

98 名前:132人目の素数さん投稿日:02/08/17 10:32
>>32の解答はまだか?

99 名前:132人目の素数さん投稿日:02/08/17 12:24
自分で考えろ。
とりあえず、Aに
お前は『Bはいい加減な人か?』と聞かれたら『はい』と答えるか?
と訪ねたらどこまで条件が絞れるか考えろ。


100 名前:32投稿日:02/08/17 12:43
(1)が3回、(2)が2回です。

101 名前:132人目の素数さん投稿日:02/08/17 16:26
>>75
遅レスですいませんが、自分の考えたのと>>80とが合わない。
それに、ジョーンズ氏はオマハに住んでいるとはいえないでしょう。

自分の論理だと機関士はスミスと出た。
4.と7.より制動手はスミスかジョーンズなんだけど、
6.からジョーンズ氏は数学者と無縁だから、ジョーンズ氏は
物理学者の住んでいないところの人。
一方、8.から制動手は物理学者と同じ都市の人。
したがって、ジョーンズ氏は7.と5.からシカゴの人。
スミスが制動手で、ジョーンズが機関士。


102 名前:132人目の素数さん投稿日:02/08/17 16:46
だから問題が悪過ぎだって

103 名前:132人目の素数さん投稿日:02/08/17 16:47
KARLさん
あなたが
角度の問題で
私はモーリーの定理を使って解けましたよ

104 名前:132人目の素数さん投稿日:02/08/17 19:20
sage

105 名前:sage投稿日:02/08/17 22:59



106 名前:KARL ◆gjHKPQSQ 投稿日:02/08/18 00:40
>>103
角度の問題って、10度20度、40度20度の問題ですか?
あなたの解答ぜひ教えてください。

107 名前:132人目の素数さん投稿日:02/08/18 20:36
>>106
えっとですねー
先に必要な知っておくべき性質を書きます
http://www.mitene.or.jp/~tomo-s/morley/morley10.html
ここの
解答2の図で言う
GE=GF
っていう性質は知っていますか?
これ知らないと
ボクの証明は使えないので
証明は明日書きます
ただ
あまり自信ないので

108 名前:132人目の素数さん投稿日:02/08/18 20:40
証明間違っていてもおこらないででくださいね

109 名前:KARL ◆gjHKPQSQ 投稿日:02/08/20 01:23
>>107
>解答2の図で言う
>GE=GF
>っていう性質は知っていますか?

解答2にGE=GFの説明(証明)は載ってますね。
だから、知っている必要はない。
というか、煩を厭わず証明を繰り返せばよいのではないでしょうか。





110 名前:132人目の素数さん投稿日:02/08/20 14:16
>>32
2回。2人に同じ質問をする。

旅人:「あなたはどの人種の人ですか?と聞かれたときにあなたが答える文字列を
ShiftJIS形式でデコードしバイナリに変換して4bit毎に偶パリティビットを設けて
はい=1、いいえ=2として答えてください」

A:「はいいいえはいはいはいいいえいいえはいいいえはいいいえはいはいいいえ…」

旅人:「あなたはどの人種の人ですか?と聞かれたときにあなたが答える文字列を
ShiftJIS形式でデコードしバイナリに変換して4bit毎に偶パリティビットを設けて
はい=1、いいえ=2として答えてください」

B:「いいえはいはいはいいいえいいえはいいいえはいはいいいえいいえはいはい…」

以後文字列を復元し、
パリティが乱れていたら適当に答える人、正論理なら真実の人、負論理ならうそつきの人です。

111 名前:132人目の素数さん投稿日:02/08/20 14:31
>>32

1回目。
旅人「ここ、地球ですよね?」
A「わかりません」

2回目。
旅人「ここ、地球ですよね?(バールを握り締めながら満面の笑みで)」
A「ガクガクブルブル…」


112 名前:132人目の素数さん投稿日:02/08/20 21:08
>>110

113 名前:132人目の素数さん投稿日:02/08/21 00:55
>>110
いい加減な人は, 有限回の質問では
正直者やうそつきと同じ振る舞いをする可能性がある.

>>32
三回でいけそう.

一つ目の質問.
三人の内一人(Aとする)を選び, 残りの二人から適当に一人(Bとする)を選んで
そいつを指差しながら,
「あなた(A)はあの人(B)がいい加減な人かと聞かれたらYesと答えますか」.
# この質問で, いい加減な人を除外できる.

一つ目の質問がNoであった場合は, 指差した相手(B)に残りの質問をする.
Yseであった場合は, 指差さなかった相手(Cとする)に残りの質問をする.
この質問でいい加減な人を除外して, 正直者か嘘つきを確実に選べたことが重要.
以下はこの質問がYesだったとする.

二つ目の質問は,
「あなた(C)はAがいい加減な人かと聞かれたらYesと答えますか」.

i) この答えがYesであれば, Aはいい加減な人. 続けて三つ目の質問をする.
「あなた(C)は残りの人(B)が正直者かと聞かれたらYesと答えますか」
Yesであれば, Bは正直者, Cは嘘つき.
Noであれば, Bは嘘つき, Cは正直者.

ii) この答えがNoであれば, Bはいい加減な人. 続けて三つ目の質問をする.
「あなた(C)はAが正直者かと聞かれたらYesと答えますか」
i) と同様に, この質問でAとCのどちらが正直者でどちらが嘘つきか決定できる.


114 名前:132人目の素数さん投稿日:02/08/21 01:07
>>32
無限大

理由:
どんなに正しい質問を繰り返しても、いいかげんに答えた結果が正直者かうそつきと
同じ答えの繰り返しになる可能性があるから。

例えば、「あんたはいいかげんか」と3人に質問した結果、
YYNあるいはYNNの答えになるが、そのYYまたはNNを判別することは不可能。



115 名前:132人目の素数さん投稿日:02/08/21 01:11
>>32
あんたは生きてる?・・・・・と思うんだけど、ところで1+1は2だよね・・・・・・・まあそれは前振りなんだけど、
日本の首都は東京?・・・・・ですが、ではオーストラリアの首都はシドニーだっけ?・・・・ってちがうよなあ、
ところで・・・・・

という長い質問をして様子を見る。

116 名前:114投稿日:02/08/21 01:22
あんたはいいかげんか?

一人だけ違う答えをした方(Yならうそつき、Nなら正直)に

どっちかを指差してあいつはいいかげんか?

これでいいのか

4回でわかるか



117 名前:132人目の素数さん投稿日:02/08/21 04:57
ここにはかりがある。12枚の金貨のうち1枚が偽物であるという。本物と偽物の質量は
異なる。このはかりで偽物を見分けるには何回用いればよいか。

118 名前:投稿日:02/08/21 04:59
最小です

119 名前:132人目の素数さん投稿日:02/08/21 05:09
>>117
ものすごく激しく既出。
帰って寝ろ。

120 名前:132人目の素数さん 投稿日:02/08/21 05:44
>>118
レスありがとう過去ログさがしてみます。

121 名前:132人目の素数さん投稿日:02/08/22 03:41
>>32
の出題者はどこいった?

122 名前:132人目の素数さん投稿日:02/08/22 21:21
ここにいますよー!

123 名前:KARL ◆gjHKPQSQ 投稿日:02/08/23 02:39
a_1,a_2,...,a_n はそれぞれ 1 または -1 であるとする。さらに
S = a_1*a_2*a_3*a_4 + a_2*a_3*a_4*a_5 + ... + a_n*a_1*a_2*a_3 = 0
が成り立つとすれば、n は 4 で割りきれることを証明せよ。



124 名前:132人目の素数さん投稿日:02/08/23 06:36
[1](2次元)平面上に相異なるn個の点P1,P2,…,Pnがあり、
かつ、2点間の距離がすべて等しいという。nの最大値を求めよ。
[2](3次元)空間上に配置される場合はどうか?
[3]一般にm次元空間(m≧4)に配置される場合はどうか?

[1],[2]はいいとして、[3]が厨房の漏れには分からん。
n=m+1? まさかn=4(固定)だったりして♪

125 名前:132人目の素数さん投稿日:02/08/23 06:38
>>114
>例えば、「あんたはいいかげんか」と3人に質問した結果、
>YYNあるいはYNNの答えになるが、そのYYまたはNNを判別することは不可能。

俺もそう思ったんだ。しかし2人が見分けがつかなくなっても、
もうひとりに聞けば分かるんじゃないかと思ったんだ。

126 名前:124投稿日:02/08/23 06:43
もう一つ、ごくつまらない問題を。
最近3つ子素数が話題になっていたみたいなので。
多分そこに既に書かれているかも知れない。
(過去ログ読んでなくてスマソ)

連続する3奇数がすべて素数であるとき、その3素数を
三つ子素数と呼ぶことにする。
三つ子素数は(3,5,7)のみであることを示せ。

127 名前:132人目の素数さん投稿日:02/08/23 06:45
http://www.jokeimage.com/damasie/true_true.jpg
コレを説明して。

128 名前:132人目の素数さん投稿日:02/08/23 06:46
>>124
n=m+1じゃないかな。
m-1次元のときの解に対して、すべての点を通る超球の中で
半径がその2点間距離になるものを選べば、
その超球の中心にP[m+1]が置ける。
これを使えばn≧m+1がいえ「そう」。
n>m+1を排除するにはどうしたらいいかな?

それより>>124は[1]の証明ってわかるの?

129 名前:132人目の素数さん投稿日:02/08/23 06:49
>>127 上の斜辺が直線でない。激しく既出過ぎて呆れる。

130 名前:124投稿日:02/08/23 07:17
>>128
[1]P1とそこからrだけ離れたP2を中心とする、半径rの円を描く。
2円の交点をP3,P4と置くと、P3P4の距離≠rだから、最大値は3。
[2]P1とP2を中心とする半径rの球面を描く。交線は半径(√3/2)r
の円。この円に1辺rの正三角形は乗せられないので、最大値は4。

いかにも泥臭く、応用の利かない考え方か?(爆

131 名前:132人目の素数さん投稿日:02/08/23 07:22
>>130
筋道はそれでよさそう。
P1,P2を置いてから
P1中心、半径r=|P1-P2|のm次元球上でP2からrの距離にあるものは
P2中心のm-1次元球上のみとなる。
これだけを証明すれば帰納的に解ける

132 名前:132人目の素数さん投稿日:02/08/23 14:19
>>123
帰納法か!
でもn=5ですらS=0を証明できないんですけど…。
どうやればいい?

133 名前:132人目の素数さん投稿日:02/08/23 14:28
f(k)=k mod n
b(k)=a[f(k)]a[f(k+1)]a[f(k+2)]a[f(k+3)]
として
S=Σ[k=0〜n]b(k)

i)n=4m+1,4m-1のとき
b(k)は奇数より
nが奇数ならSも奇数。

ii)n=4m+2のとき、
もう少し?

134 名前:132人目の素数さん投稿日:02/08/23 15:05
aga

135 名前:2チャンねるで超有名サイト投稿日:02/08/23 15:08
http://s1p.net/qqwert

 携帯対応

男性より女性の書き込み多し
女性性65%男性35%割合です
穴場的サイトです。
幼い中高生直アポ直電
OL〜熟女迄の出会い
 聞ける穴場サイトです

136 名前: 投稿日:02/08/23 15:22
>>20
数理パズルの40がわかりません
パズル神の皆様教えてください
http://www.asahi-net.or.jp/~yx3k-nkzw/MApuzzle.html


137 名前:132人目の素数さん投稿日:02/08/23 15:41
>>123=KARL

(S(n) = a1a2a3a4+a2a3a4a5+...+ana1a2a3とする。)

ii)全ての自然数mにおいて∀ak(S(4m+2)≠0)の証明

ある自然数kにおいて
S(4k+2)≠0
ならば
S(4(k+1)+2)=S(4k+6)≠0
を示す。

S(4k+2) = a1a2a3a4 +a2a3a4a5 + ... + a[4k+2]a1a2a3
偶数個項があるので2つづつ括る↓
S(4k+2) = a2a3a4(a1+a5) + a4a5a6(a3+a7) + …
+ a[4k-2]a[4k-1]a[4k](a[4k-3]+a[4k+1])――最後から2番目の項
+ a[4k+2]a1a2(a[4k+1]+a3)――最後の項

ところで、
S(4k+6) = a2a3a4(a1+a5) + a4a5a6(a3+a7) + …
+ a[4k-2]a[4k-1]a[4k ](a[4k-3]+a[4k+1])
+ a[4k+2]a[4k+3]a[4k+5](a[4k+1]+a[4k+6])
+ a[4k+4]a[4k+5]a[4k+6](a[4k+3]+a1 )
+ a[4k+6]a1 a2 (a[4k+5]+a3 )
(続く)

138 名前:132人目の素数さん投稿日:02/08/23 15:42
>>137つづき)
T(k)を次のように置く。
T(k) = S(4k+6)-S(4k+2)
= a[4k+2]a[4k+3]a[4k+5](a[4k+1]+a[4k+6])
+ a[4k+4]a[4k+5]a[4k+6](a[4k+3]+a1)
+ a[4k+6]a1a2(a[4k+5]+a3)
- a[4k+2]a1a2(a[4k+1]+a3)

ここでS(4k+6)=0と仮定すると、
どんな数列akを持ってきてもT(k)=-S(4k+2)、つまり
T(k)≠0が成り立つはずである。
ここで、T(k)=0を満たすような数列akという反例を全パターン検索で示せれば、
S(4k+6)≠0が示せる。

さらにS(6)=0を満たすような数列akがないことを全パターン検索で示せば、
数学的帰納法により、ii)が示せ、
>>133と合わせれば目的の命題の証明が完結する。

こんなのでどうでしょうか
力づくな要素がありますが。

139 名前:132人目の素数さん投稿日:02/08/23 16:16
>>128
n次元の時、m個だとすると
|P(i)-P(j)|=|P(2)-P(1)| (i>j,(i,j)≠(2,1)
この条件式の数は(m-2)(m+1)/2個(m>2)
p(1),p(2)...,p(m)にはmn個の未知数が含まれる
このうち一つを原点であるとして良い。
従って未知数の数は(m-1)n個
解が存在する為には(m-1)n>=(m-2)(m+1)/2
が必要
2mn-n>=m^2-m+2
m^2-(1+2n)m+2+n<=0
nに対してmが↑を満たせば、解が存在する可能性が高いと思われるが.



140 名前:138投稿日:02/08/23 16:21
全パターン検索してみました。
・T(k)=0を満たすような数列akという反例
a1=a2=a3=a[4k+1]=a[4k+2]=a[4k+3]=a[4k+4] = 1
a[4k+5]=a[4k+6] = -1
のときにT(k)=0になります。

・∀a1,a2,a3,a4,a5,a6に対し、S(6)=0
ありませんでした。

これにて証明終わり。
4色問題のような力づくの証明ですいません。

141 名前:132人目の素数さん投稿日:02/08/23 16:32
>>139
ひとつ置くことによって
点の置ける範囲がn次元球上からn-1次元球上に縮退していって、
最後に円となって、回転による合同性から1つしかない、でいいんじゃないの?

142 名前:132人目の素数さん投稿日:02/08/23 17:20
>>136
http://science.2ch.net/test/read.cgi/math/1025371587/

143 名前:136 投稿日:02/08/23 17:36
>>142
さんくす

144 名前:132人目の素数さん投稿日:02/08/23 21:11
前に見たことあるんですけど(半年くらい前かな?)
MITの入試問題で、宇宙人の指の数の問題しりませんか?

145 名前:132人目の素数さん投稿日:02/08/23 22:05
>>144
あったねー
何進数かを考えるやつでしょ?
確か文系の学生が答えたんだっけ?

146 名前:KARL ◆gjHKPQSQ 投稿日:02/08/24 02:17
「不変なるものに着目せよ。」あるいは「不変なるものを見つけ出せ。」
という標語を思い出してください。
S=a1a2a3a4 + a2a3a4a5 + ... + ana1a2a3
は a1〜anが 1 になったり -1 になったりするごとに変化するわけで
すが「変化の中に不変あり。」という言葉を思い出すと...
一度にみんな動き出すとわかりにくいから、たとえばa4だけ変化したとします。
a4 は -a4 に変わります。かかわる項はa1a2a3a4, a2a3a4a5, a3a4a5a6,
a4a5a6a7 の4つだけ。変化はそれぞれ 2 または -2 (1から-1 または
-1から1) だからSの変化は±2±2±2±2 = 0または±4または±8
これはどの a を動かしても同じ結果になるから mod 4 で考えれば
a1〜anが どう動いてもSの値は不変(=0)です。a1〜anのすべてを 1 とすれば
S = n だから n = 0 (mod 4)となります。

147 名前:132人目の素数さん投稿日:02/08/24 15:52
>>145
そうです!それです!

148 名前:132人目の素数さん投稿日:02/08/24 16:00
[QUIZ] MIT の入試問題
http://cheese.2ch.net/math/kako/1007/10074/1007404903.html

149 名前:132人目の素数さん投稿日:02/08/24 16:05
>>146
なるほど。順列の互換の符号の唯一性と同じような議論ですね。

150 名前:132人目の素数さん投稿日:02/08/24 16:43
>>148の2の問題の答えは「11本以上」だよね。なぜか誰も
つっこんでないけど。

151 名前:KARL ◆gjHKPQSQ 投稿日:02/08/25 01:56
Morleyの定理を使った証明はどうしたのでしょう?

152 名前:132人目の素数さん投稿日:02/08/25 20:23
http://science.2ch.net/test/read.cgi/math/1015329951/7

153 名前:132人目の素数たん投稿日:02/08/25 20:25
152のリンク先ab=imaiダターヨ・・・
漏れの予想見事的中だよ・・・

154 名前:132人目の素数さん投稿日:02/09/04 10:12


155 名前:132人目の素数さん投稿日:02/09/05 09:08


156 名前: ◆6lcpBTbA 投稿日:02/09/22 21:49
では面白い問題をひとつ。
さいころがひとつある。
これを振って24人の中から公平に1人を選びたい。
さいころは何回振ればよいか?

157 名前:132人目の素数さん投稿日:02/09/22 21:49
age

158 名前:132人目の素数さん投稿日:02/09/22 22:05
>>156
以前類似問題があったような。1個。

159 名前:132人目の素数さん投稿日:02/09/22 22:07
>>156
最大3回、期待値7/3回になる方法:

まず2回振る。出目が11-46なら、この時点で24人の中から一人決定。
出目が51-66なら、その時点で二人に絞って、3回目の偶奇で決定。

最大、期待値ともにこの方法が最小でつ。

160 名前:132人目の素数さん投稿日:02/09/22 22:30
すごい勢いで既出であろう問題


ある数を本当は100倍するところを、間違えて10で割ってしまったら
1億になりました。

正しい答えはなんですか?

161 名前:156投稿日:02/09/23 00:09
>>159
ブー。
それより少ない解があります。

162 名前:156投稿日:02/09/23 00:13
>>158
類題みつけちゃいました。
アイデアとしては既出ですね。すまそ。

163 名前:132人目の素数さん投稿日:02/09/23 00:39
>>161
さいころを振る を定義しる。

164 名前:163投稿日:02/09/23 00:44
>>161
さいころと同じサイズのくぼみを作っておいて、
ソコに向かってさいころを投げることを
振ると呼んでいいの?

165 名前:132人目の素数さん投稿日:02/09/23 06:20
二回といいたいのか。


166 名前:132人目の素数さん投稿日:02/09/23 06:24
何でもいいなら一回。


167 名前:132人目の素数さん投稿日:02/09/23 10:14
>>160
千億

168 名前:161投稿日:02/09/24 01:30
か、解答は汝の心の中に…。

…ぐはっ

169 名前:KARL ◆gjHKPQSQ 投稿日:02/10/01 00:34
f(x)=x^2+x+41とします。
f(x)が合成数となるような整数xが40個連続する例をあげて下さい。
f(40)=41^2, f(41)=41*43 というわけで 40,41 は2個連続する例
です。


170 名前: 投稿日:02/10/02 02:34
>>156
もしサイコロが宇宙空間等の無重力な場所にあれば観測者が動けばサイコロを振る必要はない。
よって0回。

なんて言ってみるテスト。

171 名前:132人目の素数さん投稿日:02/10/02 06:44
>>169
名前を見て一瞬デジャヴかと思いました。
ていうか、もろデジャヴです(既視感でなく既視だっつうの)。ご無沙汰でしたが・・・

172 名前:132人目の素数さん投稿日:02/10/02 07:03
じゃんけんで決めればサイコロは振らなくてもよい
サイコロを振るの定義をしなくてはこんな答も出てきてしまうのだが‥
とかいってみる

173 名前:132人目の素数さん投稿日:02/10/02 11:45
>サイコロ
「ます」みたいなのを斜めに置いてその中に放り込めば良い。
頂点が8通り、さらに向きが3通りで、結局24通り等確率で現れる。

174 名前:132人目の素数さん投稿日:02/10/02 12:02
S村には犬を飼っている男が10人いました。10人の男は一匹ずつの犬を飼っていたのですが、
この犬はどれも獰猛で悪賢く、飼い主の見ていないところで他人にかみついてばかりいました。

どの男も、他人の飼っている犬が人を噛んでいる光景は目にしたことがあるのですが、
困ったことに全員が「自分の犬だけはおとなしくて賢いから人にかみついたりなどしない」と思っていました。

ある日、犬嫌いで有名な村長は、見かねて、この10人の男たちを呼び出し次のように言いました。
「自分の犬が他人を噛んだことがあるならば、即刻、犬を殺してしまうこと。
もし、処分を怠った場合は、飼い主を厳罰に処す。」
猶予は10日間与えられ、意地悪な村長はさらに
「お前達はみな、他の飼い主の犬が人を噛んだことがあるかどうかを正確に知っている。
また、お前達の飼い犬のうち、少なくとも一頭は人を噛んだことがある。」
とも教えました。

10人の男たちは、毎朝犬を連れて同じ公園を散歩していたので、朝になれば誰が犬を処分し、
誰がまだ犬を処分していないか、が解ります。次の日(2日目)の朝、
ある男は誰の犬も処分されていないのを見て「みんな自分の犬がどこで何をしているのか知らないのだな」
と考えました。しかし同時に「もしかすると、俺の犬も誰かを噛んだことがあるのかも知れない」とも思えます。
罪もない自分の犬を殺すのは絶対にイヤですが、自分が厳罰をうけるのも絶対に避けたいのです。

10人の男が充分に論理的であるとすると、この犬たち、そして男たちの運命はどうなるでしょうか?

>>20のリンク先からの引用だけどいい問題だ〜

175 名前:132人目の素数さん投稿日:02/10/02 12:29
>サイコロ
目を閉じて24箇所あるカドの一つを選ぶ。
振ってはいない。0回。

176 名前:132人目の素数さん投稿日:02/10/02 18:49
だれか169を解いてクレ

177 名前:156投稿日:02/10/02 20:05
>サイコロ問題
解答は一応1回です。>>164>>173は正解。
ちなみに
・三角錐に放り込んで8頂点×3方向=24通り
・V字谷に放り込んで12辺×2方向=24通り
・サイコロと同じ大きさのくぼみに投げ込んで6面×4方向=24通り
っていうのが模範解答です。

極論で行けば「振る」という行為を考えると確かに0回でもできますし、
粘着テープや粘土の上に投げて、向いた方向によって何通りにも分けたりはできますが
まぁ現実的には1回で24通りが妥当な答えではないでしょうか。

面白いのは上の3つの決め方全てが24通りという場合の数を示すところです。
もしかすると任意の正多面体の頂点、辺、面の数と方向の数の積は常に一定?

178 名前:156投稿日:02/10/02 20:37
・罪もない自分の犬を殺すのは絶対にイヤ
・自分が厳罰をうけるのも絶対に避けたい

これら2つの条件は自分の犬がguiltyだがそれが推測されないときに矛盾が起こる。
ので、100%自分の犬がguitlyなときにだけ処分すると仮定して考える。

1日目の思考
he doesn't know guilty dogs → he kills dog
(自分の犬がguiltyであるとしか考えられないから)
he know guilty dogs → he doesn't kill dog
(自分の犬がnot guiltyの可能性があるから)

で、2日目の朝誰も処分してなかったのだから,
全員少なくとも1匹〜9匹の他人の犬がguiltyだと知っていると推測される。
ここでguilty dogは自分の知っているもののみか、
自分の知っているものと自分の犬かの2通りになる。

2日目以降は飼い主同士の情報交換がない限りこれ以上推測されないので、
答え
10日目には犬は全員無事&guilty dogの飼い主(1人〜10人)が厳罰。

こんなんでどうでしょ

179 名前:163投稿日:02/10/02 20:44
>>もしかすると任意の正多面体の頂点、辺、面の数と方向の数の積は常に一定?
もちろん。
どれも辺の数の二倍になる。

180 名前:174投稿日:02/10/02 22:28
>>178
それは問題の意味を取り違えていると思われ。
この問題の前提は、

「事実として全ての犬が獰猛であり、
また、10人の飼い主はみなお互いに、自分以外が飼っている9匹の犬は獰猛だということを知っている。」

だよ。

>100%自分の犬がguitlyなときにだけ処分すると仮定して考える。
うん、この仮定は確かに必要かな。

181 名前:178投稿日:02/10/02 23:32
>>180
>どの男も、他人の飼っている犬が人を噛んでいる光景は目にしたことがあるのですが、

ここがその条件かな。じゃあ犬全員無事、飼い主全員厳罰?

182 名前:132人目の素数さん投稿日:02/10/03 13:08
>>174
うーん、どこが面白いのかな?
とくにややこしいところはでてこないけど。


183 名前:132人目の素数さん投稿日:02/10/03 16:59
条件を素直に解釈すれば
10日目に犬が10匹とも処分されるってことだろ

噛む犬が1匹の場合で考えれば飼い主は当日に気付く
噛む犬が2匹の場合だと飼い主は1日目(翌日朝)に気付く
それを拡張していけばいいだけのことじゃん

184 名前:132人目の素数さん投稿日:02/10/03 18:41
>>180
その前提で考えると、2日目の朝、すべての飼い主は犬を処分したことが分かる
ぞ。。。
>10人の飼い主はみなお互いに、自分以外が飼っている9匹の犬は獰猛だということを知っている
つまり、
・論理的な飼い主達は他人の思考を読み、その人以外の(自分のを含めた)9匹は獰猛であることが分かる。
・自分から見てその人の犬も獰猛であることが分かる。

10人とも同様に考えるので、2日目の朝にはどの犬も処分される。・・・(答)

>>183
訳が分からんぞ。。。

>条件を素直に解釈すれば
>10日目に犬が10匹とも処分されるってことだろ
どこにそんな文意が書いてあるのか?

>噛む犬が1匹の場合で考えれば飼い主は当日に気付く
>噛む犬が2匹の場合だと飼い主は1日目(翌日朝)に気付く
そう考えた理由は?


185 名前:132人目の素数さん投稿日:02/10/03 19:55
>>156
六面体サイコロの辺は12本ある。
その一本を方向も含めて目をつぶって選んで決定する。
(men nihasawarenaimonotosuru)

186 名前:174投稿日:02/10/04 01:40
>>182
面白くないですか。。。
すいません。

>>183
俺と同じ結論です。

>>184
俺が自分で書いた
>10人の飼い主はみなお互いに、自分以外が飼っている9匹の犬は獰猛だということを知っている
この文章はなんか変だな。
「お互いに」なんて言葉はいらないな。
飼い主同士は情報交換をしないわけだし、まぎらわしい表現だった。

187 名前:132人目の素数さん投稿日:02/10/04 08:44
>>184>>186
うーん。僕もわからない。
>噛む犬が1匹の場合で考えれば飼い主は当日に気付く
>噛む犬が2匹の場合だと飼い主は1日目(翌日朝)に気付く
これ解説して。

188 名前:183 ◆vznfj8ZywE 投稿日:02/10/04 09:18
ホントにわけがわからないのか?
まあいいや、ネタでないことを信じて解説。

>噛む犬が1匹の場合で考えれば飼い主は当日に気付く

村に獰猛な犬が1匹しかいない場合を仮定して考えてくれ。
その場合は、飼い主以外はどの犬が獰猛なのかを知っているし
飼い主だけは、獰猛な犬はいないと思っている。

ところが獰猛な犬は少なくとも1匹以上いるのだから
飼い主は(十分論理的だから)村長の話があったその瞬間
「そうかオレの犬が獰猛だったのだ」と気付くんだよ。
そして獰猛な犬の飼い主はその場で犬を処分をする。

189 名前:183 ◆vznfj8ZywE 投稿日:02/10/04 09:20
>噛む犬が2匹の場合だと飼い主は1日目(翌日朝)に気付く
問題文と日付の解釈がちがっていたのでまず訂正。翌日朝は2日目と考えてくれ。

さて今度は獰猛な犬が2匹いると仮定して考えてみる。
それぞれの飼い主は互いに相手の犬が獰猛なことを知っているので
獰猛な犬は1匹だと思っている。
さて、翌朝犬の散歩に行ってみると、お互いの犬がまだ処分されていない
事実を目の前にして、次のように考えるんだ。

「奴の犬はなぜ昨日のうちに処分されないのか?
 オレは村には奴の犬が村でただ1匹獰猛なことを知っている。
 そして奴はそのことは知らない。村には獰猛な犬はいないと思っていたはずだ。
 ところが昨日の村長の話で、奴は自分の犬が獰猛だということに気付いたはず。
 この矛盾はどうして起こったのか?
 それはオレの仮定が、違っていたからだ。村にいる獰猛な犬はただ1匹ではない。
 オレの犬を含めて2匹いたのだ!!
 やつも当然(十分論理的なので)オレと同じに考えていたはずなのだから、
 今朝の散歩でオレの犬の無事を知るまでオレと同じに思っていたのだ。」

飼い主はそうして翌朝の散歩で、自分の犬の獰猛さに気が付きその日のうちに
自分の犬を処分する。
これが2日目。

190 名前:183 ◆vznfj8ZywE 投稿日:02/10/04 09:21
これで3日目以降は省略していいか?
まだ解説がいるか?

191 名前:174投稿日:02/10/04 12:55
>>189
うん、だいたい俺と一緒だ。
ただね、それは厳密な証明になってないと思うんだよね。
ちょっと以下を読んでみてほしい。

●獰猛な犬が1匹存在した場合、1日あれば全ての獰猛な犬が十分処分できる。

●「獰猛な犬がi匹存在した場合、i日目までに十分処分できる。(i=1,…,k)」と仮定すると、
 獰猛な犬がk+1匹存在していた場合どうなるか――
 ・k日目までに十分処分できれば、それでよし。(◆)
 ・k日目までに十分処分できなければ、仮定および>>189のような推論によりk+1日目に十分処分される。
 いずれにしろ、k+1日目までには十分処分できる。

以上により「獰猛な犬がn匹存在した場合、n日目までに十分処分できる」ことが示された。

ここまではいいんだよ。
でも逆に「n日で十分なのは分かった。だが本当にn日必要なのか?」って聞かれると答えに詰まってしまう。
上の証明で言えば、(◆)の状況が現れるのかどうかだな。
この「必要性」の証明が俺にはうまく出来なかった。

192 名前:183 ◆vznfj8ZywE 投稿日:02/10/04 19:29
せっかく出題が美しい(?)物語風なので、物語風に解説してみたかったのだが
ダメか?

n日目までの必要性については次の推論をすればうまく行く(と思う)

獰猛でない犬の飼い主(十分に論理的な)を仮定する。
その飼い主が、n-1日目までに得られる情報は
自分の飼い犬が実は獰猛であった場合に得られる情報と
なにか差があるか。
オレ的には今のところ有意差はないと考えている。
n日目朝になって初めて、前者は既知のn-1匹の獰猛な犬の処分を
後者は既知のn-1匹の獰猛な犬が処分されなかったことを知ることで
初めて両者の情報に差が出るのではないか?

ちょっと時間がないのでコレだけで勘弁してくれ。

193 名前:183 ◆vznfj8ZywE 投稿日:02/10/04 19:36
なんというか、つまり
両者の情報に差がなければ、n-1日までに自分の飼い犬を処分するのは
「罪もない自分の犬を殺すのは絶対にイヤ」
に反する。
さてあとは、両者の情報に差がないことを証明するだけなのだが‥

194 名前:( _ _)/スパゲティ投稿日:02/10/04 19:42
>>191
・全ての飼い主は、自分の犬が獰猛であると気づいたら即刻処分する
・全ての飼い主の思考は理論的で、完全で健全である
という条件を考えると、
「x日で十分処分できる」という語は「処分に少なくともx日かかる」という語と同義にならないでしょうか。


195 名前:183 ◆vznfj8ZywE 投稿日:02/10/04 19:52
「題意によりその情報の差はない」じゃダメ?

196 名前:183 ◆vznfj8ZywE 投稿日:02/10/04 23:58
こういうのでどう?
「飼い主同士の情報交換はなされない」を条件と考える。

犬の飼い主に提供される情報は、初日の村長の話と
翌朝以降の散歩での他の犬の生存のみである。

獰猛な犬の数をn匹とすると
n<1のとき
 村長の話「少なくとも一頭は人を噛んだことがある」から
 これはありえないので除外。
n=1のとき
 村長の話の日(1日目)で飼い主は気付く。
n≧2のとき
 1日目には、犬の飼い主は、その犬が獰猛であるかないかにかかわらず
 他の獰猛な犬について既知であるので、自分の犬が獰猛であるかどうかを
 知る手がかりはない。
 それぞれの犬の飼い主が、自分の知っている獰猛な犬の数と、実際に獰猛な
 犬の数が一致しているのかどうかを知るための情報は、n日目の散歩で
 自分の既知の犬が処分されているかそれとも処分されていないかを確認する
 以外の手段は存在しない。

以上のことにより、獰猛な犬がn匹であった場合、その飼い主が自分の犬が
獰猛であることを知るためには、n日の期間が必要である。
また、n日で十分なことは、>>188,189,191で既にわかっている。

よって、すべての獰猛な犬の飼い主はn日目(本題の場合は10日目)の散歩で
自分の犬が獰猛であることに気付き、それぞれの犬は処分される。

こんなもんでよかですかの?
あと、n日で十分なことについての3日目以降の拡張には異存はないですか?

197 名前:183 ◆vznfj8ZywE 投稿日:02/10/05 00:08
n=1のときに、獰猛な犬の飼い主は村長の話以前には
自分の犬が獰猛かどうかを知る手段はない(つまり1日が必要)
ってのを入れとかないといけませんな。すまん。

198 名前:132人目の素数さん投稿日:02/10/05 02:10
>それぞれの犬の飼い主が、自分の知っている獰猛な犬の数と、
>実際に獰猛な犬の数が一致しているのかどうかを知るための情報は、
>n日目の散歩で自分の既知の犬が処分されているか
>それとも処分されていないかを確認する以外の手段は存在しない。
ここの証明が必要かも。

自分の犬が獰猛であると仮定したとき及び
獰猛でないと仮定したときの2パターンについて、
他の飼い主の行動も考えて矛盾が起きないときに、
自分の犬が獰猛であるか否かの判断はできないと判断できる。

これを使えぃっ!

199 名前:132人目の素数さん投稿日:02/10/05 09:22
>ここの証明が必要かも。

なんで?
他の方法での情報の入手について問題には何も書いてないと思うけど‥
てことは、できないことを条件とするしかないんじゃないの?

200 名前:132人目の素数さん投稿日:02/10/06 09:00
age

201 名前:132人目の素数さん投稿日:02/10/06 13:56
獰猛な犬の問題、3日目が答えでは?
174の問題文では、1人は「他の9人の犬が自分の犬以外がすべて獰猛である」
つまり、「獰猛でない犬は自分が飼っている犬だけ」という結論に村長から、
話があった当日に気づくはず、それで考えると2日目の散歩の時に
「10匹犬がいる」ということで、自分の犬が獰猛と気づきその日のうちに
処分する。よって、答えは3日目だと思うんだが…。いかがなものか?

202 名前:( _ _)/スパゲティ投稿日:02/10/06 15:06
>>201
いや,3日目にはわからないよ。
10日目にやっと全部の犬が処分されるのは、
獰猛な犬の飼い主(全員)が

「俺の犬が獰猛でないとすると、真に獰猛な犬は9匹。獰猛な犬の飼い主(俺以外)は、こう考えたはずだ。
 『自分の犬が獰猛でないとすると、真に獰猛な犬は8匹だ。獰猛な犬の飼い主は、こう考えたはずだ。
  【自分の犬が…

    『自分の犬が獰猛でないとすると、真に獰猛な犬はいない。
     しかし、村長の話から獰猛な犬は存在する。あ、俺の犬か。』
   こう考えたはずだ。しかし奴の犬は2日目に処分されなかった。
   それは俺の犬が獰猛でないという仮定が間違っていたからだな』
  こう考えたはずだ。しかし奴の犬は3日目に処分されなかった。
  それは俺の犬が獰猛でないという仮定が間違っていたからだな』
 …
こう考えたはずだ。しかし奴の犬は9日目に処分されなかった。
それは俺の犬が獰猛でないという仮定が間違っていたからだな」

と考えなければならない。
すでに9匹の犬が獰猛だと思っていても、
推論の中の推論に入るときに獰猛な犬の推定数が減っていき、
同時に「自分の犬が獰猛でない」という仮定を翻すにはそれぞれ1日必要だからだ。

203 名前:KARL ◆.PgjHKPQSQ 投稿日:02/10/06 16:39
>>169へのヒント
n!+2,n!+3,n!+4,...,n!+(n-1),n!+n は
n-1個の連続する合成数。


204 名前:132人目の素数さん投稿日:02/10/07 00:46
それなりに盛り上がれたので >>174 の問題は
「面白い問題」と認定してみたいがどうよ?


205 名前:132人目の素数さん投稿日:02/10/07 11:47
なにか新しい問題クレ

206 名前:132人目の素数さん投稿日:02/10/07 11:58
41!は49桁とかなるんだがそれがどうしたっ

207 名前:132人目の素数さん投稿日:02/10/07 12:50
244 :132人目の素数さん :02/09/30 20:18
嘘ついちゃいけませんよ。
次の質問に、「はい」か「いいえ」で答えてください。

『あなたが次に口に出す言葉は、「いいえ」ですね?』


245 :132人目の素数さん :02/09/30 21:55
>>244
はい?


208 名前:132人目の素数さん投稿日:02/10/07 23:28
 ノウサギ、イエウサギ、ムカシウサギ、ナキウサギの4匹は、今週も月曜日
恒例の競争をしました。前回「1位が2匹」のような同順位の者はいませんでし
たし、それは今回も同様です。また、前回1位だったのは、ムカシウサギでは
ありません。
 さて、以下の発言では、前回と比べて順位が下がった者はうそをつき、そう
でない者は真実を語っています。
 4匹は前回と今回、それぞれ何位だったのでしょう?

ノウサギ「イエウサギは前回2位でした」
イエウサギ「ムカシウサギは今回2位です」
ムカシウサギ「ナキウサギは今回、順位が上がりました」
ナキウサギ「ノウサギは今回、順位が上がりました」

http://www.amazon.co.jp/exec/obidos/ASIN/4806109959/qid=1033999817/sr=1-8/ref=sr_1_2_8/250-6063690-6958656
より。正解は本を見てくれ。

209 名前:132人目の素数さん投稿日:02/10/08 03:20
>>208
宣伝かよ。

210 名前:132人目の素数さん投稿日:02/10/08 05:08
今回 1位ナキウサギ 2位ノウサギ 3位イエウサギ 4位ムカシウサギ
前回 1位ナキウサギ 2位イエウサギ 3位ムカシウサギ 4位ノウサギ



211 名前:数学意味梨投稿日:02/10/08 22:59
逝け小野田

212 名前:132人目の素数さん投稿日:02/10/09 20:08
▼こんなのいかが▼

0 < a ≦ b ≦ c ≦ d ≦ e ≦ f ≦ g ≦ h

である8個の実数 a〜h を、下の分数式の□に1個ずつ配して得られる
式の値のうち最大値を求めよ

 □   □
 ― + ―
 □   □
―――――
 □   □
 ― + ―
 □   □


侮れないかもよ

213 名前:ネオ宇治茶投稿日:02/10/09 20:10
http://matari.ichigobbs.com/anythingok/#1

214 名前:132人目の素数さん投稿日:02/10/09 21:03
>>212
いくらでも大きく出来るんじゃないの?

215 名前:132人目の素数さん投稿日:02/10/09 21:12
ABCDEF
×    4
―――――
FEDCBA

216 名前:132人目の素数さん投稿日:02/10/09 22:17
>>214
a…h は 0 < a ≦ b ≦ c ≦ d ≦ e ≦ f ≦ g ≦ h をみたす定数と思われ。


217 名前:132人目の素数さん投稿日:02/10/10 00:10
覆面算ソルバーってあったな

218 名前:132人目の素数さん投稿日:02/10/10 00:52
時代はソルバーよりプロポーザー。

219 名前:132人目の素数さん投稿日:02/10/10 23:43
>>212
候補を8C4個まで絞れたがそこから先が手ごわい…

220 名前:132人目の素数さん投稿日:02/10/11 08:44
4C2×4C2=36(通り)まで絞れたぞ

221 名前:132人目の素数さん投稿日:02/10/11 09:01
あれ?IDが・・・
自作自演がばれた・・・(汗)
では、さようなら


222 名前:132人目の素数さん投稿日:02/10/11 13:22
>>221

   ?



223 名前:132人目の素数さん投稿日:02/10/11 14:10
脳内IDが見えるタイプの人

224 名前:132人目の素数さん投稿日:02/10/11 18:39
なぜ将軍様を呼び捨てにすると政治犯になるか
小一時間かけて証明せよ。

225 名前:132人目の素数さん投稿日:02/10/11 18:55
>>212
とりあえずh以外全部1の場合を考えてみると

(h/1+1/1)/(1/1+1/1)=(h+1)/2
               ||∨
(1/1+1/1)/(1/h+1/1)=2h/(h+1)

だからとりあえずhは一番上にくると予想・・・あとどうしよう

226 名前:132人目の素数さん投稿日:02/10/11 19:14
gも上に置いとけefは分母の上か?分子の上か?・・・

227 名前:132人目の素数さん投稿日:02/10/11 20:21
>>226
数字の組み合わせによって、置く場所変わってくると思うぞ。
じっくり考えたんじゃなくて、ただの直感だけど。

228 名前:132人目の素数さん投稿日:02/10/11 21:20
補題

0<p≦q≦r≦s のとき、□/□+□/□ の□にp,q,r,sを1つずつ入れて
得られる値のうち、

最大値は s/p+r/q
最小値は p/r+q/s

役に立つかな

229 名前:132人目の素数さん投稿日:02/10/12 13:16
>>228
>>219,>>220あたりはそれを分かった上でのレスのような気がする

230 名前:132人目の素数さん投稿日:02/10/12 23:32
1,2,3,4,5,6,7,8でやってみたら777/64になった。

231 名前:132人目の素数さん投稿日:02/10/13 02:09
なんか数学っぽい問題ばっかだな〜。
「頭の体操になるような問題」のスレじゃなかったのか〜。

232 名前:132人目の素数さん投稿日:02/10/13 18:01
A氏とK氏がルーレットを使ってゲームをすることになった。赤、黒の出る確率は同じであるとする。
ルールはこうだ、
1)A氏が「黒、赤、赤」のように3つの並びを宣言する。
2)その後、K氏も同様に3つの並びを宣言する。
3)どちらかの言った並びが順序もそのままで連続して出るまで、何度でもルーレットを回し、
  先に言った並びの出た方を勝者とする。

さて、よく考えてみると実はこのゲームはK氏が圧倒的に有利なんですが、
A氏の立場で考えてみると、最も勝つ確率を高くするには、最初にどのような3つの並びを宣言すればよいでしょう?
また同様に、A氏が最も宣言してはならない3つの並びはどのような物でしょう?

 ※勿論、K氏は十分頭が良く、自分の勝つ確率を最大にするように3つの並びを宣言してくる。

233 名前:132人目の素数さん投稿日:02/10/13 20:54
>さて、よく考えてみると実はこのゲームはK氏が圧倒的に有利なんですが
この時点で降参です。「A氏が「黒、赤、赤」」の時に「黒黒赤」か「赤黒赤」って
言えばいいのかな。

234 名前:132人目の素数さん投稿日:02/10/13 20:56
>>232
おもしろそう。
A:「黒赤赤」ならK:「○黒赤」が有利、K:「赤赤○」が不利、とかそういうこと?

235 名前:132人目の素数さん投稿日:02/10/13 22:08
むずかしいね、俺もよくわからん
だれか説明をキボンヌ

236 名前:132人目の素数さん投稿日:02/10/13 22:27
赤赤赤か黒黒黒じゃ?
例えばA氏が赤赤赤。
そしたらK氏は赤赤赤で50:50。黒赤赤でも50;50。
他のもの言っても50:50・・・だよね?

ところでA氏が黒赤赤って言った時はどのくらいK氏が有利なのだろう。

237 名前:132人目の素数さん投稿日:02/10/13 22:32
>例えばA氏が赤赤赤。
>そしたらK氏は赤赤赤で50:50。黒赤赤でも50;50。
黒赤赤で50:50は変じゃないか?
黒赤赤の方が有利だと思うが。

238 名前:132人目の素数さん投稿日:02/10/13 22:36
うーん、その辺がよく分かってないんですが…、逝ったほうがいいですか?
なにやっても同じ確率に感じるのは変?

239 名前:132人目の素数さん投稿日:02/10/13 22:37
ああ、そうだった〜

240 名前:132人目の素数さん投稿日:02/10/13 22:39
K氏「赤黒赤」が有利では?

241 名前:132人目の素数さん投稿日:02/10/13 22:43
ちょっとしらみつぶしで探してみます。

242 名前:132人目の素数さん投稿日:02/10/13 23:24
A氏の赤赤赤はK氏が黒赤赤と言うと勝利確率は12.5%
同じく赤赤黒はK氏が黒赤赤と言って25%。
この二つは低すぎると思われ。
でも勝てる見込みが40%以上のことってないっぽい。
赤黒黒に対しては黒赤黒があるし、赤黒赤には赤赤黒・・・

というわけで勝てる組み合わせに赤黒赤。負ける組み合わせに赤赤赤を推します。
そういう自分が実は236・・・

243 名前:240投稿日:02/10/13 23:35
先回りできるか出来ないかって考え方で良いと思う。
自分もやっぱ負けるのは、「赤赤赤」だと思うよ。

244 名前:132人目の素数さん投稿日:02/10/13 23:48
先手の手は初めの色をAとすると
「AAA」「AAB」「ABA」「ABB」の4通り。
後手の手は「AAA」〜「BBB」の8通りになる。
よって4*8=32通りの確率を求めるだけでいいのだが…

245 名前:132人目の素数さん投稿日:02/10/14 00:07
これってゲーム理論?

246 名前:132人目の素数さん投稿日:02/10/14 00:08
>>245
ゲーム理論ぽくてオートマトンぽいけど、
結局は確率の問題ではないかと

247 名前:132人目の素数さん投稿日:02/10/14 00:13
赤黒を2進表記したときの遷移図を書いときまつ
http://rosso.1717.info/upload/data/sennizu.gif

248 名前:132人目の素数さん投稿日:02/10/14 00:28
>>232のゲームを言い換えると。

ゲームはこの8つのノードのどれかに「当たり」が現れ、
各状態で2本づつある道のどちらかを選んで移動していく。
プレイヤーはどこかの状態を選んで、
相手の選んだ状態に「当たり」が来る前に
自分の選んだ状態に「当たり」がくれば勝ち。

249 名前:132人目の素数さん投稿日:02/10/14 00:37
>>245
ミニマックス理論を使うけど、ゲーム理論と考えるほどの規模ではないと思う。
3目並べみたいなものじゃない?

250 名前:132人目の素数さん投稿日:02/10/14 05:13
ん〜。よくわかんない‥

251 名前:132人目の素数さん投稿日:02/10/14 13:32
240
赤赤赤で負ける理由は、
「一番最初にこれが成されなければ、黒の次に赤赤と並んだあとに、
赤が出なければならないわけだが、黒赤赤で先に負けてしまう。」
この説明では0、00は考慮していないんだけれどね。う〜ん

252 名前:132人目の素数さん投稿日:02/10/14 13:45
>251
なるほど。でもこの問題難しいな。

253 名前:132人目の素数さん投稿日:02/10/14 19:03
2000枚のカードが積んである。上から順に1,2,3、、、2000と番号が書いてある。
一番上のカードを下に送り、次のカードを捨てる。
という行為を繰り返したとき最後に残るカードは何か。




254 名前:132人目の素数さん投稿日:02/10/15 02:06
>>232
地道に計算しました
A氏「赤赤赤」−K氏「黒赤赤」…勝つ確率1/8
A氏「赤赤黒」−K氏「黒赤赤」…勝つ確率1/4
A氏「赤黒赤」−K氏「赤赤黒」…勝つ確率1/3
A氏「赤黒黒」−K氏「赤赤黒」…勝つ確率1/3
(赤黒逆も同様)


255 名前:名無し投稿日:02/10/16 23:39
ABCDEFGHの8つのアルファベットを辞書式に並び替えるとき
1234番目の単語は何になるか?

256 名前:132人目の素数さん投稿日:02/10/16 23:47
単語の定義をしる

257 名前:132人目の素数さん投稿日:02/10/16 23:58
辞書式の定義をしる

258 名前:リア工房投稿日:02/10/17 00:05
1・・・ABCDEFGH
2・・・ABCDEFHG
3・・・ABCDEGFH
4・・・ABCDEGHF
5・・・ABCDEHFG
6・・・ABCDEHGF

n≧2のとき、k=p(n)n!+p(n-1)(n−1)!+・・・+p(1)1!+1のとき、(0≦p(i)≦i−1)
右からk番目から文字を並べ替えるとして(p(n)+1)番目にくる文字にあたる。
1234=1*720+4*120+1*24+1*6+1*2+1+1=1*6!+4*5!+1*4!+1*3!+1*2!+1*1!+1となり、
よって、
右から6番目の数から並び替える。
辞書順に、2番目に当たる数はD。よって、ABD・・・・・となる。
のこった文字を並び替え、5番目に来る文字。よって、ABDH・・・・
同様に繰り返すと、ABDHEFGCとなる。よって、
1234番目に来る数はABDHEFGCとなる。


259 名前:132人目の素数さん投稿日:02/10/17 04:13
早とちるなよ‥
それはお前の決めたルールで解いただけだって‥

260 名前:( _ _)/スパゲティ投稿日:02/10/17 09:49
>>253
1953…かな?

>>259
まぁいいじゃん。

A、Bの2人が交互に自分の頭にピストルを当てて引き金を引く。
弾が出る確率はAが1/2、BがP。相手が先に死ねば勝ち。
A→B→・・・という順で撃つとすると、
PがいくつのときにA、Bの勝率が1/2に等しくなるか?

261 名前:( _ _)/テXテpテQテeテB投稿日:02/10/17 09:55
問題書きミスった…炒ってくr

262 名前:132人目の素数さん投稿日:02/10/17 12:15
韓国人が対馬を韓国領だと言い出してます!
http://academy.2ch.net/test/read.cgi/history/1029269877/l50

263 名前:132人目の素数さん投稿日:02/10/17 13:29
>>260
自信ないけど
う〜ん・・・・・・P=1かな?

264 名前:( _ _)/うどん投稿日:02/10/17 15:22
引っ掛け問題だったんだけど問題の出し方間違えてすぐ気づかれてしまいました

>>263
正解!

で、これAがP,Bが1/2の確率のときはどうなるんだろ
普通に難しい問題になるのかな

265 名前:132人目の素数さん投稿日:02/10/17 16:22
>>264
自信ないけど
う〜ん・・・・・・P=1/3かな?

266 名前:リア工房投稿日:02/10/17 17:57
学校で間違いに気づいたので訂正。

一個ずつずれるね。1234番目はACGDEFHB。

>>259
題意が違いますか?漏れは
「文字をアルファベット順に若い方から(=辞書に載る順番で)
並び替えて行くとき、1234番目にできる文字(単語?)を求めよ。」
と解釈しましたが。


267 名前:132人目の素数さん投稿日:02/10/17 18:00
悪人の最後の言葉でそいつの死刑の方法が決まる。
そいつの言っていることが正しいなら斬首刑。
そいつの言っていることが嘘なら絞首刑。
悪人が無罪になるにはどういった言葉を言えば良いでしょう。

268 名前:リア工房投稿日:02/10/17 18:04
>>267
「私は絞首刑です。」
・嘘なら絞首刑で「本当」となり矛盾。
・本当なら斬首系で「嘘」になり矛盾。


269 名前:あっはーなみこし投稿日:02/10/17 20:22
7日間で金利3割。一年間借り続けたらどうなる?
年率何パーセントでしょう?
実話。

◆\\\\\短期ヤミ金融業者ですが、何か?\\3\\◆
http://money.2ch.net/test/read.cgi/manage/1032843091/

270 名前: ◆MGCfgPOJXA 投稿日:02/10/17 20:34
>>269
8415万over?


271 名前:132人目の素数さん投稿日:02/10/17 20:37
(13/10)^(52+1.25/7)乗=881863.458300450037601096815919193倍。
年率約88186344.83%(w



272 名前:132人目の素数さん投稿日:02/10/17 21:08
1円借りたら1年後に9千万?

273 名前:132人目の素数さん投稿日:02/10/17 21:19
一年後に9十万。


274 名前:132人目の素数さん投稿日:02/10/17 21:23
ああそうか。アホでごめん。

275 名前:132人目の素数さん投稿日:02/10/17 21:46
88186245.83%では?

276 名前: ◆ncKvmqq0Bs 投稿日:02/10/17 21:53
100%=1倍

277 名前:253投稿日:02/10/17 22:05
>>260
違います。

278 名前:132人目の素数さん投稿日:02/10/17 23:42
1,3,5,...,1997,1999 :2
1,5,9,...,1993,1997 :4
1,9,17,...,1985,1993 :8
1,17,33,...,1969,1985 :16
1,33,65,...,1953,1985 :32
33,97,161,...,1889,1953 :64
ここまであってる?

279 名前:132人目の素数さん投稿日:02/10/18 01:57
全部二進数で表すと

カードが2000枚

?1が1000枚

?01が500枚

?001が250枚

?0001が125枚

?00001が63枚

?100001が32枚


100001が残る。10進数に直して33。

280 名前:132人目の素数さん投稿日:02/10/18 03:10
1953。


281 名前:132人目の素数さん投稿日:02/10/18 03:15
1942

282 名前:132人目の素数さん投稿日:02/10/18 04:02
漏れが高校生の頃考えた問題。面白くなかったらごめんなさい。

問い
三角形の三辺の長さがn,n+1,n+2となるような三角形Sを
考える。Sの面積が自然数となるnの値を小さい順に並べると、
規則のある数列になることを示せ。またその一般項を求めよ。

283 名前:132人目の素数さん投稿日:02/10/18 04:06
>>282
n+1=(2+√3)^m+(2-√3)^m
三辺をn-1、n、n+1とした方が計算しやすいよ。

284 名前:132人目の素数さん投稿日:02/10/18 04:47
>>283
4分でとはすごい、、でも数列になることを示す部分がないよ。


285 名前:132人目の素数さん投稿日:02/10/18 05:51
1年にその前の年に伸びた半分の長さのぶんだけのびる木がある。
初めは1mで一年後は1m50cmまた1年後は1m75cmになる。
1000年後には何cmになっているでしょう?

286 名前:肉体労働者投稿日:02/10/18 06:01
ちょっと待っててね
100+50+25+12.5+6.25+3.125+1.50625+0.753125+0.3765625+・・
待っててね

287 名前:132人目の素数さん投稿日:02/10/18 06:52
2mぐらい

288 名前:肉体労働者投稿日:02/10/18 06:55
ち。


289 名前:132人目の素数さん投稿日:02/10/18 09:30
今100mの高さだけど、毎年、ランダムに1mのびるかちぢむかする木がある。
100年後の高さと今の高さの差の期待値は何メートル?

290 名前:132人目の素数さん投稿日:02/10/18 10:39
ゼロメートルぐらい

291 名前:( _ _)/うどん投稿日:02/10/18 12:45
>>258
順列から自然数への全単射の作り方ですな。
http://www.ic-net.or.jp/home/takaken/nt/slide/hash.html
こことかおもしろいよ

292 名前:132人目の素数さん投稿日:02/10/18 16:16
>>260、280
正解です。

293 名前:( _ _)/うどん投稿日:02/10/18 22:30
>>292
n枚のカードなら、2n+1-2^kが残る。
(kはn≦2^kとなる最大の自然数)

294 名前:( _ _)/うどん投稿日:02/10/18 22:38
>>265
正解!
p/(1-r)=r/(1-r)を解けばよいね。(r=(1-p)/2)
ということは先に撃つほうは1/3の確率で弾が出るピストル、
後に撃つほうは1/2の確率で弾が出るピストルで撃つとすると公平なゲームになる。
俺だったらどっちとるかな…
先手かな

295 名前:132人目の素数さん投稿日:02/10/18 22:41
A君は24km/h、B君は18km/hで同時にP点を出発して、Q点で折り返すサイクリングを始めました。
A君はQ点に着いたあと10分間休んでP点へ引き返した所、Q点から12.5kmのところでB君と出会いました。
P点からQ点までの距離を求めなさい。

296 名前:132人目の素数さん投稿日:02/10/18 22:43
>>278(33,97,161,...,1889,1953 :64)続き
33,161,289,...,1825,1953 :128
161,417,...,1697,1953 :256
417,929,1441,1953 :512
929,1953 :1024
1953 :2048
終了。
政界です>>292

297 名前:132人目の素数さん投稿日:02/10/18 22:46
sequence (Rn)of rational numbers having a limit lim Rn that is an irrational number

超、超大至急にこの問題の解を知りたいんです!
数学の神様方、助けてくださいませ!

298 名前:132人目の素数さん投稿日:02/10/18 23:01
>>297
そもそも、問題になってないんだが。

有理数の数列Rnは、有理数ではない極限値 lim Rn を持ち、
(動詞なし)

299 名前:132人目の素数さん投稿日:02/10/18 23:07
>>298
つか、とっても紳士的な朝鮮人様が、


韓 : 日本人の知識迫ヘ test
作成時刻 : 2002.10.18 22:37:17

sequence (Rn)of rational numbers having a limit lim Rn that is an irrational number
定木解いて見なさい!!!!!!!クハハハハハハ


と素敵な出題をしてくださったわけなんです。
問題になってないわけなんですか?

300 名前:132人目の素数さん投稿日:02/10/18 23:43
>>297
フィボナッチ数とでも応えておけ。

301 名前:298投稿日:02/10/18 23:46
>>300
補足すれば、それの隣接2項間の比かな。
まあ隣接じゃなくてもいいけど

302 名前:132人目の素数さん投稿日:02/10/19 21:09
数列の問題作って見ました。工房の時作った問題。

(1) tを自然数とし、Xを実数とする。(X<t)
数列A[n],B[n]は次のように与えられるとする。
A[0]=0,B[0]=1
A[n+1]=A[n]+(X/t)B[n]
B[n+1]=B[n]-(X/t)A[n]
このとき、lim[t→∞]A[t]および、lim[t→∞]B[t]を求めよ。

(2) (1)において実数z,r(-r≦z≦r)をつかって、
A[0]=z,B[0]=(r^2-z^2)^(1/2)を入れたときどうなるか。説明せよ。

(3) tを自然数とし、Xを実数とする。(X<t)
数列C[n],D[n],E[n]は次のように与えられるとする。
C[0]=6,C[n+1]=C[n]+(X/t){C[n]-6}^(1/3)
D[0]=4,D[n+1]=D[n]+X/(t×{D[n]^(1/2)})
E[0]=0,E[n+1]=Σ[k=1〜n]X/{t×exp(E[k])}

において、lim[t→∞]C[t]、lim[t→∞]D[t]
および、lim[t→∞]E[t]を予測せよ。


303 名前:132人目の素数さん投稿日:02/10/19 21:51
スレ違い

304 名前:132人目の素数さん投稿日:02/10/19 22:25
(・∀・)スンスンスーン♪ ( ゚д゚)ハッ!

305 名前:132人目の素数さん投稿日:02/10/19 23:38
http://www3.to/jnr

306 名前:132人目の素数さん投稿日:02/10/20 11:10
双曲線 x^2-y^2=1 の第一象限部分をCとする
C上に点 (p,q) を取るとき、Cの 1≦x≦p の部分と x軸 および 直線 x=p
で囲まれた部分の重心座標を求めよ

…もしかしたら計算めんどいだけだったりして

307 名前:132人目の素数さん投稿日:02/10/20 12:13
>>306
これ基本関数で解けます?積分がすごいことになるんだけど・・・


308 名前:132人目の素数さん投稿日:02/10/20 16:05
306の計算、積分の変換がどうしても分からないんだったら
この曲線を左に45度傾けるとおなじみの関数y=1/(2x)になるので、
後はごり押しで解くというやり方がある。

これは2次方程式を2回程やって求まる範囲で1/2xを定積分すればいいだけなので
後はどうにかなると思う。

309 名前:132人目の素数さん投稿日:02/10/20 16:07
定積分すればいいだけ→定積分すれば後は四則演算だけなので

に訂正

310 名前:132人目の素数さん投稿日:02/10/20 19:33
次の問題くれ

311 名前:132人目の素数さん投稿日:02/10/20 19:49
>>310
そんな貴方にこの問題を
まず、適当に自然数を一つ考える
それが偶数なら2で割り
それが奇数なら3倍して1を足す
これを繰り返した時
最初に考えた数字がどんなのであれ最終的に1になるとこを証明せよ。

312 名前:132人目の素数さん投稿日:02/10/20 19:53
>>311
名前忘れたけど未解決問題と

313 名前:132人目の素数さん投稿日:02/10/20 20:00
a,bを 1/a+1/b=1,a>0,b>0 をみたす有理数とする。
このようなa,bの組{a,b}が、rによって定められるxの2次方程式
(r-1)x^2-(r^2)x+r^2=0
の2解{α,β}ですべて与えられるようにするためのrの条件は
rが 1<r≦2 の有理数の範囲で動けばよいことを証明せよ。

日本語が少しおかしいかもしれない・・・

314 名前:132人目の素数さん投稿日:02/10/20 20:02
>>311
ふう、やっと解けたよ・・・
ただ、ここに解答を書くにはあまりにもスペースがないので
ここに証明を記すことはできないが・・・

315 名前:132人目の素数さん投稿日:02/10/20 20:07
漢字、片仮名、平仮名、句読点だけを使って(アラビア数字の使用は不可)
体について説明せよ。


316 名前:132人目の素数さん投稿日:02/10/20 20:44
>>315
頭から足までをまとめていう語。身体。体躯。

317 名前:311投稿日:02/10/20 20:49
解いてage

318 名前:132人目の素数さん投稿日:02/10/20 20:57
>>317
やだ。自分で解け。

319 名前:311投稿日:02/10/20 20:58
>>318
そんな殺生な・・・

320 名前:132人目の素数さん投稿日:02/10/20 21:02
>>319
いま一所懸命がんばってるので、もそっとと待って栗。

ttp://science.2ch.net/test/read.cgi/math/1033052984/

321 名前:313投稿日:02/10/20 21:28
312のついででいいので解いてage

322 名前:321投稿日:02/10/20 21:31
311→312
間違いウツだsage

323 名前:132人目の素数さん投稿日:02/10/20 22:07
>>313
まずr=1で解いてみる
その時成り立たないから
r≠1となる
その後両辺を(r-1)で割る
すると
α+β=r^2/(r-1)・・・@
αβ=r^2/(r-1)・・・A
次に
1/a+1/b=1 を変形して
(a+b)/ab=1・・・B
@とAをBに代入すると
1=1となり rの値に関係なく成り立つ
あとは0<a、0<bとなるようなrの範囲を解くと
1<r≦2と出てくる

324 名前:132人目の素数さん投稿日:02/10/20 22:19
>>302
(1)lim(t→∞)A[t]=sinX
lim(t→∞)A[t]=cosX
(2)A[0]=r*sinY,B[0]=r*cosYとおくと、
lim(t→∞)A[t]=rsin(X+Y),lim(t→∞)A[t]=r*sin(X+Y)
lim(t→∞)B[t]=r*cos(X+Y)

(3)がわからん。。。


325 名前:132人目の素数さん投稿日:02/10/20 22:25
>>313
r=3の時
2x^2-9x+9=0となり
(x-3)(2x-3)=0
x=3,3/2
この2つの解を
1/a+1/bに代入すると1になるぞ
しかもちゃんと両方正の数だし有理数でもある。

326 名前:323投稿日:02/10/20 22:32
>>325
すまんちゃんと計算したら
1<rの有理数だったよ

327 名前:313投稿日:02/10/20 22:49
>>325
もちろんr=3でも成り立つが証明すべきことは1<r≦2ですべて得られることだから問題ない
>>323氏の解答に1<r≦2と2≦rでえられる解の組が一致することを証明すればよい。

328 名前:132人目の素数さん投稿日:02/10/20 22:56
a、b、cはそれぞれ異なる素数とする。 a*b>c
a*bをcで割った余りは1か偶数か素数である。

適当に考えたので反例あるかも。反例か証明プリーズ。
駄問だったらすいません。

329 名前:323投稿日:02/10/20 23:18
>>327
すまん、文盲だった(;´д⊂)
全ての組み合わせが1<r≦2ででてくるってことの証明は
r^2/r-1のグラフから出来るよ
極小値が2とグラフより
f(r)=r^2/r-1 (1<r) で出てくる値全てが
f(r)の1<r≦2の範囲で出てくる。ってとこかな

330 名前:132人目の素数さん投稿日:02/10/20 23:23
なんかスレの方向が違ってきたのだが。

331 名前:330投稿日:02/10/20 23:23
違ってきたのだが、しかも、ageてしまったのだな(泣)

332 名前:330投稿日:02/10/20 23:23
俺は。

333 名前:DJテクナナシー ◆bg2dvJlr8s 投稿日:02/10/21 00:05

乱数放送をみんなで解読しよう
http://ex.2ch.net/test/read.cgi/korea/1034982829



チェケラ!

334 名前:132人目の素数さん投稿日:02/10/24 13:34
http://www.kalva.demon.co.uk/index.html

335 名前:132人目の素数さん投稿日:02/10/24 15:05
[問]
3辺が整数である直角3角形がある。3辺の長さをひとつの変数で表しなさい。


ちなみに2変数では斜辺をZ残りの辺をX、Yとすると
XYZが整数m、n(m>n)を用いて、
X=m^2-n^2
Y=2mn
Z=m^2+n^2
とあらわせられるのは有名だね。


336 名前:132人目の素数さん投稿日:02/10/24 15:36
X=t-1
Y=2t
Z=t+1


337 名前:132人目の素数さん投稿日:02/10/24 16:24
X=t^2-1
Y=2t
Z=t^2+1


338 名前:132人目の素数さん投稿日:02/10/24 17:00
>>335
出題の文面が不適切。

339 名前:132人目の素数さん投稿日:02/10/24 23:05
三角柱を適当な平面できった切断面によってすべての形の三角形ができることを示せ。

340 名前:132人目の素数さん投稿日:02/10/24 23:23
>>339
面白い。けど簡単かな
まず無限の高さをもつ三角柱っていう前提が抜けているのでそれを補足。
かつスケールは関係なく内角の和が180°になる全ての3角の組み合わせが
表せるだけという点も補足。

高さの方向をZ軸にとり、
3つの直線と切る面の交点をA(原点としても一般性を失わないのでそうする),B,Cとして、
B,CのZ座標を変えると輸BC+唯CA+佑AB=180°なる全ての角度の組み合わせが作れることを証明
っていう流れかな。実際の角度求めるのめんどくさいからやらないけど。

341 名前:132人目の素数さん投稿日:02/10/24 23:32
>ちなみに2変数では斜辺をZ残りの辺をX、Yとすると
>XYZが整数m、n(m>n)を用いて、
>X=m^2-n^2
>Y=2mn
>Z=m^2+n^2
>とあらわせられるのは有名だね。

これって「星の王子さま」のサンテグジュペリが見つけた定理ってほんと?
「ファラオの問題」っていう本にそう書いてあったそうな…

342 名前:339投稿日:02/10/24 23:47
>>340
別に三角柱の高さは無限でなくてもO.K.なはずだよ。
三角柱の1つの角(頂点)に近い部分でカットすれば出来る。

ところで面白くかつ難しい問題のほうがいいの?

343 名前:132人目の素数さん投稿日:02/10/24 23:53
>>342
高さがどの一辺よりも十分小さいような三角柱でも?
正三角柱だった場合どうやって鈍角作るんだ?

344 名前:132人目の素数さん投稿日:02/10/24 23:54
>>343
角から取れば?

345 名前:132人目の素数さん投稿日:02/10/25 00:05
3角柱を倒して、上の辺をつまんでひねると多少は鈍角3角形が出きるが・・・

346 名前:132人目の素数さん投稿日:02/10/25 00:12
以下のようなゲームを行う。

・コインを投げて表が出たら1円もらう
・裏が出たら1円失う

資金0円からスタートして、資金が負になっても好きなだけゲームを続けることができるとする。
また、もし目標の金額がたまれば、即座にゲームをやめることができる。
例えば、ゲームを始めてから3回連続裏が出て、その次に1回表が出ると、資金は-2円になる。

これについて以下の各問に答えよ。

問1)8回ゲームをするまでの間に、資金が一度も1円を超えない確率を求めよ。
   8回より前にゲームを中断する場合は考慮しないものとする。

問2)このゲームを資金0から開始した人は誰でも、
   いつか必ず(確率1で)資金を+1000000円にすることが可能であるか、
   不可能であるか。答えだけ記せ。
   途中であきらめてゲームを中断する場合は考慮しないものとする。

問3)問2の解答を証明せよ。

347 名前:132人目の素数さん投稿日:02/10/25 01:22
e^(-n)とか出てくるんじゃ無いの?

348 名前:132人目の素数さん投稿日:02/10/25 01:36
問題
cos1°を10進数表記で少数展開したとき、少数の途中に2002という数列が並ぶことはあるか?
sin1°、tan1°ならどうか?

349 名前:132人目の素数さん投稿日:02/10/25 02:01
>>348
その問いに対してどういう風にアプローチする?
その辺考えた?

350 名前:343投稿日:02/10/25 03:12
>>344
ごめん、ホントにわからない。角からって?

351 名前:132人目の素数さん投稿日:02/10/25 03:45
344じゃないがおそらく角(かど)を角(かく)と読み違えてないか?

352 名前:343投稿日:02/10/25 04:15
いや、ちゃんと角(かど)と読んでいるが‥
どうやっていいのかホンキでわからない

353 名前:132人目の素数さん投稿日:02/10/25 04:18
一息つけよ
時間をおいて見直したら気づくことも多いしな…

354 名前:343投稿日:02/10/25 04:36
ん、今夜はとりあえず寝て、明日また考えてみる。
さんきゅ

355 名前:132人目の素数さん投稿日:02/10/25 13:41
f(x)は[a,b]上で連続かつ(a,b)上で微分可能であるとする。このとき、f(a)とf(b)が異符号で
(a,b)上f´(x)≠0ならば、f(x)=0は(a,b)内にただ1つの解を持つことを示せ。

356 名前:132人目の素数さん投稿日:02/10/25 13:51
>>355
マルチ

357 名前:132人目の素数さん投稿日:02/10/25 13:55
>>355
中間値の定理と平均値の定理を使えば明らかだろ。
臭いからアッチ逝け。氏ね。

358 名前:あふぉん投稿日:02/10/25 13:56
y=x^xのグラフってどんな感じになるの?

359 名前:132人目の素数さん投稿日:02/10/25 14:52
ttp://www.tomato.sakura.ne.jp/~rakkyou/sideT/index.html
このHPにある小学生の問題は本当に小学生の問題だったぞ(どっかの本に載ってた)。
一般化してみると奇麗な式になったね。
ちなみにオレは40秒でとけた。

注意:問題とけても書いてある通りにしない方がいいです。まぁそういうのが好きな方は別ですけど。


360 名前:132人目の素数さん投稿日:02/10/25 14:59
そりゃ小学生でも説けるだろ
一般化も小学生でも出来ると思うが
しかし何つーサイトだ

ところで 
>>339の問題って
角度が超小さくするためには無限じゃなくちゃ無理なんでないの?

361 名前:132人目の素数さん投稿日:02/10/25 15:01
ああ
ごめん
無限の必要ないわ

362 名前:132人目の素数さん投稿日:02/10/25 16:03
>>339
有限の高さでは無理ですよ
与えられた三角柱の底面の内角のうち最大のものの大きさをαとすると、
カドを切り取る方法では max( α, 90°) 以上の内角をもつ三角形をつくれません

363 名前:132人目の素数さん投稿日:02/10/25 20:54
そんなことはないよ。ひとつの角の側面同士はa度だが
底面と側面は90度あるからね

364 名前:132人目の素数さん投稿日:02/10/25 21:09
>>363
鈍角三角形つくってみな

365 名前:KARL ◆.PgjHKPQSQ 投稿日:02/10/25 22:58
>>328
>a、b、cはそれぞれ異なる素数とする。 a*b>c
>a*bをcで割った余りは1か偶数か素数である。
>
>適当に考えたので反例あるかも。反例か証明プリーズ。
>駄問だったらすいません。

反例 a=5,b=11,c=23

366 名前:ユカリ投稿日:02/10/25 23:09

プランテック製の「 RX-2000V 」を改造済み
にした、アイティーエス製の「 RX-2000V 」↓
http://user.auctions.yahoo.co.jp/jp/user/neo_uuronntya#.2ch.net/

現在、本当に人気がある様です。
≪宣伝ではありません≫

関連ホームページ↓
http://www.h5.dion.ne.jp/~gekitoku/
http://www.h4.dion.ne.jp/~gekiyasu/
http://www.h5.dion.ne.jp/~gekirea/
http://www.h4.dion.ne.jp/~shinsetu/
http://www.planning-instigator.com/silentpeople/best/index2.html

367 名前:132人目の素数さん投稿日:02/10/25 23:40
>>359
そーゆーのはだから問題だけ晒せば良かったんじゃないか、
得する奴は殆どいないんだしと、建て前として言っておく。

368 名前:132人目の素数さん投稿日:02/10/26 07:24
>>354
例えば正三角柱だとする。
上面の正三角形の三つの頂点をABCとする。
BCを1:10くらいに内分する点をPとする。
PAを結んだ線を切り口としてBの方向にできるだけ薄く削り取るように切断。
断面はどんな形ですか?

369 名前:132人目の素数さん投稿日:02/10/26 07:27
2つの隣り合った面の角度がαであるとき、
それらに交差する面で切ったときの角度は高々α以下である。

底辺が鋭角三角形である三角柱がもつ面の角度は全て90°以下なので、
断面で鈍角三角形は作れない。

370 名前:132人目の素数さん投稿日:02/10/26 07:27
>>368>>354>>364のまちがい。すまん。
まあ354も悩んでたみたいだし、いいか‥

371 名前:not354投稿日:02/10/26 07:29
>>368
せいぜい直角三角形かな

372 名前:369投稿日:02/10/26 07:37
うそです。

373 名前:369投稿日:02/10/26 07:38
一度うそをついてみたかったんですよ
ははは

374 名前:369投稿日:02/10/26 07:38
さーて次はどこでつきましょうか
腕が鳴る

375 名前:132人目の素数さん投稿日:02/10/26 07:38
区間 [0,1] からランダムに実数を1つ選び記録する、
という作業をくりかえす。

記録した数の合計が1を超えるまでの作業回数の
期待値はいくらか。


376 名前:132人目の素数さん投稿日:02/10/26 09:14
正方形ABCDのBからDにABを半径とする1/4円を描く
BからCへBCを直径とする半円を描く
1/4円と半円の交わった部分の面積を求めよ。
正方形の一辺の長さをaとする。
中学生並の数学知識で解いてください。

377 名前:132人目の素数さん投稿日:02/10/26 10:10
>>368
一つの角は60度くらい(∠B)、∠BAPは極小、∠BPAは約120度まで行くかな?
(∠Bと言っても実際には、少しずれた所にあるから二つに切れるのですが)

正三角形で無ければ?

378 名前:132人目の素数さん投稿日:02/10/26 10:16
>>376
円弧の交点をPとした時
∠BAPは中学生の数学知識では出せないのだが‥
それでも面積出るの?


379 名前:132人目の素数さん投稿日:02/10/26 10:39
>>375


380 名前:132人目の素数さん投稿日:02/10/26 11:00
どうして4以上の偶数は素数の2つの和で表せるのですか?

381 名前:132人目の素数さん投稿日:02/10/26 11:09
>>375
Σ[n>0](1/n! - 1/n+1!)*(n+1)
=Σ[n>0]1/n-1!
=Σ[n≧0]1/n!
=e

382 名前:132人目の素数さん投稿日:02/10/26 12:39
>>368
で、>>339が有限の高さの三角柱では無理という点は同意ですか?
そこが問題だと思うのですが

383 名前:132人目の素数さん投稿日:02/10/26 14:17
>>376
a^2/4(3arctan(1/2) + π/2 - 2)って答えになるのですけど…

384 名前:132人目の素数さん投稿日:02/10/26 21:07
>>381
解説キボンヌ

385 名前:現役高校生投稿日:02/10/26 21:16
曲線:y=x^2 と 曲線:y=-1/logx
の交点の座標を求めよ。

先生に聞いたら解けてませんでした

386 名前:132人目の素数さん投稿日:02/10/26 21:24
Σ[n>0](1/n! - 1/n+1!)*(n+1)
=Σ[n>0]{n/n!+1/n!-(n+1)/((n+1)!)}
=Σ[n>0]{1/((n-1)!)+1/n!-1/n!}
=Σ[n>0]1/((n-1)!)
=Σ[n≧0]1/n!
(= 1 + 1 + 1/2 + 1/6 + ...)
=e

387 名前:132人目の素数さん投稿日:02/10/26 21:38
>>386
スマン、なんで1行目の式が出てくるかわからないんだ。
そこから後の変形はおk。

388 名前:368投稿日:02/10/26 22:05
>>382
有限の高さの(というか十分に薄いとでもいうか‥)正三角柱から
角を切り落とす方法で鈍角三角形(ただし鈍角<120°)が取り出せるとこ
まではわかりました。
一般に角度a°の角からは(180−a)°未満までの鈍角が切り出せそうな
ので、一番切り出せる角度小さいのは正三角柱ではないかと思いそれで考えて
いました。
角を切り取る方法では、それ以上の角度の鈍角を切り出す方法が思いつきませ
ん。他の方法はまだ試していませんが、今のところ思いつきません。
また、120°までの角が切り出せるからと言っても、他の2角を任意にでき
るかどうかまでは考えていません。

「有限の高さの三角柱では無理」に同意できるほどには、まだ考えていないと
いうことです。直感的には無理そうであるとは思うのですが‥

389 名前:132人目の素数さん投稿日:02/10/26 22:22
>>385
0^2=0 , lim[x→0]{-1/log(x)}=0
なので (0,0)
ダメか?

390 名前:現役高校生投稿日:02/10/26 22:25
y=-1/logx は真数条件で定義域が0<x だから y=-1/logxには(0,0)が存在しないのでは?

391 名前:132人目の素数さん投稿日:02/10/26 22:31
x=1/eのとき
logx=-1

392 名前:132人目の素数さん投稿日:02/10/26 22:36
もう一つ交点があるようだが、どっちにしろ
通常の記法で表すのは無理っぽ。

393 名前:現役高校生投稿日:02/10/26 22:40
やはり無理なんでしょうか....
授業中にふと数値を入れ替えてみた問題なんですけど、
通常の記方以外なら....グラフのみですか?

394 名前:132人目の素数さん投稿日:02/10/27 02:42
1〜nまでの自然数によって構成される集合Sがある。
この集合からk個の要素を持つ異なる部分集合をm個取り出し、
それらをS(1),S(2),…,S(m)とする。
異なる、i,jについてS(i)∩S(j)の要素が

1) 0になる場合のmの最大値をn,kを用いて表せ。
2) 1になる場合のmの最大値をn,kを用いて表せ。
3) 2になる場合のmの最大値をn,kを用いて表せ。
3) 6になる場合のmの最大値をn,kを用いて表せ。

395 名前:132人目の素数さん投稿日:02/10/27 03:24
>>394
誤:異なる、i,jについてS(i)∩S(j)の要素が
正:異なる、i,jについてS(i)∩S(j)の要素数が


396 名前:132人目の素数さん投稿日:02/10/29 08:08
あたらしいもんだいきぼん

397 名前:132人目の素数さん投稿日:02/10/30 01:50
・f(n,m)≡f(n+1,m)+f(n-1,m)+f(n,m+1)+f(n,m-1) (mod 2)
・全てのn,mに対しf(n,m)は0か1
(n,mは整数)
を満たす関数f:N×N→{0,1}が与えられた時、

・任意のN×Nの有限部分集合Aに対し
「あるk,l∈N(kl≠0)があって、全ての(a,b)∈Aでf(a+k,b+l)=f(a,b)となる。
 しかもこのような(k,l)の組が無数に存在する。」

というのを満たさない事があるか?

398 名前:132人目の素数さん投稿日:02/10/30 01:51
全てのn,m∈Zに対し
・f(n,m)≡f(n+1,m)+f(n-1,m)+f(n,m+1)+f(n,m-1) (mod 2)
・f(n,m)は0か1
を満たす関数f:Z×Z→{0,1}が与えられた時、

・任意のZ×Zの有限部分集合Aに対し
「あるk,l∈Z(kl≠0)があって、全ての(a,b)∈Aでf(a+k,b+l)=f(a,b)となる。
 しかもこのような(k,l)の組が無数に存在する。」

というのを満たさない事があるか?

に訂正。


399 名前:132人目の素数さん投稿日:02/10/30 19:27


400 名前:132人目の素数さん投稿日:02/10/31 04:01
同一平面上にどの2つも共有点をもつ4つの円がある。ただし、ここでは内部も含めて円ということにする。
次の(1)(2)は正しいといえるか。正しければ証明を、正しくなければ反例をあげよ。

(1)どの2つも共有点があるならば、ある3つの円の共有点が存在する。
(2)どの3つも共有点があるならば、4つの円の共有点が存在する。

401 名前:132人目の素数さん投稿日:02/10/31 22:58
>>400
(1)偽
反例=10円玉3つを品の形に並べて、内部に小さい円。


402 名前:401投稿日:02/10/31 23:03
>>400
(2)偽
反例=品の形をつくって、そのうちの1個と全く同じ位置に同じ半径の円。

403 名前:132人目の素数さん投稿日:02/11/01 03:05
確か未だに非可算個の超越数は得られてないと思ったが、
card2^R個の不連続関数もまだ得られていないのだろうか?

404 名前:132人目の素数さん投稿日:02/11/01 07:41
>>403
非可算個の超越数って構成できるの?

405 名前:132人目の素数さん投稿日:02/11/01 07:43
>>404
どういう意味?

406 名前:132人目の素数さん投稿日:02/11/01 08:14
>>404
あるR上の関数 f(x) で、代数的数を値としてとりえるのが
高々カサン無限個のxのときってのが作れればいいんじゃないの?

407 名前:132人目の素数さん投稿日:02/11/01 10:18
XY平面上で
直線y=xの点Pから、放物線y=x^2の任意の接線に対して対称となる
点をQとする。点Qの軌跡を求めてちょ。

408 名前:403投稿日:02/11/01 10:55
>>406
f(x)=xとすればいい?
そうでなくて、「これが超越数だ!」という数を具体的に
非加算個作る。
例えば、Cantor集合K={x|x∈[0,1]、3進展開したときに0か2のみが現れる数}
としたときに、π+Kは、全て超越数?という予想があるらしいが
証明はまだされていないはず。

409 名前:404投稿日:02/11/01 10:58
そうやるのかぁ。

410 名前:132人目の素数さん投稿日:02/11/02 00:19
>>407
ぐにぐに曲がった面白い曲線になりますな。
曲線の式は特に面白くないので省略。
ハイ、次っ!

411 名前:400投稿日:02/11/02 11:14
>>402
ゴメンナサイ。条件不足だった。
4つの円はどの2つも異なるものとする
としてください。だからやり直して。
>>401は正解。

412 名前:132人目の素数さん投稿日:02/11/02 17:10
>>400
(2)偽
反例=品の形をつくって、そのうちの1個と全く同じ位置にちょっとだけ大きい半径の円。

413 名前:402投稿日:02/11/03 22:53
>>412
そうそう。ほんとはそっちが先に思いついたんだけどね。
説明ムツカシイから意地悪な方かいちゃったw

414 名前:132人目の素数さん投稿日:02/11/03 23:24
>>402>>412-413
重なってる円2つをA,B、その他の円をC,Dとすると
CDの重なってる部分にはA,Bは重なるの?

重ならないのならA,C,D or B,C,Dの共有点は無くて条件を満たさなくて、
重なるのならA,B,C,Dの共有点があるような気がするのだけど…

415 名前:132人目の素数さん投稿日:02/11/04 01:00
3つ選ぶって事は、
4C3=6通りあるから・・・

416 名前:132人目の素数さん投稿日:02/11/04 01:13
A[0]=0
A[n]=c*A[n-1]+1/n (n>1 0<c<1)
となる数列A[n]に対し、
A[n]→0(n→0)となる事を証明せよ。

417 名前:132人目の素数さん投稿日:02/11/04 01:58
A[n]=Σ_[k=1,n] c^(n-k)/k
あとはアーベル変形でできそう。

418 名前:KARL ◆.PgjHKPQSQ 投稿日:02/11/04 02:30
Lucas 数 L(n) は次のように定義される。
L(0)=2, L(1)=1
L(n)=L(n-1)+L(n-2) (n=2,3,4,...)

次の命題を証明せよ。

 pが3より大きな素数で、p^kがその任意のベキであるとする
(kは正の整数)。すると2p^k番目のLucas数 L(2p^k)はつねに3で
終わる(1位の数が常に3)

419 名前:132人目の素数さん投稿日:02/11/04 02:45
L(n+12)=L(n) (mod10)
p=±1 (mod6)

420 名前:132人目の素数さん投稿日:02/11/04 11:43
ムズすぎ
ttp://dms100.org/worksucks/lame.html

421 名前:132人目の素数さん投稿日:02/11/04 12:10
>>420
>>420
ブラクラ
 ブ ラ ク ラ

422 名前:132人目の素数さん投稿日:02/11/04 12:19
>>421
ブラウザはクラッシュしない。それ以外のがクラッシュするだけ

423 名前:132人目の素数さん投稿日:02/11/04 12:40
ふ〜〜ん

424 名前:402投稿日:02/11/04 20:25
>>414
あ、そうか。じゃあ俺が間違ってました。訂正。
(2)=偽:反例
3つの円を接して品の形をつくる。
それを全部含むような大きな円を書く。

425 名前:132人目の素数さん投稿日:02/11/04 22:38
>>400の(2)の反例になるような4つの円があったとする。
その中から円を3つ選びそれをA,B,Cとする。
条件より円A,B,Cに含まれる点、円A,Bに含まれるが円Cに含まれない点、
円A,Cに含まれるが円Bに含まれない点、円B,Cに含まれるが円Aに含まれない点、
の4つの点が存在しなければならない。
よって中学生でよくやる3つの円で出来るベン図みたいにA,B,Cは交わってる。

次に真ん中にある丸まった三角形、これの頂点を結んだ3本の直線によって平面を7つの領域に分ける。
中にある領域をO、そしてOと隣接した領域をP,Q,Rとする。

すると残りの円Dの任意の点がOに含まれてなくて、
P,Q,Rには少なくとも1つ、円Dの点が入ってなきゃいけなくなる。

P,Q,Rからどのように一つずつ点を選んでも、その3つで出来る三角形は領域Oを含んでしまい、
円が凸な図形であるという事から円DはO内の点を含んでしまい矛盾する。

よって>>400(2)は真である。

…上手い証明が出来んかった。

426 名前:400投稿日:02/11/05 01:41
(2)は真です
>>425は別に下手な証明で無いと思うよ

427 名前:400投稿日:02/11/05 03:06
私の考えていた解答はこうです。


4つの円をA,B,C,Dとする。A内の点をPa、A,Bの共通部分の点をPabなどとかく。
Pabcdなる点が存在すれば真である。

条件を満たす図形にはPabc,Pabd,Pacd,Pbcdの4点がある。これを次の(@)〜(C)に場合分けして考える。
(@)この4点のうち2点が重なる。
(A)この4点のうち3点が一直線上にある。
(B)この4点が凸四角形を作る。
(C)この4点が凹四角形を作る。

(@)のときはその点は当然Pabcdであり、題意を満たす。

(A)のときは例えばPabc,Pabd,Pacdがこの順に並んだとすると、
円が凸な図形であることからPabdはPabcとPacdのどちらも含む円A,Cに含まれる。
したがってPabd=Pabcd

(B)のときはPabc,Pabd,Pacd,Pbcdがこの順に反時計回りにあるとして、
対角線PabcPacd上の任意の点はPacであり、対角線PabdPbcd上の任意の点はPbdである。
凸四角形の対角線は交点を持ち、それはPabcdである。

(C)のときはPbcdがへこんでいる頂点とし、その相対する頂点をPabcとすれば、
線分PabdPacd上の任意の点はPadで、直線PabcPbcdと線分PabdPacdは交点をもつので
Pbcdは線分PadPabc上の点、すなわちPaであるからPabcdである。

428 名前:425投稿日:02/11/05 07:52
そういう手があるか。感心

429 名前:132人目の素数さん投稿日:02/11/07 20:45
一辺の長さが1の正四面体が2枚の平行な平面の間にすっぽり入るとき、
その2枚の平面の間の距離の取り得る値で最小なのを求めよ。

430 名前:132人目の素数さん投稿日:02/11/07 20:52
(√2)/2 かな?

431 名前:429投稿日:02/11/07 21:36
>>430
証明キボンヌ。

432 名前:132人目の素数さん投稿日:02/11/07 21:41
だな‥

433 名前:132人目の素数さん投稿日:02/11/07 22:40
一つの面(正三角形)に注目してその命題を考えてみると
垂線の幅である√2/2が最小となり、
それを底面として正四面体を考えたらその幅よりも狭くなる事はありえなく
(狭くなるのなら正三角形の時点で狭くできるはずである)
正三角形での幅で十分おさまる為。

数学的じゃなくて理屈的な証明(説明?)だな

434 名前:433投稿日:02/11/07 23:09
(補足)
正四面体の垂線を考えると2/3となり、
2/3>√2/2なので√2/2が最小である。

435 名前:132人目の素数さん投稿日:02/11/08 12:48
くだらない(以下略)で出した問題。多分これであってる。

問題
1/(a_1)+1/(a_2)+…+1/(a_n)=1 ,(a_i)>1 (i=1,2,…,n),n>=2のとき
(a_1)+(a_2)+…+(a_n)の最小値を求めよ。

今度はきちんと答を書きます。(一週間以内には)

436 名前:工房ですか!?投稿日:02/11/08 18:06
>>435

n=2のとき、1/a_1+1/a_2=r (0<r<=1)を満たすa_1は、a_1=a_2=2/rを
満たし、最小は4/r=(2^2)/rである。・・・(計算省略)

n=kのとき、1/(a_1)+1/(a_2)+・・・+1/(a_k)=r (0<r<=1)を
満たすa_1〜a_kはa_1=a_2=・・・=a_k=k/rを満たすとき、
a_1+a_2+・・・+a_kの最小値k^2/rをとると仮定する。

n=k+1のとき、1/(a_1)+1/(a_2)+・・・+1/(a_k)=p (0<p<r<=1)−−*として、
p+(1/a_[k+1])=r (0<p<r<=1)とするとき、
仮定より、*を満たすa_1〜a_kはすべてk/pのとき、−−@
a_1+a_2+a_3+a_4+・・・+a_kの最小値k^2/pを取るので、

1/(a_[k+1])=r-pのときの最小値:−−A
k^2/p+(a_[k+1])=k^2/p+1/(r-p)となる。−−B

ここで、Bをf(p)とし、pを動かして最小を求めると、微分して、
p=rk/(1+k)のとき、最小となる。

これを@、Aに代入すると、a_1=a_2=・・・=a_k=a_[k+1]=(k+1)/rとなり、
また、Bに代入すると最小は(k+1)^2/r^2となるので、n=k+1のときも
成り立つ。

よって、n>=2について、1/(a_1)+1/(a_2)+1/(a_3)+・・・+1/(a_n)=rを
満たすa_i(i=1,2,3,・・・,n)について、a_1+a_2+a_3+・・・+a_n=n^2/rで
ある。
ここで、r=1を代入すると、題意と一致するので、
最小値はn^2となる。


437 名前:132人目の素数さん投稿日:02/11/08 18:08
知恵遅れ

438 名前:132人目の素数さん投稿日:02/11/09 00:19
ai≧1なら
a1+a2+…+an
=(1/a1+1/a2+…+1/an)(a1+a2+…+an)
=n+Σ(ai/aj+aj/ai)
≧n+Σ(√(ai/aj)(aj/ai))
=n^2
等号成立はai=1、と出来るのにねぇ。

最小値の無い事証明せんとあかんからもちっと長くなる。

439 名前:438投稿日:02/11/09 00:26
そういやこの手の問題やってると他スレで出た次の問題を思い出すのよ。

n≧3として、
x_1,x_2,・・・,x_n は正の実数として
x_1+x_2+・・・x_n=1 を満たしている。
このとき

x_1/√(x_1+x_2) + x_2/√(x_2+x_3) +・・・
+x_(n-1)/√(x_(n-1)+x_n) + x_n/√(x_n+x_1)
の最大値を求めよ。

どうしてもラグランジュの未定乗数法使わないと出来なかった。

440 名前:438投稿日:02/11/09 00:28
438訂正。

ai≧1なら
a1+a2+…+an
=(1/a1+1/a2+…+1/an)(a1+a2+…+an)
=n+Σ(ai/aj+aj/ai)
≧n+Σ(2√(ai/aj)(aj/ai))
=n^2
等号成立はai=1

441 名前:132人目の素数さん投稿日:02/11/09 03:59
激しく概出ですが

0,1mmの紙を50回二つ折りにし続けるとその厚さはどのくらい?

442 名前:132人目の素数さん投稿日:02/11/09 04:31
>>441
20%くらいじゃないの?


443 名前:132人目の素数さん投稿日:02/11/09 05:19
>>441
何回折っても紙の厚みは変わらないよ。
まあ全体の厚みなら(折れたとして)太陽にちょっと届かないぐらい。


444 名前:132人目の素数さん投稿日:02/11/09 07:32
>>442
茶化すぐらいなら寝ておきなさい。

445 名前:132人目の素数さん投稿日:02/11/09 08:07
>>442
ちゃんと計算しる!89.1%だろが!!

446 名前:132人目の素数さん投稿日:02/11/09 08:12
tasurenowadaiwomotikomunayo

447 名前:132人目の素数さん投稿日:02/11/09 09:14
>>446=
Don't talk about other thread at here.

448 名前:132人目の素数さん投稿日:02/11/09 09:27
1億キロくらい?

449 名前:132人目の素数さん投稿日:02/11/09 11:57
% bc <<< '2^50'
1125899906842624


450 名前:132人目の素数さん投稿日:02/11/09 15:22
グリコキャラメルの1粒300mは有名だが、
フルマラソンをするには141粒(15箱)必要だということが
ふと頭をよぎった。 

451 名前:132人目の素数さん投稿日:02/11/09 20:40
この問題を解けるヤシいないか?
問題
7、7、11、12
この数字でうかびあがる俳優さんのニックネームは?

ヒントはメール欄

452 名前:435投稿日:02/11/12 12:38
>>436
正解!!
ちなみに俺の回答(たしかこんな感じ。あってると思う)

1=1/(a_1)+1/(a_2)+…+1/(a_n)
両辺に(a_1)+(a_2)+…+(a_n)をかける
(a_1)+(a_2)+…+(a_n)={1/(a_1)+1/(a_2)+…+1/(a_n)}{(a_1)+(a_2)+…+(a_n)}
={{√1/√(a_1)}^2+{√1/√(a_2)}^2+…+{√1/√(a_n)}^2}{{√(a_1)}^2+{√(a_2)}^2+…+{√(a_n)}^2}
>={√(a_1)/√(a_1)+√(a_2)/√(a_2)+…+√(a_n)/√(a_n)}^2
={1+1+…+1}^2=n^2
等号成立は(a_i)=(a_j)(j=1,2,…n)で、1=1/(a_1)+1/(a_2)+…+1/(a_n)に代入すると、(a_i)=nを得る。
よって最小値はn^2でその時(a_i)=n。

ここまででおよそ20秒。不等式の知識さえあれば厨房でも解ける、本当につまらん問題だった。

453 名前:132人目の素数さん投稿日:02/11/12 13:28
ここにも頭悪い奴が・・・

454 名前:132人目の素数さん投稿日:02/11/12 13:57
>>452
不備含

455 名前:132人目の素数さん投稿日:02/11/12 17:04
αを1以上の有理数とする。
Σ[k=1〜n]1/k^α
が整数となるような2以上の自然数nが存在するようなαは存在するか。
存在するならα及びnを求め、存在しないなら証明せよ。

456 名前:455投稿日:02/11/12 19:21
ついでにもう1問

実数a_i(i=0,1,2,……n)によって定められるn次方程式
Σ[i=0〜n]a_ix^i=0
の重解も含めたn個の解が正の実数のときn^2a_na_0とa_(n-1)a_1の大小を調べよ。

457 名前:452投稿日:02/11/13 12:20
あっ、間違えた。
√1/√(a_i)→√(1/(a_i))
√(a_i)/√(a_i)→√((a_i)/(a_i))
コピペミス。

458 名前:132人目の素数さん投稿日:02/11/13 15:58
>>436 の解答は、「3回繰り返すと笑いがとれる」という喜劇の格言に忠実で
好感がもてる。>>452 はよくある解答(等号成立条件もうちとはっきり書かん
と減点くらうかも)なんだが、技巧に走って面白みにかける。特殊不等式を
使うんなら、算術平均 >= 幾何平均 >= 調和平均 という関係を使って、
(a_1 + a_2 + … + a_n)/n >= n/(1/a_1 + 1/a_2 + … 1/a_n)
で終りだというツッコミが、くだらんスレであったが、そのものズバリで楽み
がないかもしれん。


459 名前:132人目の素数さん投稿日:02/11/13 18:49
>>452の不等式が理解できないなぁ。
コーシーシュワルツじゃないよねぇ。もしそうなら誤答だし。

460 名前:132人目の素数さん投稿日:02/11/13 19:01
素直に烙印押してあげれば?誤答だって。

461 名前:132人目の素数さん投稿日:02/11/14 12:30
大昔、高校の頃に感動したけど、今みるとそうでもないものなんで
sage とくが、a, b > 0 で
f_n(a, b) = ((a^n + b^n)/2)^(1/n) (n ≠ 0),
f_0(a, b) = lim(n→0)f_n(a, b)
とすると、
n > m ならば f_n(a, b) >= f_m(a, b) (等号成立は a = b のとき)。

実は、f_0(a, b) = √(ab) で、f_1 が相加平均、f_0 が相乗平均、
f_{-1} が調和平均。


462 名前:132人目の素数さん投稿日:02/11/14 12:42
まだわからんのか!!このIDIOTS!!
(Σ[i=1,n]((b_i))^2)*Σ[i=1,n]((c_i))^2)>=(Σ[i=1,n](b_i)*(c_i))^2
((b_i))^2=(a_i)
((c_i))^2=1/(a_i)
もちろん
(a_i)>0だから不適合はなし。もし最小値がn^2より下なら、この不等式は成り立たないし(悔しかったら否定してみろ!)、それより上なら等号成立になる条件((a_1)=(a_2)=…=(a_n)))で得た結果より大きくなるから矛盾。
もしかしてとけなかった?カワイソウ。
>>458
調和平均 ってなんですか?

まあこの話題はこれで無視して(これ以降は放置って意味ね)

つぎの問題
高校1〜3年生の問題(大学生以上は無視してください。もちろん中学生以下は歓迎)
(a_i)nとし、
Σ[i=1,n](a_i)=n
とするとき
(a_1)*(a_2)*…*(a_n)
の最大値を求めよ。
ちなみにどっかの低級理系私立大学の入試問題

463 名前:132人目の素数さん投稿日:02/11/14 12:46
頭悪い奴って何処にでもいるんだね。

464 名前:132人目の素数さん投稿日:02/11/14 14:00
稀に見る逸材

465 名前:132人目の素数さん投稿日:02/11/14 14:30
462は貴重な存在です
むやみにおどしたりフラッシュをたかないように

466 名前:132人目の素数さん投稿日:02/11/14 15:12
コテハンでおながいします
トリップもおながいします

467 名前:132人目の素数さん投稿日:02/11/14 19:13
642が自信たっぷりなのが素晴らしい。
その自信をオレにも少し分けてくれよ。

468 名前:132人目の素数さん投稿日:02/11/14 23:10
暴れだした

469 名前:132人目の素数さん投稿日:02/11/15 20:21
許してやれよ、勘違いしていられるのは高校生のうちなんだから。まあ、
タイポが多くてしかもミスに気がつかないのは、問題をパズルみたいに
とらえていて、背景に何があるのか、これっぽっちも分かっていないか
らだろうけど。

しかし、今日日の若い子はホントに調和平均知らんのか。それとも
>>462 の受けてきた教育が不幸なのか。ちなみに、>>461 は当然、N個
の場合に拡張できる、つまり、
a_i > 0, n > m で、(Σa_i^n/N)^(1/n) >= (Σa_i^m/N)^(1/m)。
これは、m != 0 とき、a_i^m =x_i, n/m = p とおくと、
Σx_i^p/N >= (Σx_i/N)^p ということなんで、y = x^p が下に凸なこと
からほとんど自明。m = 0のところは連続性ってことで。


470 名前:132人目の素数さん投稿日:02/11/15 22:20
グラフの凸性を簡単に使うのは好きくない
と、言ってみるテスト

とはいえ掲示板で書き込むのにいちいち断りを入れていたら大変だけどね

ところで>>455はα=1のときとα≧2のときは無いことが証明できたんだが
1<α<2のときが微妙だ。
1/k^αが無理数でしかもただ1通りに表わせることとその和が無理数であることを使っていいならできるんだが・・・

471 名前:470投稿日:02/11/15 22:58
α≧2のとき
1<Σ[k=1〜n]1/k^α<1+∫[1〜n]dx/x^α<2
より整数となることはない

α=1のとき
Σ[k=1〜n]1/k^αを既約分数で表わしたとき
分子は奇数で分母は偶数(数学的帰納法などで証明できる)
したがって整数となることはない

以下補完誰か頼む

472 名前:132人目の素数さん投稿日:02/11/20 00:48
2つの自然数n,mがある。
n,mを素数pで割った余りをn_p,m_pとして、
全ての素数pに対しn_p≦m_pとなる時、

n=mとなる事を証明してみ。ちょとむずいよ。

473 名前:132人目の素数さん投稿日:02/11/20 01:47
>>472
p > max(n,m) をとってきたとき、n_p(=n) <= m_p(=m) だから、n <= m。
m の任意の素因数pに対し、m_p = 0。
このpに対し、n_p <= 0 だから、nはpで割り切れる。
つまり、nはmの倍数であり、n >= m。

以上より、n = m

これでいいよね?

474 名前:473投稿日:02/11/20 01:50
あ、しくじった。

475 名前:472投稿日:02/11/20 03:00
例えばn=12,m=16の場合、p>max(n,m)に対してn_p(=n)<=m_p(=m)
そしてmの任意の素因数p(2しかないけど)に対し、m_p=0でn_p=0でやっぱりn_p<=m_pってわけですな。
473様、もうちょっと頑張ってくだされ。

476 名前:132人目の素数さん投稿日:02/11/20 18:37
m,n≠1をかんがえる。
題意よりm,nの素因数は同じでなければならない。(∵m,nを割り切る素数は少なくとも1つ存在する)
あとはm<p<n(n<p<m)となる素数が存在すればいいんだがそうは簡単にい神崎だった
きっと「全ての素数pに対しn_p≦m_pとなる」という条件を使うんだろうな。
あまりの周期性とか?

477 名前:132人目の素数さん投稿日:02/11/20 23:56
ある町には2つの病院があって、
大きい方の病院では毎日約45人、
小さい方の病院では約15人の赤ちゃんが生まれる。
もちろん生まれる赤ちゃんの約50%は男の子である。
しかし、男の子の生まれる正確な割合は日によって変わる。
50%よりも高い日もあれば低い日もある。
それぞれの病院で、一年間の間生まれた赤ちゃんのうちの60%以上が
男の子だった日を記録した。
そのような日をたくさん記録したのは、どちらの病院だと思いますか?

478 名前:132人目の素数さん投稿日:02/11/21 00:11
スタートからnマスでゴールするすごろくがある。さいころを振って出た目の数だけ進めていくが、
ゴールする際は、必ず出た目ちょうどでゴールに着かなくてはならず、出た目の数が大きすぎた場合は
多い分だけ戻ることとする。

例:n=4の場合
スタートでさいころを振って6が出た場合、2だけ多いので、ゴールから2マス戻って2のマスの位置に来る。

x回目でちょうどゴールに着くとした場合、E(x)をnを用いて表せ。n≧3とする。



479 名前:132人目の素数さん投稿日:02/11/21 00:15
A(1)=1 、
S(n)→1(n→∞)、
n(n-2)×A(n+1)=S(n) (n≧1)

をみたす数列A(n)の一般項を求めよ。ここでS(n)はA(1)からA(n)までの和である。


480 名前:132人目の素数さん投稿日:02/11/21 00:57
>>477
小さい方じゃないの?

481 名前:132人目の素数さん投稿日:02/11/21 04:14
さらに小さい病院では毎日ひとりの(ry

482 名前:132人目の素数さん投稿日:02/11/21 18:05
>>472
だめだ。これむずい。もうそろそろ答えおしえてたも。

483 名前:真赤ロ投稿日:02/11/21 20:02
<<マッカロのナンバーマシン>>
ここに、ある計算機械がある。
ある数を入れると、計算規則に基づきある数を出力する。

☆規則0
・マシンが扱える数とは、正の整数のみ。負の数や分数などは扱えない。
・数Nは普通のやり方でアラビア数字1〜9の連なりとして書かれる。数の
 中に0が含まれている場合は扱えない。

☆規則1
・任意の数Xに対して、数2X(2にXを続けたもの。Xの2倍ではない!)は
 受け入れ可能であって、2XはXを生じる。

たとえば、253は53を生じる。27482は7482を生じる。23985は3985を生じ
る。つまり、2Xをマシンに入れると最初の2が取り除かれて、残されたXが
出てくる。


484 名前:真赤ロ投稿日:02/11/21 20:03
<<マッカロのナンバーマシン>>
☆規則2
「同伴」とは:任意の数Xに対して、数X2Xは特に重要な役割を担う。よっ
て、数X2Xのことを数Xの同伴と呼ぶ。たとえば、7の同伴は727、594の同
伴は5942594となる。

・任意の数XとYに対して、XがYを生じるならば、3XはYの同伴を生じる。

たとえば、規則1から27は7を生じるから、327は7の同伴すなわち727を生
じる。このように327は727を生じる。また、2586は586を生じる。よって
32586は586の同伴を生じる。それは5862586である。


485 名前:真赤ロ投稿日:02/11/21 20:04
<<マッカロのナンバーマシン>>
☆受け入れられない数
いままでの数は、2か3のいずれかで始まっている。たとえば4から始まる
数だとどうなるか?
→4から始まる数は受け入れられない。このマシンで受け入れられる数は
2か3で始まるものばかりで、しかも2か3で始まる数であっても受け入れら
れない数がある。

2か3で始まるどんな数が受け入れられないのか?
→たとえばXが単なる2の場合は受け入れられない。というのは、単なる2
は規則1か規則2のいずれにも当てはまらないからだ。しかし、2から始
まる任意の数2Xは受け入れられる。
また、すべて3だけからなる数も受け入れられないし、32や332なども受け
入れられない。しかし、任意の数Xに対して2Xは受け入れられるし、332Xや
3332Xも受け入れられる。まとめれば、受け入れ可能数とは2X,32X,332X,
3332Xなど、3の重複した任意の長さの数字の後に2Xの続く数字である。


486 名前:真赤ロ投稿日:02/11/21 20:06
<<マッカロのナンバーマシン>>
ここで、2XはXを生じるし、32XはXの同伴を生じる。さらに332XはXの同伴
の同伴、3332XはXの同伴の同伴の同伴を生じる。同伴の同伴のことを二重
同伴、同伴の同伴の同伴のことを三重同伴などと呼ぶことする。

ここで問題。

@自分自身を生じる数Nがある。つまりNをマシンに入れると、同じ数Nが出てくる。このようなNとは?

A自分自身の同伴を生じる数Nがある。つまりNをマシンに入れると、数N2Nが出てくる。このようなNとは?

B任意の数Aに対し、AYを生じる数Yが存在する。このようなYとは?

C数Nは数N2を生じるとする。果たしてこのようなNは存在するのか?

487 名前:132人目の素数さん投稿日:02/11/21 20:11
>>483-486
うわー奇遇だねぇ。スマリヤンの本から引っ張ってきたね?
おれもその本今日ぱらぱらとページめくって読んだよ。

488 名前:真赤ロ投稿日:02/11/21 20:17
肝心なトコロで誤訳多し。

489 名前:132人目の素数さん投稿日:02/11/21 20:54
>>489もっと簡単に問題を理解出来るのがいいです
はっきりいって長い

490 名前:132人目の素数さん投稿日:02/11/21 20:55
面白い問題教えてくれっつースレだよな。ここは

491 名前:真赤ロ投稿日:02/11/21 21:06
>>489
そうかも。スマソ

492 名前:132人目の素数さん投稿日:02/11/21 21:18
n, k を任意の自然数とします。
k個以上の素因数を持つ、連続するn個の自然数が
存在することを示して下さい。

493 名前:132人目の素数さん投稿日:02/11/21 22:18
>>472 の答えおしえてくれ。
>>492 これはできる。
p(i,j) (i:1〜n,j:1〜k)を相異なる素数とする。合同方程式
x≡-i (mod p(i,j))
の解をpとする。p+iはp(i,j) (j:1〜k)でわりきれる。どうよ?

494 名前:132人目の素数さん投稿日:02/11/21 22:38
正の整数nに対し、B(n)はnを二進表現したときの1の個数とする。
Σ[i=1, ∞]B(i)/(i(i+1))
を求めよ。

495 名前:132人目の素数さん投稿日:02/11/21 22:45
>>493
x≡-i (mod p(i,j))
って、iが変われば解も変わるんでないの?
変なこと言ってたらスマソ

496 名前:132人目の素数さん投稿日:02/11/21 22:49
>>495
てか
x≡-i (mod p(i,j))
を連立合同方程式とみなして、これらnk個の方程式全部みたすような整数pを
とってこいって意味。そのようなpの存在は中国の剰余の定理ってので保証されてる。

497 名前:132人目の素数さん投稿日:02/11/21 23:05
領域D_nを次のように定義する。
  D_n={(x,y)|0<x<π,nsinx<y<(n+1)sinx}
D_0,D_1,D_2,……のなかで格子点をふくまないものが無限に存在する可能性について述べよ。(証明してもよい)
ただし、必要ならば関数電卓などのtoolを用いてよい。
また、このような性質をもつものをいくつか求めよ。

498 名前:132人目の素数さん投稿日:02/11/21 23:19
【 田代祭り】モーニング娘。LOVEオーディション2002【再び】
1 名前:名無し募集中。。。 02/11/19 23:51 ID:t2y9frsV
ブサイクな女に投票してモーニング娘。の息の根を止めましょう。

関連リンク
http://www.tv-tokyo.co.jp/ テレ東HP
http://audition.tv-tokyo.co.jp/list.cgi 候補者一覧

みなさん、ぜひ284に一票入れてやって下さい。
http://audition.tv-tokyo.co.jp/img/candidates/large/00284.jpg

499 名前:132人目の素数さん投稿日:02/11/21 23:54
>>496
レスさんきゅ。ある程度理解デケタ。

500 名前:132人目の素数さん投稿日:02/11/21 23:58
NTTのマークを方程式で表せ。
カンニングしたいヤツは、ブルーバックスのバックナンバーを探してみそ。

501 名前:132人目の素数さん投稿日:02/11/22 00:09
NTTのマークが思い出せない。

502 名前:132人目の素数さん投稿日:02/11/22 01:36
>>501
www.ntt.co.jp


503 名前:132人目の素数さん投稿日:02/11/22 05:12
x = (t^3-t)/(1+t^4)
y = -(t^3 - 2 - √3)/|t^3|
でどんなグラフになっている?

504 名前:132人目の素数さん投稿日:02/11/22 05:13
x = (t^3-t)/(1+t^4)
y = -{t^3 - (2 + √3)t}/|t^3|
で。

505 名前:132人目の素数さん投稿日:02/11/22 05:15
x = (t^3-t)/(1+t^4)
y = -|t|*t{t^3 - (2 + √3)t}/(1+t^4)
で。


506 名前:KARL ◆.PgjHKPQSQ 投稿日:02/11/24 01:55
次の a) b) c) 3つの場合について
多項式 f(x), g(x) から足し算、引き算、掛け算のみによって
多項式 h(x) = x を得ることができるかどうかを判定せよ。
a) f(x) = x^2 + x, g(x) = x^2 + 2
b) f(x) = 2x^2 + x, g(x) = 2x
c) f(x) = x^2 + x, g(x) = x^2 -2


507 名前:132人目の素数さん投稿日:02/11/24 01:57
また変なのがきた

508 名前:132人目の素数さん投稿日:02/11/24 09:57
h(x)=x=(x^2+x)-(x^2+2)+2
=f(x)-g(x)+2
こんな事がしたいのか?

509 名前:132人目の素数さん投稿日:02/11/24 11:31
>>508
2もfとgから作らないと

510 名前:132人目の素数さん投稿日:02/11/24 15:14
>>509
・・・・・・

511 名前:ミヤハwモIツ?ヒLマqツ?ツ?ツ?ツ?ツ?ツ?投稿日:02/11/24 19:53
f(x)/f(x)+f(x)/f(x) = 2 って使っていいの?

512 名前:132人目の素数さん投稿日:02/11/24 19:58
割り算だめじゃ?

513 名前:KARL ◆.PgjHKPQSQ 投稿日:02/11/27 00:30
>>506
c)はできるが、a),b)はできない。
証明してみてください。

514 名前:132人目の素数さん 投稿日:02/11/27 00:45
微分とか使ったらa),b)とかできねえ?

515 名前:132人目の素数さん投稿日:02/11/27 02:37
>>514
>多項式 f(x), g(x) から 足 し 算 、 引 き 算 、 掛 け 算 の み に よ っ て

516 名前:132人目の素数さん投稿日:02/11/27 15:38
一つだけ言わせてくれ!

面 白 い 問 題 お し え て ー な !

517 名前:132人目の素数さん投稿日:02/11/27 19:10
コイン(1円玉、5円玉、10円玉、50円玉、100円玉、500円玉の6種類)が
何枚か入った財布を持ってあるものを買いに行きました。財布の中から
1枚コインを出すと、そのものが1つ買えておつりが出ます。ただし2つ以
上は買えません。また、最初の1枚に財布の中からコインを7枚追加し、
計8枚で同じものを買おうとすると、ちょうど9つ買えておつりは出ませ
んでした。しかもこのとき、8枚の中で同じ種類のコインは3枚までしか
ありませんでした。さて、買おうとした「あるもの」の値段はいくらで
しょう?消費税はないものとします。

518 名前:132人目の素数さん投稿日:02/11/27 19:30
>>517
財布の中から50円玉を出して1つ買うことはできた。
9つ買おうとしてコインを8枚全部出したのにお金が足りなかった。
けど、9つも買ってくれるならってことでまけてくれた。
それでおつりちょうだいって言ったら怒られた。

519 名前:132人目の素数さん投稿日:02/11/27 22:11
>>472
これやっとできたかも。チェビシェフの定理っての使うんだね。おもろかった。
>>497
これムヅカッタ。これ無限に存在するで正解?
なぜさげ進行な空気かよめないage

520 名前:傍観者投稿日:02/11/27 22:15
未完のみの数え方なら知ってます
それはまず皮をむいて根気よく数えます
するとびっくり あっ 忘れちゃツタ
加護ちゃん辻ちゃん
おねむしてる 

521 名前:132人目の素数さん投稿日:02/11/27 22:17
31.12.22.103.32゙.63゙.103.21゙.71.101.92.55.45.75.41゙.42.55.14.12.22.83.13.44゙75.14.52.103.24゙.103.52.51.92.43.43.11.93.55.61.22.55.34.12.44゙.61.51.12.41゙.95.13.41.71.73.32.12.95.

これの暗号の解読をしていただきたいんですけどわかる人いますか?

522 名前:132人目の素数さん 投稿日:02/11/27 22:21
>>517
>同じ種類のコインは3枚までしかありませんでした。
これ二通りの意味にとれる。


523 名前:132人目の素数さん投稿日:02/11/27 22:24
>>521
50音

81や
83ゆ
85よ

101わorん
103ん
105わorん

524 名前:132人目の素数さん投稿日:02/11/27 22:24
さいきんじぶんが(以下略

525 名前:132人目の素数さん投稿日:02/11/27 22:46
分からない問題をここで質問するの止めれ。
まぁ面白いならいいんだけどね…

526 名前:132人目の素数さん投稿日:02/11/27 23:00
さいきんじぶんがまわりのともだちのえいきようで
75.14.52.103.24゙.103.52.51.92.43.43.11.93.55.61.22.55.34.12.44゙.61.51.12.41゙.95.13.41.71.73.32.12.95.

あかさたなはまやらわ
ポケベルか?

527 名前:132人目の素数さん投稿日:02/11/27 23:15
わからない問題↓
http://science.2ch.net/test/read.cgi/math/1038402320/l50

528 名前:473ではありませんが…投稿日:02/11/28 12:35
>>472
>>473の考えを用いて証明してみます。
自信はありませんが…

m<=p,n<=pとなる素数pでm,nを割るとm_p=m,n_p=nとなり、
m_p>n_pという条件からm>=n。
つぎにmが素数である場合と素数でない場合とで場合わけをする。
mが素数の場合、素数mでm,nを割るとm_p=0,0<=n_p<=m_p=0よりn_p=0
したがってnは1以上の整数aを用いてn=a*mとおける。m>=nより、m>=a*mつまり1>=aとなるためa=1
よってm=n。
mが素数でない場合、mは素数p_1,p_2,…,p_nを用いてm=p_1*p_2*…*p_nとおける。
m,nを素数p_1で割るとm_p=0となりn_p=0となる。
よって1以上の整数a_1をもちいてn=a_1*p_1とできる。
これと同様のことをp_2,p_3,…,p_nについて行なうと、
1以上の整数bを用いてつぎのようにおける。
n=b*p_1*p_2*…*p_n=b*m
m>=n=b*mよりb<=1となってb=1。つまりm=n。
いずれの場合にもm=nとなる。



529 名前:132人目の素数さん投稿日:02/11/28 16:51
>>528
> mは素数p_1,p_2,…,p_nを用いてm=p_1*p_2*…*p_nとおける。
p_i に重複を許すと、後の証明で躓くから、ダウト。
指数も考えなくちゃ。

m = p_1^c_1 * p_2^c_2 * ... * p_n^c_n, (a_i ∈ N)


530 名前:132人目の素数さん投稿日:02/11/28 21:47
>>519 でできたっていってた証明まちがってた。むずかしいよこれ。
できたひと解答うぷきぼん。

531 名前:KARL ◆.PgjHKPQSQ 投稿日:02/11/28 23:05
>>506
>次の a) b) c) 3つの場合について
>多項式 f(x), g(x) から足し算、引き算、掛け算のみによって
>多項式 h(x) = x を得ることができるかどうかを判定せよ。
>a) f(x) = x^2 + x, g(x) = x^2 + 2
>b) f(x) = 2x^2 + x, g(x) =2x
>c) f(x) = x^2 + x, g(x) = x^2 -2

a)の2式に対して足し算、引き算、掛け算をどんなに繰り返しても
決してxという答えは出てきません。仮にできたとしてみます。

Z(f(x),g(x))をf(x),g(x)に対していくつか加減乗の演算を繰り返
したものとする。これがxに等しくなったとすれば
 Z(f(x),g(x)) = x  ...... (1)
となる。Z(f,g)は f,g に足し算、引き算、掛け算を施したものである
ことに注意する。
さてここで、x=2とおいてみると(1)は
Z(6,6) = 2 となる。これは6だけを使った足し算、引き算、掛け算の
答えが2になることを意味する。これは矛盾。したがってa)はどんなに
がんばってもxにはなりません。b)についても同じような考え方で証明
できます。


532 名前:132人目の素数さん投稿日:02/11/28 23:19
n を正の偶数とする。n×n の正方形の盤があり,
n^2 個の単位正方形の升目に分かれている。
異なる二つの升目が共通の辺を持つときに
それらは隣接していると言う。
N 個の升目に次の条件を満たすように印を付ける:
「任意の升目(印の付いているものおよび印の付いていないもの)に対して
少なくとも一つの隣接した印の付いている升目がある」
このような事の出来る N の最小値を求めよ。



533 名前:132人目の素数さん 投稿日:02/11/29 00:57
1から番号のついた無限個の箱、すべてにボールが入っているとき、さらに無限個のボールを箱にいれたら100万円。いれろ。

534 名前:132人目の素数さん投稿日:02/11/29 01:33
Hilbert's Hotel by George Gamov in "One, Two, Three, ... Infinity",
Macmillian & Co., London, p.17 (1946)


535 名前:間違えました528です。投稿日:02/11/29 16:18
>>529
あっ、そっか。
例えば16=2*2*2*2となるけど、12=2*6で2でわりきれるからね。
うーん、m_p=0となるような素数pでnを割ったあまりが必ず0になるだからm=nと思ったのだが…。
おなじようなやり方でできるのだと思うのだけどだれかできますかね?

PS
でもよく考えたらあの問題は当たり前のことをいっているような気がする。
まぁ、当たり前のことを証明するのはゲキムヅなのはよく知ってるのだが。

536 名前:132人目の素数さん投稿日:02/11/29 22:04
>>532
n=2→N>3
n=3→N>6
n=4→N>9
n=5→N>14
...
n=2m→2m^2+1
n=2m+1→2m^2+m+1

537 名前:536投稿日:02/11/29 22:09
イコールを忘れてみました。

538 名前:132人目の素数さん投稿日:02/11/29 23:06
>>535
もともと4で両者を割って、m',n'で考えればいいいだけじゃん。
最大公約数で割っても大小関係が変わらないのだから。

539 名前:132人目の素数さん投稿日:02/11/29 23:14
>>538
大小関係変わるんじゃない?

540 名前:132人目の素数さん投稿日:02/11/29 23:17
age

541 名前:132人目の素数さん投稿日:02/11/29 23:42
>>539
ごめん、大小関係変わるね。

542 名前:132人目の素数さん投稿日:02/11/29 23:53
>>536
悪いが何を言ってるのかさっぱりわからない。

俺の予想では min N = (n^2 + 2n)/4
となるのだが、スマートに照明できない。

543 名前:132人目の素数さん投稿日:02/11/30 00:12
>>542
n=2のときN=2になって既に破綻しているよ
■□
□■
でNは3以上必要でしょ。

544 名前:132人目の素数さん投稿日:02/11/30 00:20
>「任意の升目(印の付いているものおよび印の付いていないもの)に対して
>少なくとも一つの隣接した印の付いている升目がある」
だから■をしるしつきますめとするとこれ自信に隣接するますめは
どれも□だからだめって意味じゃないの?でも
■■
□□
で条件をみたすのでn=2のときはN=2だろうな。

545 名前:132人目の素数さん投稿日:02/11/30 02:23
>>544
そんな意味だったらnの値に関わらず、常にN=2になって問題にもならない。

546 名前:132人目の素数さん投稿日:02/11/30 08:02
N=3n ?

547 名前:132人目の素数さん投稿日:02/11/30 08:02
N=3(n-2) だった。

548 名前: ◆BhMath2chk 投稿日:02/11/30 08:10
n^2/4≦N。


549 名前:132人目の素数さん投稿日:02/11/30 08:20
>>545
常にN=2にはならない。


550 名前:132人目の素数さん投稿日:02/11/30 08:53
東海道本線東京駅から新大阪駅まで各駅で行ったら最低何分?

551 名前:132人目の素数さん投稿日:02/11/30 09:33
各駅ってのは「こだま」のことか?

552 名前:EVE ◆8ZTCcC3EVE 投稿日:02/11/30 17:21
ある四面体の中に半径1,2,3の球が内接している。また、これら3つの球も
互いに外接している。このような条件を満たす四面体の体積の最大値を求めよ。

553 名前:132人目の素数さん投稿日:02/11/30 17:27
墓石に一辺の長さが1の正17角形(ABCD…NOPQ)が正確に刻まれている。

(1)辺AB,BCそれぞれの中点同士を結んだ線の長さを求めよ。

(2)(1)と同様に、中心から隣接するそれぞれの点と点の中点同士を結んでいき、
  こうして新しく出来た正17角形に上の動作を繰り返し行っていく。
  一辺の長さが1/1700を割るのは何回目の動作のときか。

554 名前:553投稿日:02/11/30 17:28
(2)(1)と同様に、隣接するそれぞれの点と点の中点同士を結んでいき、

でした。中心からを抜いてください。

555 名前:542投稿日:02/11/30 17:55
>>532                       ←←   ←←
                           ■■□□■■□□
     ←←                   □□□□□□□■↑
     ■■□□    □□■■□□.   □□■■□□□■↑
     □□□■↑ ↓■□□□□■.   ■□□□□■□□
□□  □□□■↑ ↓■□□□□■.   ■□□□□■□□
■■  ■■□□    □□■■□□.   □□■■□□□■↑
               □□□□□□   □□□□□□□■↑
               ■■□□■■   ■■□□■■□□
               →→   →→
このように前の正方形に巻きつけるような形で2列ずつタイルを
貼り付けていくことで、題意を満たす図形を次々と生成できる。

このとき印つきタイルの数は最小である。なぜなら、印つきタイルと
それらが隣接するタイルを考えると、過不足無く正方形全体が
覆えているからである。(←かなり曖昧な説明)

正方形の一辺がmらm+2に増えると、印つきタイルはm+2個増加する。
故に N = Σ[m=1 to n/2] 2n = (n^2 + 2n)/4

556 名前:132人目の素数さん投稿日:02/11/30 18:52
>>551
新幹線じゃないだろ

557 名前:132人目の素数さん投稿日:02/11/30 19:03
>>542
ごめん、あなたの「最小の理由」がよくわからん。



558 名前:132人目の素数さん投稿日:02/11/30 19:09
>>555
要は
  □□
□■■□
  □□
の敷き詰め問題に帰着する感じね

559 名前:132人目の素数さん投稿日:02/11/30 19:21
>>558
□□□
■■■
□□□
nが奇数のときはこういうのもあるよ。

560 名前:558投稿日:02/11/30 19:22
ただ単調に敷き詰めたパターンの正方形部分を切り出すだけではうまくいかない。
盤外に■があるときはその隣はサポートされないから。
この分で行くと十分大きなnに対して
N=n^2/4になるのではないかと予想できる。
>>555のN=(n^2 + 2n)/4は余分な■が2n個――つまり端っこ4辺/2の分だけいるということ。
かなり妥当な解だといえる。

561 名前:132人目の素数さん投稿日:02/11/30 19:23
557です。これは

>>552
ごめん、あなたの「最小の理由」がよくわからん。

の間違いです。というわけで、解説きぼん。

562 名前:558投稿日:02/11/30 19:37
N=8別解とN=9
□■■□□■■□  □
□□□□□□□□  □
■□□■■□□■  ■
□□□□□□□□  □
□■■□□■■□  □
□□□□□□□□  □
■□□■■□□■  ■
□××□□××□  □

□■■□□■■□□
N=8→n<=18

□■■□□■■□×  ■
□□□□□□□□□  □
■□□■■□□■■  □
□□□□□□□□□  □
□■■□□■■□×  ■
□□□□□□□□□  □
■□□■■□□■■  □
□□□□□□□□□  □
□■■□□■■□×  ■

□□□□□□□□□
N=9→n<=25

サポートされない×の部分がサポートされるように
■を付け足す(N=8のとき16+2,N=9のとき22+3)

563 名前:558投稿日:02/11/30 19:40
というのは嘘なんですがね

564 名前:132人目の素数さん投稿日:02/11/30 19:43
>>562
上から3、左から1の■が■と隣接してないよ!

565 名前:132人目の素数さん投稿日:02/12/01 02:12
オレにとってパチスロとは数学であり、数楽である。

566 名前:132人目の素数さん投稿日:02/12/01 03:29



567 名前:132人目の素数さん投稿日:02/12/01 06:45
パチスロってどのあたりが数学なの?

568 名前:132人目の素数さん投稿日:02/12/01 08:33
出目?

569 名前:132人目の素数さん投稿日:02/12/01 16:18
統計を数学に含めたくないっての?

570 名前:EVE ◆8ZTCcC3EVE 投稿日:02/12/01 20:19
>>561
ごめん、俺も問題設定あやふやになってきた・・・(汗)
というわけで、問題変えます(爆

半径1,2,3のいずれにも接する平面が2つある。
この二つの平面のなす角をθとするとき、tanθを求めよ。

これならいいかな。

571 名前:132人目の素数さん投稿日:02/12/01 21:37
パチスロって
仕組みを知ってる人間は回すだけで台の設定を分かるもんなのか?
仕組みを知ってる人間が台の設定を知ったら勝てるもんなのか?
統計学が関係するとすれば前者。数学が関係するとすれば後者。
目押しみたいな現実問題をどう定式化するかは数学の範疇じゃないと思われ。

後者が真ならパチスロは知識で勝てることになるから、バレたら出入り禁止では。
そうなるとバレない方法が問題になるから、統計でも数学でもない気がするが。
逆に後者がもし偽なら統計的な手法では勝てんかもよ。
仕組みを知っていて、小一時間回して設定を推定できたとしても
勝てる保証がないわけだし。


572 名前:132人目の素数さん投稿日:02/12/02 02:47
確実に「大負けしない」
これだけでも十分

573 名前:有名かな…投稿日:02/12/02 02:53
ある三桁の偶数は、何乗してもその下三桁が元の数に一致するという。それは何?
(答はメール欄)

574 名前:題意に加えてください@573投稿日:02/12/02 03:01
正の整数乗ならば…

575 名前:132人目の素数さん投稿日:02/12/02 03:01
カジノの方がまだ数学的

576 名前:132人目の素数さん投稿日:02/12/02 03:10
倍賭けし続ければいつかは勝てる。

577 名前:132人目の素数さん投稿日:02/12/02 03:30
>>576
それは資金が無限大でかつ
勝った時の報酬が負けたときのコストと等しいときにだけ使える技

とかマジレスしてみる

578 名前:132人目の素数さん投稿日:02/12/02 03:36
バイバイン(怒羅衛門)

579 名前:132人目の素数さん投稿日:02/12/02 04:05
いろいろな打ち方を試し
その結果を観測してより勝てる方法が導け出せるのであれば
パチスロには有効な学術的なアプローチがあるということになろう
じゃんけんなんかはそれがない
(個人の癖というものががないとするならだけど)

パチスロに数学が関係するか?統計が関係するか?
パチスロ自体に数学や統計が関係するかなぞはあまり意味を持たない
それに対して数学的にアプローチをするか
統計的にアプローチするかということである
暗号の解読に
その日の気分によって鍵を試してみるという
非学術的アプローチがあるように
だから

オレにとってパチスロとは数学であり、数楽である。

オレにとって、という部分が>>565の要点だと思うのです

580 名前:132人目の素数さん投稿日:02/12/02 06:33
てんで話にならねえ

581 名前:132人目の素数さん投稿日:02/12/02 12:29
>>573
c:\>ruby -e "(100..999).each{|i|p i if(i**2%1000==i)}"
376
625

582 名前:132人目の素数さん投稿日:02/12/02 12:37
2乗だけいえば十分なの?


583 名前:132人目の素数さん投稿日:02/12/02 12:41
数学的帰納法
= は1000を法にして合同ってこととすると
i**n = i**2 * i**(n-2) = i * i **(n-2) = i**(n-1)

584 名前:132人目の素数さん投稿日:02/12/02 12:48
c:\>ruby -e "m=1;while(1)do m*=10;(m/10...m).each{|i|p i if(i**2%m==i)}end"
1
5
6
25
76
376
625
9376
90625
109376
890625
2890625
7109376
12890625
Rubyだと、遅すぎてこれが限界。
任意の桁数について、N^2 ≡ N (10^(1+log_[10][N])) を満たす、整数Nは存在するだろうか?

585 名前:132人目の素数さん投稿日:02/12/02 13:26
十進法以外ではどうかという問題もあるかも。


586 名前:132人目の素数さん投稿日:02/12/02 16:04
5 + 6 = 11
25 + 76 = 101
376 + 625 = 1001

587 名前:132人目の素数さん投稿日:02/12/02 19:08
ところで>>553の問題って解けるの?
17角形ってどっかの歴史的にえらい数学者のやつだよね。


588 名前:132人目の素数さん投稿日:02/12/02 21:50
性17角形の作図は、ガウスがある朝、ベッドから起きたときにひらめいたってヤツだよな。

589 名前:132人目の素数さん投稿日:02/12/02 23:20
>>586
a^2 ≡ a ならば、(1-a)^2 ≡ 1-2a+a^2 ≡ 1-a


590 名前:某平成教育委員会の問題風投稿日:02/12/06 20:38
20グラム、15グラム、10グラム、5グラムの4つの砂袋があります。
A君、B君、C君、D君にそれぞれ砂袋を渡します。ただしこの時全員が渡された砂袋の重さはだれにもわからないように渡しました。
そこでA君とB君とC君とD君は天秤を使って自分の砂袋の重さを決定しようと考えました。
その結果
B君の砂袋よりA君の砂袋のほうが重く、
D君の砂袋よりA君の砂袋のほうが重く、
D君の砂袋よりB君の砂袋のほうが重いことがわかりました
しかしここでB君とC君の砂袋の中身が混ざってしまいました。
ところがこの時だれもが「自分の砂袋の重さがわかるぞ」といいました。
なぜでしょう。

ってわかるよね。ごめんクソで。

591 名前:132人目の素数さん投稿日:02/12/06 21:42
クソ

592 名前:132人目の素数さん投稿日:02/12/07 17:16
>>590
各人に渡した砂袋が20グラム、15グラム、10グラム、5グラムのどれかである
ということを明らかにしてないと、C君の砂袋の重さが他の砂袋とどのような関係で
あるかはわからないよ。と一応言ってみる。

593 名前:132人目の素数さん投稿日:02/12/08 04:26
>>587
長さは一意に定まるから、解ける。

594 名前:565投稿日:02/12/08 04:30
>>579
流石だね。あんたが最後に言ったとおりだよ。
オレは自称、平成のパスカルである。
ギャンブルは全て数学としてアプローチして楽しんでるよ。
そうした方が確実に勝てるんだよ。
ギャンブルは勝たなきゃ面白くないだろ?

595 名前:579投稿日:02/12/09 06:32
おう
でも保険金とかは数学的に考えるなよ。
ニュース増えるから(笑)

596 名前:132人目の素数さん投稿日:02/12/09 15:05
c は |c|<2 の実数の定数として、2次の正方行列Aが
A=(0 -1)
  (1  c)
で定められている。このときA^nを求めよ。
必要ならば c を適当な関数を用いて別の文字で置き換えてよい。(t=1+c^(1/3) など)

適当にやってたら結果がきれいになったので出してみました。
やり方によってはきたなくなるけど、c の置き方しだいだね。

597 名前:596投稿日:02/12/09 19:49
ってゆーか問題出してからこの置換気付くのか?
と思ったのでどのように置換するか(メール欄)に書いておく。
見たくない人は自力でやってみて。

598 名前:132人目の素数さん投稿日:02/12/10 02:40
(∵ )

599 名前:132人目の素数さん投稿日:02/12/10 02:58
>>596-597
なんだコイツ?

600 名前:ガイシュツならゴメソ投稿日:02/12/10 12:44
どっかの本に載ってたやつ。厨房レベルの癖に1分もかかった。
辺ABを斜辺とする直角三角形があります。辺AC、辺BC上にAD=BEかつ辺ABと辺DEが平行になるようにD、Eをおく。ADの長さをAC、BCを用いて表せ。

おわびと訂正
>>590間違えました。ちゃんと推敲すべきだった。
B君の砂袋よりA君の砂袋のほうが重く、
D君の砂袋よりA君の砂袋のほうが重く、
”C”君の砂袋よりB君の砂袋のほうが重いことがわかりました
です。すみません。
ちなみにこれもパクリです。すみません。

今度こそオリジナル作るぞい。


601 名前:132人目の素数さん投稿日:02/12/10 12:48
(x-a)(x-b)/(c-a)(c-b) + (x-b)(x-c)/(a-b)(a-c) + (x-c)(x-a)/(b-c)(b-a)
を簡単にせよ。(制限時間30秒)

602 名前:132人目の素数さん投稿日:02/12/10 13:12
1

603 名前:132人目の素数さん投稿日:02/12/10 15:57
f(x)=(x-a)(x-b)/(c-a)(c-b) + (x-b)(x-c)/(a-b)(a-c) + (x-c)(x-a)/(b-c)(b-a)
とおいたら f(a)=f(b)=f(c)=1 で f(x) は2次以下だから...
脳内所要時間 5秒

604 名前:132人目の素数さん投稿日:02/12/10 19:25
>>600
前半、何か条件が違わないか?
後半、それでもD、B、C君のが決まらない気がするが‥

605 名前:132人目の素数さん投稿日:02/12/10 20:40
激しく概出かもですが

任意の四辺形の各辺に正方形を作る。向かい合う位置にある正方形の中心
を結ぶ線分は互いに直交し、かつ長さが等しい事を証明せよ

606 名前:600投稿日:02/12/11 12:56
再び御免。最近、謝ってばっか。
>>590の問題とけます。あとで考えてみたらとけました。
うーん、「間違った問題=解けない」と思い込み、>>592の言葉に惑わされてしまった自分に反省。

>>604
前半の条件は(本に載ってたやつでは)AD=CEだったかも知れません。ただ答えはおなじになります。
ようするに「辺AC上にDを、辺BC上にEをおくのですが、この時AD=BEでABとEDが平行になる。」ってこと。
絵をかけば一発でわかるのですが。
あと後半の問題(>>590も含め)べつに「A君B君C君D君のそれぞれの砂袋のおもさを求めよ」とはいっておりません(前回のレスでこういうべきだった。すまん)。
問題は「何故決められるのか?」という意味です。問題を良くよんで!
(そうそう、答えに「おもさをはかる」ってのはなしね。使えるのは天秤だけ。)

PS
>>590でうっかり砂袋を混ぜてしまうどあほうはこの世にいるのでしょうかね?(何!俺だけ!?)



607 名前:132人目の素数さん投稿日:02/12/11 15:15
>>606
まあおちけつ。

前半:B君とC君の砂袋の中身が理想的に均等に混ざり合ったとかいう条件はなくていいのか?
後半:AC、BC上にAD=BEかつAB//DEになるようD、Eが置けるのは
   直角二等辺三角形の場合だけで、任意の直角三角形では不可のような気がするのだが?



608 名前:132人目の素数さん投稿日:02/12/11 16:22
辺ABを斜辺とする直角三角形があります。辺AC、辺BC上にAD=BEかつ辺ABと辺DEが平行になるようにD、Eをおく。
DE並行ABなら
直角2と右辺3角形じゃなくちゃ成り立たないよ

609 名前:132人目の素数さん投稿日:02/12/11 16:25
ADの長さをAC、BCを用いて表せ
ADは変数なのに
定数で決まるなんておかしくない?

610 名前:132人目の素数さん投稿日:02/12/11 19:05
>>605
・垂直・長さについて
複素数平面上の4点をA(0),B(2),C(2α),D(2β)とする。このとき、
四辺形ABCDについて考える。
辺ABに対する正方形の中心をz_1とすると、z_1=2/√2(cos(-45)゚+isin(-45)゚)
辺ADに対する    〃  z_2とすると、z_2=2β/√2(cos45゚+isin45゚)
辺CDに対する    〃  z_3とすると、z_3=2β + 2(α-β)/√2(cos45゚+isin45゚)
辺BCに対する    〃  z_4とすると、z_4=2 + 2(α-1)/√2(cos(-45)゚+isin(-45)゚)
整理して、z_1=1-i,z_2=β(1+i),z_3=β(1-i)+α(1+i),z_4=(1+i)+α(1-i)
このとき、z_1とz_3を結ぶ直線と、z_2とz_4を結ぶ直線が直交し、その
線分の長さが等しいことを示す。
(z_3-z_1)/(z_4-z_2)を調べると、z_3-z_1=α(1+i)+(β-1)(1-i)
z_4-z_2=(β-1)(-1-i)+α(1-i)=-i{(β-1)(1-i)+α(1+i)}=-i(z_3-z_1)
よって、(z_3-z_1)/(z_4-z_2)=-i
これより、線分(z_3-z_1)は、線分(z_4-z_2)の-i倍であるので、
長さが等しく、垂直である。

ごめそ、低脳な漏れにはここまでだ。(交点を持つことを証明できない!)


611 名前:132人目の素数さん投稿日:02/12/11 21:10
>>610
交点を持たない四角形が存在します。


612 名前:132人目の素数さん投稿日:02/12/11 21:30
>>610
複素平面を使うのはすばらしいアイデアですね。
でしたら、任意の四角形の各辺のベクトルを,a,b,c,d
として、a+b+c+d=0 となるように考えたらいかがでしょうか

613 名前:132人目の素数さん投稿日:02/12/11 22:36
それで一周して元に戻ってくるって意味になるんだろうな・・・

614 名前:132人目の素数さん投稿日:02/12/11 22:56
>>613
都合上 四辺形のある点を原点に選び、ここから2a+2b+2c+2d=0 が四辺形になる条件
(cを2aの始点とする)
また2aのところにできる正方形の中心はp=a+iaで示せる
という意味です

615 名前:132人目の素数さん投稿日:02/12/11 22:58
訂正(0 を2aの始点とする)


616 名前:再びm(_ _)m <スマーン投稿日:02/12/12 15:07
>>607
砂袋の問題:
は?完全に(理想的に)混じったかどうかは関係ないっしょ?
例えば10グラムの水と20グラムの油を混ぜたら30グラムになるのは当然。
まさか君は25グラムになるとでもいいたいのか?

三角形の問題:
すまん。AD=CEが正しかった!しかもガイシュツ(だいぶ前だが)であることが発覚!!
無視してください。
後半ちなみにその場合の答えAD=AC×BC÷(AC+BC)
騒がせてすまん。

付け加え
>>609
条件がついているので”変数”ではありません。
一見変数(あるいは自由に変わる数)に見えて実は定数だった。なんて経験を持つわし。

これじゃ低能な荒しと変わらんな。暇じゃなかったら放置キボン。

617 名前:132人目の素数さん投稿日:02/12/12 16:21
>>616
砂袋:で、結局どの問題が正しいんだ?

618 名前:関西投稿日:02/12/12 18:15
「教えてーな」はしばしば「教えてーや」と言われることがある。板違いなのでsage。

619 名前:132人目の素数さん投稿日:02/12/13 00:25
600は問題だすのはいいんだがもっと推敲してな

620 名前:132人目の素数さん投稿日:02/12/13 00:30
>>600
AD*(AC-BC)とか出てきたんだが・・・?

621 名前:132人目の素数さん投稿日:02/12/13 00:44
AD*(AC-BC)=0な

622 名前:132人目の素数さん投稿日:02/12/13 14:37
>>617
初めにだしたやつが間違い後で訂正したやつが正しいのだが、
両方とけることはとける(まぁ、偶然だが)。ただ解答は異なる。

>>619
はい。反省します。
>>620
ごめん。時間のムダ使いさせて。
>>621
いや、ほんと悪いと思ってる。
今後こんなことがないように(でもあるんだろうな)気をつけます

623 名前:132人目の素数さん投稿日:02/12/13 15:54
ティモテが、駄スレリストに認定しますた (゚∀゚)ティモテ!

       /⌒彡      谷山! 志村! よそう!!
     / 冫、 ))       しゃあねえからしぬ               ∧∧∧∧
    / ~ヽ ` , (((( ティモテ    アデール群、イデール群         <   禿 >
    | \ y  ))))  ティモテ〜   胸くそ悪い証明             < と し >
    |   ニつ))つ        数学書をパクりやすい店         < 痛 く  >
    |、ー‐ <((        聖書を数学的に咀嚼せよ           < 感 駄 >
    /   ヾ \、    数学つまらなすぎ                 <  !!! ス >
  // しヽ__)〜  【重大な】数学の基本はなんですか【シツモソ】 <    レ >
 ~〜〜〜`      数学てかっこいい!!16歳の女子ですよー       ∨∨∨∨

624 名前:132人目の素数さん投稿日:02/12/13 17:33
>>622あんまり落ち込まんようにな。次から注意すればいいんじゃない?
>>606まあ、Aは20グラムって決まるから、
Aと(B+C)の砂袋を天秤に乗せてどっちに傾くかで分かる罠

625 名前:132人目の素数さん投稿日:02/12/14 23:27
age

626 名前:132人目の素数さん投稿日:02/12/17 07:37
afe

627 名前:132人目の素数さん投稿日:02/12/17 11:20
>>624
下らんレスにこたえてくれてありがと。
ちなみに>>590のこたえは?ってわかる罠。


628 名前:132人目の素数さん投稿日:02/12/17 11:33
VAIO+VALUESTAR×Prius=???

629 名前:FUCKIN’投稿日:02/12/17 11:35
1分で倍に分裂するボールが、瓶に入っています。
60分で一杯になります。
瓶の半分になるには、何分かかりますか?

630 名前:132人目の素数さん投稿日:02/12/17 11:59
おーマイダーリン

631 名前:132人目の素数さん投稿日:02/12/17 14:18
図のようにマッチ棒を並べます。1本のマッチ棒を動かして、1を表すようにしてください。ただし、動かす1本のマッチ棒以外に触れてはいけませんし、分数を表す横棒のマッチを動かしてはいけません

コレの答え教えれ

632 名前:わからん。。。。投稿日:02/12/17 14:49
砂袋の問題わからん。A>B>Dの順番でCがどこかの間に入るのはわかって。
Cが20のときにA+D<B+Cとなって、Cが15か10のときにC+B=A+Dになって、Cが5のときにC+B=AになるのはわかるけどCが15もしくは10のときにどっちなのかわからない。

633 名前:132人目の素数さん投稿日:02/12/17 16:03
>>632
>>600の訂正ミル!

634 名前:132人目の素数さん投稿日:02/12/17 16:05
>>631図のようにマッチ棒を並べます
ってまさか>>630のことか??

635 名前:132人目の素数さん投稿日:02/12/17 22:04
>>629五十九分
>>631立てる

636 名前:132人目の素数さん投稿日:02/12/18 06:53
図はいずこに?

637 名前:132人目の素数さん投稿日:02/12/18 19:15
>>635
>>629はネタの答えを期待してたと思われ

638 名前:132人目の素数さん投稿日:02/12/18 19:32
>>629

11ヶ月

639 名前:132人目の素数さん投稿日:02/12/18 22:52
>>629
3540seconds

640 名前:132人目の素数さん投稿日:02/12/19 00:41
>60分で一杯になります。
>>一時間で一杯になります。

641 名前:即答投稿日:02/12/19 01:02
>>629
59秒

642 名前:132人目の素数さん投稿日:02/12/19 13:09
>>641
分だろ

643 名前:132人目の素数さん投稿日:02/12/20 04:20
「袋にチップが1枚だけ入っていてその色は半々の確率で赤か白だと
いうことだけが分かっている。この袋に白いチップを1枚入れて袋を
振ってからチップを1枚とりだしたら白だった。いま袋に残っている
チップが白である確率を求めよ」

これに類似した問題は高校でもやるんだっけ。

644 名前:132人目の素数さん投稿日:02/12/20 22:06
赤白→赤→捨て
赤白→白
白白→白
白白→白
2/3
こんな問題を書いたからには
>>643は1/2説を提唱する義務がある

645 名前:132人目の素数さん投稿日:02/12/20 22:23
どうでもいいが
運がいいやつは本当にいいか
自分でそういう環境を作ってるらしい
統計と風水っぽいので
なんかもっとも縁起のいいようにするらしい
しかも本当に運がいいから困る
でも運が悪くてもいいから俺は天才になりたい
運は猛烈に悪いけど

646 名前:132人目の素数さん投稿日:02/12/21 00:29
覆面算。

サセテサセテ+イイヨイイヨ=イクワイクワア

解をすべて出してね。

647 名前:132人目の素数さん投稿日:02/12/21 00:58
>>576
>>577
あと、実際のルーレットには0と00がある事をお忘れなく・・・・

648 名前:投稿日:02/12/21 01:00
さて問題です。
@3桁の数字を思い浮かべてください。(百の位の数字は一の位の数字より大きくしてね)
A次にその数字をひっくり返して、最初の数字からひっくり返したものを引いてください。
B Aで出てきた数字をまたひっくり返して、今度は足してみてください。

出てきた数字はいくつになりました?

649 名前:132人目の素数さん投稿日:02/12/21 01:48
>>647
そうなんだよ0と00があるんだよ。
世の中楽しまなくちゃ。
その前に石原さんかな?プッシュプッシュ・・・もう要らない?
>>648


650 名前:132人目の素数さん投稿日:02/12/21 01:50
>648
1089?

651 名前:132人目の素数さん投稿日:02/12/21 05:33
3桁の数を a*100+b*10+c (a>c)とする
a*100+b*10+c - c*100+b*10+a = (a-c-1)*100+9*10+(10+c-a)
a>cなので0<=(a-c-1)<10 、0<(10+c-a)<10
(a-c-1)*100+9*10+(10+c-a) + (10+c-a)*100+9*10+(a-c-1)
= (a-c-1 + 10+c-a)*100 + (9+9)*10 + (10+c-a + a-c-1)
= (-1+10)*100 + 18*10 + (10-1)
= 1089

(a-c-1)9(10+c-a)
(10+c-a)9(a-c-1)
1089

652 名前:132人目の素数さん投稿日:02/12/21 08:39
>>643はどこへ逝った

653 名前:132人目の素数さん投稿日:02/12/21 09:51
「袋にプッチモニが1枚だけ入っていてその色は半々の確率で赤か白だと
いうことだけが分かっている。この袋に白いプッチモニを1枚入れて袋を
振ってからプッチモニを1枚とりだしたら白だった。いま袋に残っている
プッチモニの..ッが白である確率を求めよ」

これに類似した問題は高校でもやるんだっけ。


654 名前:132人目の素数さん投稿日:02/12/21 18:49
>>644
今、袋には「白のチップ(A)」と「白か赤のチップ(B)」が入ってる。
どっちを取り出すかは1/2。
今Aを取り出したとしよう。じゃ、袋にあるのはBで白であるのは1/2。
故に(1/2)*(1/2)=1/4
次にBを取り出したとしよう。これが白であるのは1/2。
じゃ、袋にあるのはAだから白である確率は1。
故に(1/2)*(1/2)*1=1/4
互いに排反なので求める確率は(1/4)+(1/4)=1/2

あと、条件付確率の問題として解いたらどうなるのかな2/3になるのかな。
てことはこの解法が間違ってるわけだな

655 名前:132人目の素数さん投稿日:02/12/21 18:52
「これが白であるのは1/2。」

ここかな。既に白であったのだし。


656 名前:132人目の素数さん投稿日:02/12/21 18:53
じゃ、どうあの解答を変えるといいのだろうか

657 名前:132人目の素数さん投稿日:02/12/21 19:15
654では0点
思いっきり「条件付確率の問題」の問題だよ

658 名前:132人目の素数さん投稿日:02/12/21 19:29
今日から宿題やるぜ
誉めろ    

659 名前:132人目の素数さん投稿日:02/12/21 19:30
>>658
今更夏休みの宿題かよ。おめでてーな。

660 名前:132人目の素数さん投稿日:02/12/21 19:36
袋にチップが2枚入っている。チップの色は赤か白であるということしか
分かっていない。チップを袋からとりださずにそれぞれの色を求めよ。



答え: 白と赤が1枚ずつ

もし袋にチップが3枚入っていて、そのうち2枚が白でもう1枚が赤とすれば、
袋からチップを1枚とりだしたときそれが白である確率は2/3となる。
これ以外の場合は2/3にならない。つまり3枚から1枚とりだしたときに
それが白である確率が2/3なら、中身は白白赤である…(*)。

2枚のチップの場合、赤赤、赤白、白白である確率は1/4、1/2、1/4である。
ここに白のチップを1つ入れる。すると中身が、白赤赤、白白赤、白白白に
なる確率はやはり1/4、1/2、1/4である。いま袋からチップを1枚とりだして、
それが白である確率は、(1/3)(1/4) + (2/3)(1/2) + (3/3)(1/4) = 2/3

よって(*)より白を入れた後の袋の中は、白白赤、ということになるので、
最初から入っていたチップの色のは、白と赤が1枚ずつだったことになる。

661 名前:132人目の素数さん投稿日:02/12/22 00:02
>>657
分かってたよ
>>643は1/2説を提唱する義務がある」
をやって見たかっただけ。
ほかに挑戦するヤシいねえかあ?

662 名前:132人目の素数さん投稿日:02/12/22 00:22
問題
ルービックキューブの操作の仕方は何通り?

663 名前:132人目の素数さん投稿日:02/12/22 00:35
43,252,003,274,489,856,000

664 名前:132人目の素数さん投稿日:02/12/22 11:09
>>663
それは面の色の組み合わせ?

665 名前:132人目の素数さん投稿日:02/12/22 11:43
さて問題です
使っていない画用紙は?


666 名前:132人目の素数さん投稿日:02/12/22 23:56
>>665
計算用紙に

667 名前:132人目の素数さん投稿日:02/12/23 02:21
問題です
白粉を塗った顔は?

668 名前:132人目の素数さん投稿日:02/12/24 00:34
>>667
最近めっきり老けたバカ殿です

669 名前:132人目の素数さん投稿日:02/12/24 00:36
3通りくらいじゃないの

670 名前:132人目の素数さん投稿日:02/12/24 01:32
俺の実力は
http://www2.biglobe.ne.jp/~optimist/
ちょっと見せるとこんなもんです。
これからもどんどん見せていきます。


671 名前:132人目の素数さん投稿日:02/12/24 01:38
>>670
そのHP作者に迷惑かかるようなことすんなヴォケ
誰かにウイルスメールでも送らせる魂胆だろが

672 名前:132人目の素数さん投稿日:03/01/01 21:23
>>1に書いてある過去スレVol.2に書いてあった
ttp://www.adventurespace.net/joker2.html
の問題が面白かった。

673 名前:132人目の素数さん投稿日:03/01/02 22:24
ttp://ww2.wt.tiki.ne.jp/~keixx/quiz/quiz.html
も面白かった。

674 名前:132人目の素数さん投稿日:03/01/05 20:09
最近受験数学ばっかだね。新作期待あげ。

675 名前:132人目の素数さん投稿日:03/01/05 20:58
大昔に、「自転車の前輪がある曲線を描いたとする。
後輪はどういう曲線になるのか」という問題を見たことがある。
これって有名な問題?

676 名前:132人目の素数さん投稿日:03/01/05 20:59
両方サイクロイドじゃないの?

677 名前:675投稿日:03/01/05 21:02
>>676
地面につく跡の話だよ。


678 名前:132人目の素数さん投稿日:03/01/05 21:07
x radian シフトしたってこと?

679 名前:132人目の素数さん投稿日:03/01/05 21:11
自転車の前輪と地面の接点がある曲線(例えばy=f(x))上を動いたとする。
後輪と地面の接点はどういう曲線を動くか?じゃなかったかな。

680 名前:132人目の素数さん投稿日:03/01/05 21:14
y=f(x)の形よりパラメータ表示したほうが良さそう。
x=f(t),y=g(t) t>=0

681 名前:132人目の素数さん投稿日:03/01/05 21:19
俺は聞いたことないなぁ・・・

682 名前:132人目の素数さん投稿日:03/01/05 21:46
え?前輪と一緒じゃないの?同じ道通るよね

683 名前:679投稿日:03/01/05 21:47
>>682
泥道を自転車で走ったら、タイヤの跡が一致するかい?

684 名前:132人目の素数さん投稿日:03/01/05 21:51
少し真ん中にしぼむような感じでしょ。自転車

685 名前:投稿日:03/01/05 23:49
数当て
@まず適当な4桁の数字を作ってください。
A次に@で作った数の数字を入れ替えてもう一つ
 4桁の数字を作ってください。
B2つの数の大きい方から小さい方を引いて下さい。
Cその答えの4桁の中から3つの数字を書き込んで下さい。
D残りの1つを漏れが当てます。

686 名前:132人目の素数さん投稿日:03/01/06 00:01
>>685
一発で当てれるのかな。
2と7と9

687 名前:投稿日:03/01/06 00:07
>>686いきなり難しい‥
ええぃ、0か9

688 名前:132人目の素数さん投稿日:03/01/06 00:09
0と0と0。
ってゆーかムリじゃねぇ?

689 名前:投稿日:03/01/06 00:12
>>688
0でしょ?

690 名前:686投稿日:03/01/06 00:13
>>687
どちらか一つに絞ることはできる?
元の数字、でなく残りの一つ、ってのがミソなのかな。

691 名前:投稿日:03/01/06 00:19
>>686
スルドゥイ‥、悪いが2と7と9の場合
一つに絞れないんだす。いきなりだから知ってるのかと思った
他の数字なら一発であてるYO!


692 名前:686投稿日:03/01/06 00:23
元の数字 3141(π)
入れ替えたやつ 4113
引いた4桁の数字 0,2,7,9
でした。

じゃあ、0,3,9なら?

693 名前:132人目の素数さん投稿日:03/01/06 00:24
>>685
んじゃあ試しに。3,6,7

694 名前:投稿日:03/01/06 00:25
6だYO〜〜〜!!

695 名前:投稿日:03/01/06 00:27
スマン694は>>692でつ。
>>693は2だろ〜?


696 名前:132人目の素数さん投稿日:03/01/06 00:29
>>694
当たり。
2371 - 1732 = 0639 でした。。
そろそろ理屈を考えてみるか。

697 名前:132人目の素数さん投稿日:03/01/06 00:33
9の倍数

698 名前:686投稿日:03/01/06 00:33
なるほど。696の場合、0,3,9じゃなく3,6,9と言われたら
即答できなかったわけだね?>Q

699 名前:693投稿日:03/01/06 00:33
>>695
あたり。6523-3256=3267です。
さて、なぜわかるのやら。


700 名前:686投稿日:03/01/06 00:33
先越された。

701 名前:投稿日:03/01/06 00:40
>>698
その通りでつ。

702 名前:132人目の素数さん投稿日:03/01/06 00:42
>>689
0010-0001もあるね。

703 名前:投稿日:03/01/06 00:48
>>702
ハッ!危なかった!てっきり
aaaa-aaaaかと思ってたYO




704 名前:132人目の素数さん投稿日:03/01/06 15:03
9の倍数の意味がわからない訳なんだが
オレだけか

705 名前:132人目の素数さん投稿日:03/01/06 21:36
桁を入れ替えても9で割った余りは等しい。
引き算すれば余り0

706 名前:132人目の素数さん投稿日:03/01/06 22:31
差が9の倍数

707 名前:132人目の素数さん投稿日:03/01/06 23:21
解答キボ〜ンヌ。

708 名前:132人目の素数さん投稿日:03/01/06 23:23
引っくり返したものの数値の差は9の倍数になる。

709 名前:132人目の素数さん投稿日:03/01/07 05:33
n人をグループに分ける。
グループは何個できてもいいし、人数ばらばらでもいいし、
人数が1人でもいい。
全員でn人のとき、分け方は何通りあるか?

一般項は難しそう。つーか漸化式もでないかな…
ちなみにn=1,2,3,4のときそれぞれ1,2,5,16通りです

710 名前:132人目の素数さん投稿日:03/01/07 06:00
新数学演習か 解法の探求・確率編に載ってたよ
これを さらに難しくした問題も知ってる
とりあえず書店に行って調べてみな
自分で調べるのも勉強のうち、がんがれ!

711 名前:132人目の素数さん投稿日:03/01/08 03:01
>>710
>>709へのレスかな?考えてます。
これ関数の総ての変数単一化の組み合わせの導出に使いたいんだけど。
生成木は書けます
X,Y,Z
-Y,Z[X]
--Z[XY]
---[XYZ] No.1
---[XY,Z] No.2
--Z[X,Y]
---[XZ,Y] No.3
---[X,YZ] No.4
---[X,Y,Z] No.5
(あとn=4のときは15通りでしたね。スマソ)
a(0,l)=l
a(k,l)=(k-1)a(k-1,l)+a(k-1,l+1)
としてnのときa(n,0)通りとか?これってアッカーマン関数みたいなものかなあ…

712 名前:132人目の素数さん投稿日:03/01/09 16:27
>>685さんによく似た問題(外出ならスマソ)
1.適当な3桁の数字を考えてください。ただしこのとき1桁目と3桁目の数字は異なるようにしてください。(例:427)
2.1.で考えた数字の1桁目と3桁目の数字を入れ替えて新しく3桁の数字を作ってください。(例:724)
3.1.と2.の数字のうち大きいほうから小さいものを引きます。(例:724−427=297)
4.3.でつくった数字の1桁目と3桁目の数字を入れ替えて新しい数字を作ってください。(例:792)
5.3.の数字と4.の数字を足してください。(例:297+792=1089)
6.漏れがその答えを予想。(例:その答えは5.の例と同じですね?)
7.もし違った香具師がいたら、1.〜5.までの手順をよく読んでもう一度やってください。(例:895になったぞゴルァ。ナヌ計算ミス?)
8.余力があれば証明してください。(例:メンドイ省略。 Σ(´Д`)スマソ)


713 名前:132人目の素数さん投稿日:03/01/09 16:50
>>712
いきなりネタばらし。
99*a+99*(11-a)=1089=const.
aは最初の3桁の数の1の位と3の位の差

714 名前:132人目の素数さん投稿日:03/01/09 20:14
x^4 + y^4 + z^4 = w^4
を満たす自然数x、y、z、wの組をひとつもとめよ




715 名前:132人目の素数さん投稿日:03/01/09 21:30
2682440^4+15365639^4+18796760^4=20615673^4

716 名前:132人目の素数さん投稿日:03/01/09 21:32
正解!

717 名前:132人目の素数さん投稿日:03/01/09 21:32
朝飯前だぜ

718 名前:132人目の素数さん投稿日:03/01/09 22:29
どうやって求めるのだ?

719 名前:132人目の素数さん投稿日:03/01/09 22:36
ないしょ

720 名前:132人目の素数さん投稿日:03/01/09 23:05
夕飯後だったぜ!

721 名前:132人目の素数さん投稿日:03/01/11 06:57
>>709は分かったのだろうか?
この問題は、なかなか面白いんだけどね

722 名前:山崎渉投稿日:03/01/11 12:12
(^^)

723 名前:132人目の素数さん投稿日:03/01/11 19:35
>>721
>>711に漸化式だけ書いたんだけど間違ってるみたい。
一人ずつ順番に、
今あるグループのどれに入りたいかを宣言する、または、
自分ひとりの新しいグループを作ると宣言する、
のいずれかをしていけばいいです。
まず1人目はグループが無いから新グループを作るしかないんです。
でもってこの時点では1通り。
2人目は1のグループに入る(A2)か、新グループ作成(B2)の2通り、
3人目は(A2)のとき1,2のグループに入る(A3)か新グループに入る(B3)か、
(B)のとき1に入る(C3)か,2に入る(D3)か,新(E3),
4人目は(A3)に対して、(B3)に対して、(B4)に対して…うきゃーーー!!!
ってなるんですよ、ほんとに
どうすればいいんですか

724 名前:132人目の素数さん投稿日:03/01/11 21:16
>>723
すまんが、>>709の問題について少々ツッコませてもらう。
組合せの数なら、a(1)=1、a(2)=2、a(3)=2、a(4)=5 だぞ。
a(1)=1、a(2)=2、a(3)=5、a(4)=15 となるのは順列。
なぜなら3人の場合、組合せは(1,1,1)、(1,2)の2通りしかないから。
全員に名前を付けた場合に、はじめて a(3)=5 となる。

725 名前:724訂正投稿日:03/01/11 21:23
3人の場合、組合せは(1,1,1)、(1,2)、(3)の3通りあるね。
a(1)=1、a(2)=2、a(3)=3、a(4)=5 ですた。

726 名前:132人目の素数さん投稿日:03/01/11 22:25
全員に名前つけた場合
n=1の時 1
n=2の時 1+1
n=3の時 1+(1+1*2)+1
n=4の時 1+{1+(1+1*2)*2}+〔(1+1*2)+1*3〕+1
分かれたグループに人が何人いるか考え、1人のグループを@をいう風にすると
それぞれのグループの個数が何個あるかが求めるa(n)通り
n=kには
@は1個(これは常に1)
Aはn=k−1の@の数*1+Aの数*2
Bはn=k−1のAの数*1+Bの数*2
Cはn=k−1のBの数*1+Cの数*2



kマルはn=k−1のk-1マル*1+kマル*2
@〜kマルまでの個数を足す。
これで出したのが上の式の意味。こんがらがりやすいから気をつけてな。
一般項は良く分からなかった。誰か出して!もうすぐ出る予感

727 名前:132人目の素数さん投稿日:03/01/11 23:54
問 実数 R の次のような部分集合について調べ答えよ。

 1.t∈Rが代数的数であることの定義を述べよ。
   また、代数的数の全体がNと対等であることを示せ。

 2.t∈Rが超越数であることの定義を述べよ。
   また、超越数の全体はRと対等であることを示せ。

 3.カントール集合Cの定義を述べよ。
   また、CはRと対等であることを示せ。
   

728 名前:132人目の素数さん投稿日:03/01/12 00:08
>>726
間違えた
「1人」のグループを@をいう風にすると

n人が一つのグループにいる時を@、二つに分裂してたらAという風にすると
n+1人目は@の時@にそのまま入って@になるか、新グループ作ってAになるか。
Aのときは一方のグループに入ってA、もう片方のグループでA、新グループ作ってBになるか。
ってことが言いたかった。

729 名前:132人目の素数さん投稿日:03/01/12 00:29
人間A,B,C,D,Eが居る。この5人を階段に一段ずつ前を向いて立たせ(図)、赤白2色の帽子をかぶせた
彼らには「赤帽をかぶせたのは2人、白帽をかぶせたのは3人」という情報だけがあたえられている
このとき一人だけ自分がかぶせられた帽子の色を答えられたとすると、それはA〜Eのうち誰か
(ただしBとEの前には壁がつくられていて前が見えないとする。DからAが見えることもない)
      E
     D|●
前← C ●|□
 |B ● □|□
A|● □ □|□
●|□ □ □|□
□|□ □ □|□

>>70に似たものかもしれませんが、長方形に敷き詰められた図形を並び変えると面積が変わってしまう(25→24など)のトリックって誰か分かりますか?

730 名前:724投稿日:03/01/12 00:57
>>726>>728
だから、全員に名前を付けたりなんかしたら、>>709の問いとは異なるだろうが。
n=3 のときは、a(n)=3 にならなければおかしいでしょうに。

731 名前:132人目の素数さん投稿日:03/01/12 09:56
>>724
この手の問題はおかしなもので、
「n人の人間」なら、n人は全て名前の異なる個性あるものと扱い、
「n個のりんご」なら、n個は全て区別のつかない無個性のものと扱う、
という暗黙の了解があります。


732 名前:724投稿日:03/01/12 11:08
>>731
ええ!?そんなこと初めて聞いたぞ!!
そうなの?それなら>>728で合ってるよな。
鬱だ氏膿・・・・・

733 名前:132人目の素数さん投稿日:03/01/12 13:04
>>729
D。B・Cが赤帽被ってたらDは自分が白とわかる。

734 名前:132人目の素数さん投稿日:03/01/12 13:51
>>728
全く間違えてた。たびたびごめんなさい。
n=kには
@は1個(これは常に1)
Aはn=k−1の@の数*1+Aの数*2
Bはn=k−1のAの数*1+Bの数*「3」
Cはn=k−1のBの数*1+Cの数*「4」



kマルはn=k−1のk-1マル*1+kマル*「k」
@〜kマルまでの個数を足す。
n=1 (A)
n=2 (AB) (A,B)
n=3 (ABC) (AB,C) (AC,B) (A,BC) (A,B,C)
n=4 (ABCD) (ABC,D) (ABD,C) (AB,CD) (AB,C,D) (ACD,B) (AC,BD) (AC,B,D) (AD,BC) (A,BCD) (A,BC,D) (AD,B,C) (A,BD,C) (A,B,CD)
(A,B,C,D)
a(4)=15だね。>>726でも15になってるし。
一般項でないのかなあ?

735 名前:729投稿日:03/01/12 13:55
>>733
おしい!分かったのが一人というのがポイントです。
しかし、説明不足だったかもしれません
分かった人には『分かった』と言わせることにしましょう。つまり、それを判断材料に自分の帽子の色を考えてよいとする。
って、答え出ちゃいますね(汗

736 名前:132人目の素数さん投稿日:03/01/12 14:04
>>735
そっか。スマソ


だとしたらC。Dに見えている帽子は赤白→D(・・・・わかんねー)
Cに見えているBの帽子は赤→(DがわかんないのならBと漏れのどちらかが赤でどちらかが白・・・・漏れは白!)「わかった!」
B(え?え?Cは赤か白かどっちだったんだろ・・・?・・・・わかんねー)
A・E(・・・・これだけじゃサーパリわからん)

Cに見えているBの帽子が白でも成り立つかな?


737 名前:132人目の素数さん投稿日:03/01/12 14:07
>>735
どういう意味?
5人のうち誰の帽子の色が分かれば、全員の帽子の色が決定するか
っていう問題??分からん・・・

738 名前:132人目の素数さん投稿日:03/01/12 14:12
「△ABCの辺AC上に点Dを、辺AB上に点Eを取る。
線分BDの中点をM、線分CEの中点をNとするとき、
△AMNの面積は四角形BCDEの面積の四分の一倍と
なることを証明せよ」

739 名前:132人目の素数さん投稿日:03/01/12 14:51
>>738
AE→=sAB→、AD→=tAC→(1≧s、t≧0)として
四角形BCDE=三角形ABC-三角形AED=△ABC*(1-st)だから
△AMNをベクトルでごちゃごちゃして△AEDで表す。
って言う方針で求まりますか?

740 名前:sage投稿日:03/01/12 14:56
>>738
点Aを通り、MNに平行な直線と
直線MD,直線NDの交点をそれぞれ
F,Gとおくと
 △AMN
=△GMN
=△DMN+△GMD
=△DMN+△EMD
=四角形DEMN
=1/2・四角形BNDE
=1/4・四角形BCDE

で、どうかな?

741 名前:132人目の素数さん投稿日:03/01/12 15:01
△GMN
=△DMN+△GMD ??

742 名前:738投稿日:03/01/12 15:03
>>739
中2の問題なので、ベクトルを使わなくても解けるはずです。


743 名前:738投稿日:03/01/12 15:09
>>740
741さんと同じところが疑問です。

744 名前:132人目の素数さん投稿日:03/01/12 16:13
>>737
いや、そうじゃなくて「自分の帽子を当てる事が出来たのはたった1人だけ」というのがミソで
全員の帽子を決定する必要はない・・・・というより、決定されてはいけない。

733がアウトなのは、Dが「わかった!」と言うとそれを受けてB・Cも自分が赤帽を被っている事がわかってしまうからだ。



745 名前:729投稿日:03/01/12 16:36
>>736
正解です。
Bが白だとダメですね
>>737分かりにくくてスマソ
>>744
なかなか言葉にすると長くて大変です。原作?(聞いた時の話)は死刑囚の設定です。
しかも壁の位置が違いました(汗
複雑な設定にするとおもしろいかも知れませんが、本格的な「数学」ができないもので…

746 名前:132人目の素数さん投稿日:03/01/12 23:59
>>740
いつでもOKなのかな?それで。

747 名前:132人目の素数さん投稿日:03/01/13 12:06
>>738
場合分け無しで解けるの?

748 名前:132人目の素数さん投稿日:03/01/13 13:09
あれ?「B:白 C:白」もありますよ。
あとCは「赤白」のケースのみわかるのだから同時にBにもわかってしまいアウトのはず。
「白赤」「白白」のケースだとCもDもわからないのでB(白)のみ正解。

なんかこのノリで「5人中4人がわかるけど残りの一人はさっぱりわからない」
状況は発生するのかな?壁の区切りはお好きなように、誰の発声かはすべてわかるって条件で。

749 名前:132人目の素数さん投稿日:03/01/13 15:29
>>740
書いてる事は合ってると思うし場合分けもいらないと思うが、
なぜ僭MD=僞MDなの?
EGとBDの平行の証明は?メネラウス腐るほど使えば出来るかもしれんが。

750 名前:132人目の素数さん投稿日:03/01/13 15:56
ある家族がちょうどn人の子供をもつ確率PnをαP^nとする。ここで
n≧1, P0=1-αP(1 + P + P^2 + ・・・・)である。n人の子供の性別
分布は、すべて同じ確率をもつものとする。K≧1に対して、ある家族が
ちょうどK人の男の子をもつ確率は2αP^K/(2-P)^(K+1)であることを示せ。


751 名前:赤白問題作問者投稿日:03/01/13 22:35
>>748
(◎_◎)

752 名前:740投稿日:03/01/13 23:18
>>749
やっぱりバレました?メネラウスで示すくらいなら
地道な方法で求めたほうがスマートかなと。
問題作った方は、等積変形で示すことを
求める気がしまして。

EGとBDが平行になることのいい証明ありませんですか?>どなたか

753 名前:132人目の素数さん投稿日:03/01/13 23:32
ワイン飲みかた講座ですか?
ロゼが無いようですが。

754 名前:132人目の素数さん投稿日:03/01/15 02:17
>>グループ分け問題(>>709)
n=iのときにグループ数がjとなるわけ方の個数をg(i,j)と表すことにする.
ex.
g(1,1)=1 → 1
g(2,1)=1,g(2,2)=1 → 2
g(3,1)=1,g(3,2)=3,g(3,3)=1 → 5
g(4,1)=1,g(4,2)=6,g(4,3)=7,g(4,4)=1, → 15

一般にg(k,m)はg(k+1,m)に+m,g(k+1,m+1)に+1寄与する。
これでちょっと変わった筆算をすればどんどんだしていける.
g(5,1)=1,g(5,2)=11,g(5,3)=20,g(5,4)=9,g(5,5)=1 → 42
g(6,1)=1,g(6,2)=23,g(6,3)=71,g(6,4)=56,g(6,5)=14,g(6,6)=1 → 166
これを漸化式で書くと,

g(k,1)=1
g(k,m)=g(k-1,m-1)+mg(k-1,m) (k≧m>1のとき)
g(k,m)=0 (m>kのとき)

よって分け方の総数はΣ[n≦k≦1]g(n,k).
漸化式は出せたんだけどね…


755 名前:754投稿日:03/01/15 02:18
おれいつもageるの忘れる

756 名前:( _ _)/754投稿日:03/01/15 02:20
この関数のオーダーが知りたいキモチ…

757 名前:( _ _)/754投稿日:03/01/15 02:49
>>754
g51=1,g52=15,g53=25,g54=10,g55=1→52
g61=1,g62=31,g63=90,g64=65,g65=15,g66=1→=203
のミスでしたスマソ

758 名前:( _ _)/754投稿日:03/01/15 03:09
g(k,2)=g(k-1,1)+2g(k-1,2)=2g(k-1,2)+1
よりg(k,2)=2^(k-1)-1と判明.
g(k,3)=g(k-1,2)+3g(k-1,3)=2^(k-1)+3g(k-1,3)-1
ううん…

a(k+1)=2^k+3a(k)-1,a(3)=1
ってどうやって解くんだっけ。
これ導入つきの入試問題にいいかと思う。
もうねよっと☆ミ

759 名前:132人目の素数さん投稿日:03/01/15 05:03
>>758
> a(k+1)=2^k+3a(k)-1,a(3)=1
> ってどうやって解くんだっけ。

a(k+1)=3a(k)-1 の特性解1/2を使って
a(k+1)-1/2=2^k+3{a(k)-1/2}にしてから
両辺を2^(k+1) or 3^(k+1)で割るのが普通でない?

本当は
a(k+1)+2^(k+1)-1/2=3{a(k)+2^k-1/2}
と変形すれば
{a(k)+2^k-1/2}は等比数列だから簡単だけど。

760 名前:132人目の素数さん投稿日:03/01/15 06:55
g(k,m)=1/(k-1)!*Σ[k≦x≦1]{(-1)^(k-x)*C[k-1,m-1]*x^(m-1)}
従って
f(n)=Σ[n≦m≦1]{1/(n-1)!*Σ[n≦x≦1]{(-1)^(n-x)*C[n-1,m-1]*x^(m-1)}
だな

761 名前:132人目の素数さん投稿日:03/01/15 07:56
>>754
>一般にg(k,m)はg(k+1,m)に+m,g(k+1,m+1)に+1寄与する。
規則的だというのはわかったけど、そういう規則だとは気づきませんでした。
>>754-760
ついに出たね。やはりΣが2つ入るのか・・・オレにはここまで解けましぇん。

762 名前:740投稿日:03/01/16 01:40
>>749
やっぱり場合分け無しだと無理があるようです。
今のところ、この方の解法が有力な模様。
http://science.2ch.net/test/read.cgi/math/1042497098/233

763 名前:132人目の素数さん投稿日:03/01/18 03:34
>>760
g(k,m)=1/(k-1)!*Σ[1≦x≦k]{(-1)^(k-x)*C[k-1,m-1]*x^(m-1)}
どうやって これを出したんでしょうか?
ずっと考えてるんですが、まだ分かりません

764 名前:132人目の素数さん投稿日:03/01/18 10:36
結局、徹夜して考えてしまったけど、>>760の式がどうやって導かれたか分からずじまい…
漸化式を満たすか計算し直したところ、>760は誤植あり。
正しくは g(k,m)=1/(m-1)!*Σ[1≦x≦m]{(-1)^(m-x)*C[k-1,m-1]*x^(k-1)} …☆

「生徒のグループ分け」で検索すると、某HPに同じ問題があった
そこでは、k人をmグループに分ける分け方の総数はP(m,k)と変数の位置が逆になっていたから
>>760は、そこを考慮せずに式を写したときに文字を置き換え間違えたらしい

上の☆式が、どうやって導かれるか教えて下さい
確かに漸化式を満たしますが、天下り的でスッキリしません
よろしくおねがいします (さて、バイト逝ってくるか…、眠い…)

765 名前:132人目の素数さん投稿日:03/01/18 19:01
>>764
眠いままバイトに行くんじゃねえよ、ボケが。
仕事をなめんじゃねえぞ、こら!

766 名前:132人目の素数さん投稿日:03/01/18 19:37
AとBが袋から玉を一個ずつ取り出していき
先に白球を引きあてた者を勝ちとする。

引く順番はABBAABBAABBAA…とする。

今、袋の中には8個の黒球が入っている。

Bに最も有利であるようにするためには白球、黒球を何個ずつ加えれば良いか?

767 名前:132人目の素数さん投稿日:03/01/18 19:48
AとBが玉袋から一個ずつ取り出していき
先に赤球を引きあてた者を負けとする。

引く順番はABBAABBAABBAA…とする。

今、袋の中には8個の金球が入っている。

Bに最も有利であるようにするためには金球、赤球を何個ずつ加えれば良いか?


768 名前:132人目の素数さん投稿日:03/01/18 20:11
>>765=760

769 名前:132人目の素数さん投稿日:03/01/18 21:04
>>767
>>751

770 名前:132人目の素数さん投稿日:03/01/18 23:19
A4用紙あるいはB4用紙の縦横比はいくつか?

771 名前:132人目の素数さん投稿日:03/01/18 23:31
男女10人づつで総当りの乱交パーティをする。ただし、病気もちがいるので、
コンドームを使う。同姓愛者はいない。
最低何個のコンドームを用意する必要があるか

772 名前:132人目の素数さん投稿日:03/01/18 23:34
>>770
だいたい√2倍?

773 名前:132人目の素数さん投稿日:03/01/18 23:56
>>770
答えを先に言うと、
1:(1+√5)÷2 → 2:(1+√5)

実は770の問題はユークリッドの黄金比の問題であったりする。


774 名前:132人目の素数さん投稿日:03/01/19 00:06
>>773
嘘つけ。大体√2。これは裁断に便利だから。
ちなみに、名刺が大体黄金比になっている。

775 名前:132人目の素数さん投稿日:03/01/19 00:14
おっーーーーと意見が分かれますたーーーーー!!!!!!!

776 名前:132人目の素数さん投稿日:03/01/19 00:14
羅漢の心理学で、対象としての自己が黄金比で表されるとか何とか言うのを
読んだことがあるよ。さっぱり意味が分からなかったけどね。

777 名前:132人目の素数さん投稿日:03/01/19 00:19
√2に決まってるだろ。そうじゃなかったら、切るたびに縦横比が変わってしまう。

778 名前:132人目の素数さん投稿日:03/01/19 12:42
あるプログラムを実行すると無限ループになる可能性があるか否かを判断する
プログラムを作ることはできないと聞いたことがあります。
どなたか、簡単に証明をお願いします
他で書き込みしましたが反応がないのでここに書かせていただきます


779 名前:132人目の素数さん投稿日:03/01/19 13:56
僕は友人に2000円のものを立て替えて買ってあげました。
そのあと僕は友人に5000円渡しました。そして友人は6000円の物を買いました。
友人はいくら僕に返せばいいでしょう?

780 名前:132人目の素数さん投稿日:03/01/19 13:58
>>778
では訊くが、人は無限ループになる可能性があるか否かを
どうやって判断しているんだい?
そこに気づけば「プログラムを作ることはできない」等という愚答は
あり得ないけどね。

781 名前:132人目の素数さん投稿日:03/01/19 14:05
>>779
千円と六千円の物?

782 名前:132人目の素数さん投稿日:03/01/19 14:17
いや2000円のものです

783 名前:132人目の素数さん投稿日:03/01/19 15:05
僕は友人に2000円のものを立て替えて買ってあげました。
そのあと僕は友人に5000円渡しました。
そして友人は渡した5000円で6000円の物を買いました。
友人はいくら僕に返せばいいでしょう?




784 名前:132人目の素数さん投稿日:03/01/19 15:05
↑のは訂正です

785 名前:132人目の素数さん投稿日:03/01/19 16:14
7000円

786 名前:132人目の素数さん投稿日:03/01/19 16:25
>>780
人はどうやって無限ループにならないことを判断しているのでしょうか?
宇宙人が全く存在しないことを証明することが困難なように、
絶対に無限ループにならないことを証明するのは困難ではないでしょうか
人間の判断は網羅的ではないと思います。まだ残っている可能性を無視して
このプログラムは無限ループしないと判断しているだけだと思うのですが…


787 名前:132人目の素数さん投稿日:03/01/19 17:23
>>786
要するに、人間様でさえこのプログラムは無限ループするかどうかの判断は出来ない
とキミはこう言いたいわけだな。
それが正しいなら、君の言うとおり無限ループになる可能性があるか否かを判断する
プログラムを作ることはできないと主張してもいいよ。
そうでなければやはり愚答としか言いようがない。

788 名前:132人目の素数さん投稿日:03/01/19 18:47
>>778
質問の仕方が誤解を招く。

「ある特定のプログラムP」が停止するか無限ループするかを
判定するプログラムなら、Pの内容によっては作成可能。

しかし、「どんなプログラム」を与えても、それが停止するか
無限ループするかを判定するような、万能プログラムは作成不能。

789 名前:778投稿日:03/01/19 19:37
私はこの問題を放送大学でたまたまやっているのを見た記憶があります。
証明方法は背理法をつかって、もしそのような判定プログラムがあるとして、
そのプログラムの最後にある追加コーディングをしたものを、判定に
かけるとある矛盾が生じるから、そのような判定プログラムは存在しない、
といったストーリーでした。
この文脈でご回答をしていただけるかたいらっしゃいませんか


790 名前:788投稿日:03/01/19 19:57
停止判定プログラム H (P,x) が存在すると仮定する。

すなわちHは、プログラムコードPと、それへの入力xを引数に取り、
P(x)が停止するなら1を出力して停止、P(x)が無限ループするなら
0を出力して停止するものとする。

ここで、次のようなプログラムTをつくる。
いまHが存在すると仮定しているので、これは作成可能である。

T(z):= H(z,z)=0 のとき停止、H(z,z)=1 のとき無限ループ

このときT(T)を考えると、これが停止するとしてもしないとしても
矛盾が生じる。

791 名前:132人目の素数さん投稿日:03/01/19 20:34
一辺が10の正方形ABCDの辺ABを直径とする円と
点Cを中心とする半径10の円とで囲まれる面積を求めよ

792 名前:778投稿日:03/01/20 22:31
>>790
「このときT(T)を考えると、これが停止するとしてもしないとしても
矛盾が生じる。」の意味を詳しく教えていただけないでしょうか?
文科系の人間なので、イマイチピントきません。

793 名前:132人目の素数さん投稿日:03/01/20 22:56
>>792
788じゃないけど、

T(T)が停止すると仮定する。 (1)
Hの定義より、H(T,T)=1。
Tの定義より、H(T,T)=1のときT(T)は無限ループ。(2)
(1)と(2)は矛盾。

T(T)は無限ループと仮定する。 (3)
Hの定義より、H(T,T)=0。
Tの定義より、H(T,T)=0のときT(T)は停止。(4)
(3)と(4)は矛盾。

という事は、Hは存在しない(Hの存在を仮定したのが矛盾)。


794 名前:778投稿日:03/01/20 23:06
ありがとうございました。
よく考えて見ます

795 名前:132人目の素数さん投稿日:03/01/20 23:11
>>790
H(z,z)のzってプログラムコードと引数がごっちゃになってない?

796 名前:132人目の素数さん投稿日:03/01/20 23:24
【問題】
次のようなゲームの最適解をもとめよ。
お互いの間で任意の自然数を紙に書いて出し合う。
原則小さい数字を書いたほうに1点が与えられるが、1小さい場合は
大きい数字を書いたほうに2点が与えられる。同じ数字の場合は引き分けと
する。
このゲームに最善のを尽くすためにはどのような確率で各自然数を
紙に書いてだすべきか?

797 名前:132人目の素数さん投稿日:03/01/21 22:23
「この文章には1がx個ある」「この文章には1がx個、2がy個ある」
「この文章には1がx個、2がy個、3がz個ある」…と文章(x等は自然数)
を作って行くとき始めに矛盾のない文章が成立するのはどの時で、
どんな文章になるか答えよ


798 名前:132人目の素数さん投稿日:03/01/21 22:32
>>797
成立しない


799 名前:797投稿日:03/01/21 22:42
6,7あたりで成立する文章を作ることができる。
昔、PCで計算させたことがある。

800 名前:132人目の素数さん投稿日:03/01/21 22:51
「この文章には1が4個、
2が3個、3が2個、
4が2個、5が1個、
6が1個、7が1個ある」
A.7のとき

801 名前:132人目の素数さん投稿日:03/01/21 22:55
8以上の場合はどうなの?


802 名前:132人目の素数さん投稿日:03/01/21 22:55
n番目で成立するとしたときに、nに
Maxが存在するか、というのはどうだろう。

803 名前:132人目の素数さん投稿日:03/01/21 23:01
というか、2ケタ以上の数はどうやってカウントするんだ?
「202が452009個」という文章には「20」が2つあると数えていいんかいな
そのときに一度カウントした「20」の中に「2」をカウントしていいのか、とか。


804 名前:132人目の素数さん投稿日:03/01/21 23:11
>>803
それはたぶんダメっしょ
美しくないもん

805 名前:132人目の素数さん投稿日:03/01/21 23:12
>>796
何をもって最適とするのか?
題意がいまいちつかめん。

806 名前:132人目の素数さん投稿日:03/01/21 23:15
>>796
2が三分の一、残りは1

807 名前:132人目の素数さん投稿日:03/01/21 23:28
>>806
相手が2しか出さなかったら負けるじゃん
それで最適って言っていいの?

808 名前:132人目の素数さん投稿日:03/01/21 23:44
>>807
最適と無敵は違うぞ。
806が正しいかどうかは知らんが。

809 名前:132人目の素数さん投稿日:03/01/21 23:52
>>808
だから結局相手の戦略によって全然違ってくるんじゃないのってことよ
何をもって最適と言うのさ?

810 名前:132人目の素数さん投稿日:03/01/21 23:57
一つの案としては相手がどんな戦略で来ても常に勝率5割以上をキープできる
出し方があるならそれを最適といってもいい気がするけど、
そんな出し方はなさそうだし、やっぱり題意がわからん。

811 名前:132人目の素数さん投稿日:03/01/21 23:57
コインを投げて、表が出たら+1点、裏が出たらー1点というゲームをする。
ただしゲームは0点から始まり、0より少ない点数もあるとする。
n回コインを投げたときまでに、一度は100点を超える確立をP(n)とするとき、
n→∞でP(n) = 1 となることを証明せよ。

812 名前:132人目の素数さん投稿日:03/01/22 00:10
別に「100点」でなくても任意の点でいいっしょ

813 名前:132人目の素数さん投稿日:03/01/22 00:16
あー、前に見たことあるけど結局解けなかった気が…
誰か証明して
>>811

814 名前:793投稿日:03/01/22 00:21
>>795
プログラムコードとはプログラムを引数として与えられるように
コード化してある、という意味と思われ。
例えば任意のプログラム(ならびにその引数)を、1と0からなる
順列にコード化するような単射が存在する。

だけど、それでも>>790のH(P,x)の定義は問題がない訳ではない。
厳密には、

I(P):= Pがプログラムコードなら1を、それ以外なら0を出力して停止
E(P,z):= I(P)=1ならばP(z)、I(P)=0なら無限ループ
H(P,z):= E(P,z)が停止するなら1を、無限ループなら0を出力して停止

という3つのプログラムのうち、I(P)とE(P,z)が存在することを
証明しておいてH(P,z)の存在を仮定すれば、以下>>790の証明に繋がる。
もちろん、I(P)とE(P,z)の存在は証明されているよ。


815 名前:132人目の素数さん投稿日:03/01/22 00:22
ここに1、2、3〜10gの分銅が1個ずつと
1gの金貨が何枚か入っている袋がある
袋の重さは無視でき、金貨は最高で10枚だとすると
天秤を使って金貨の枚数を特定するには最低何種類の分銅を使う必要があるか。
もちろん袋の中身を取り出すことはできない。



816 名前:132人目の素数さん投稿日:03/01/22 00:23
ランダムウォーク問題だったっけ。
図解雑学「確率」かなんかに幅広くかつ薄く書いてあった。

817 名前:132人目の素数さん投稿日:03/01/22 00:30
>>815
>>815
完全情報ゲームの一種。
答えは3種類かな。分銅(金貨軽い,同じ,金貨重い)
5(3(1(-,1,2),3,4),5,5+3(5+1(-,6,7),8,5+3+1(-,9,10)))

818 名前:132人目の素数さん投稿日:03/01/22 01:37
>>796の問題、常に勝率5割以上をキープする戦略なんて無さそうって
気軽に言っちゃったけどそうでもないな。もしかしたらありそうだ。

あるという前提のもとで計算してみたら

0≦P(1)≦1/3
2/9≦P(2)≦1/3
2/9≦P(3)≦1/3

まではしぼれた。

819 名前:132人目の素数さん投稿日:03/01/22 02:01
>>796
[+1,+1,+1,+1,+1,…]
[+0,-2,+1,+1,+1,…]
[+2,+0,-2,+1,+1,…]
[-1,+2,+0,-2,+1,…]
...
×
[P(1),P(2),…]

[1,0,0,0…]
この無限×無限行列を解けばいいんだね…

820 名前:132人目の素数さん投稿日:03/01/22 02:04
>>811
http://www.sigmath.es.osaka-u.ac.jp/~m-date/java/srw/random2.html
でx=100で試した結果…
20%くらいじゃないの?(マジで

821 名前:132人目の素数さん投稿日:03/01/22 03:11
一個見つけたぞ!

P(1)=1/16
P(2)=5/16
P(3)=4/16
P(4)=5/16
P(5)=1/16

これで常に勝率5割以上をキープできるはず。

822 名前:132人目の素数さん投稿日:03/01/22 03:15
感想。
さっき友達とこのゲームをやった時はお互い1〜3しか出さなかったのに
4の確率が5/16と高いことに驚いた。

823 名前:132人目の素数さん投稿日:03/01/22 06:14
勝率5割っていう言い方がよく分からんのだが。
この場合勝ちっていうのは十分な回数やったあとで相手よりポイントが高いってこと?


824 名前:132人目の素数さん投稿日:03/01/22 13:39
>>823
ま、そういうことだね。
要するに相手がどんな数字を言ってきても獲得点数の期待値が0以上になってるってこと。
点数制になってる以上、一回一回の勝ち負けの確率を論じても仕方ないっしょ。

825 名前:132人目の素数さん投稿日:03/01/22 13:57
ちなみに相手が1〜4までしか出さないような素人の場合、
相手が一回以上1を出してくれれば

P(1)=0
P(2)=2/5
P(3)=1/5
P(4)=2/5

で勝率5割を「越える」。

826 名前:132人目の素数さん投稿日:03/01/22 14:13
10^10匹の猿が宇宙の年齢(10^18秒)の間、タイプライターに向かって座っていると過程する。
ちょうど、現在の人類の人口の3倍だ。また、一匹の猿がタイプできるのは一秒につき10キーだと過程する。
1つのタイプライターにあるキーは44個だ。大文字を打つところを小文字で打ってもよしとする(文頭、人名など)
シェークスピアのハムレットが10^5文字で構成されているとする。

宇宙が終わるまでに猿がハムレットをタイプ仕上げる事は出来るだろうか?

827 名前:132人目の素数さん投稿日:03/01/22 15:15
『去年まで金無し君だったのに、二年で350万貯めた。』
上記命題の真偽について考察せよ。

828 名前:132人目の素数さん投稿日:03/01/22 15:15
「去年」まで金無し君だったのに、「二年」で350万貯めたとはどういうことなんでしょう?

───────┼─────── 2年 ───────┼
 
━━━━━━━●
  ↑金無し君        ↓貯蓄中
              ○━━━━━━━━━━━━━━━━━★
                                350万貯めた↑

350万貯めるまで2年かかったと命題にあるので、
金無し君の期間と、350万貯める期間が重複しないと仮定すると、
金無し君の期間がもっとも最近まであったとしても
2年前となってしまうのですが。

829 名前:132人目の素数さん投稿日:03/01/22 15:15
あ、もしかすると「金無し君=貯金0」という定義が間違っているのかもしれません。

───────┼─── 1年.───┼─── 1年.───┼
 
━━━━━━━━━━━━━━━━●
  ↑金無し君        ↓貯蓄中
              ○━━━━━━━━━━━━━━━━━★
                                350万貯めた↑

時間毎の貯金量が一定だと仮定すると、1年間で175万たまるわけですが、
「貯金175万以下は金無し君」と定義すれば、
「去年まで金無し君」と「二年で350万貯めた」を両立させることができます。
つまり、作者にとっては貯金175万以下は金無し君ということなんでしょうか?

830 名前:132人目の素数さん投稿日:03/01/22 15:36
去年(の始め)まで金無し君ということではないか?
今年の年末の発言ならばあり得るかもしれない。

831 名前:132人目の素数さん投稿日:03/01/22 16:21
>>826
(1/44)^(10^5)*((10^19)C(10^5))
・・・一匹の猿がテキトーに打った中に「ハムレット」が含まれる確率

(100/44)^(10^5)*((10^19)C(10^5))・・・10^10匹の猿中に
「ハムレット」を中に含む文章を打った猿の頭数の期待値

↑よって明らかに、「1」を遙かに超えるので、ほぼ確実に1頭は居る。


832 名前:132人目の素数さん投稿日:03/01/22 16:26
テキトーに打っても数打ちゃハムレットが書けるのか。
俺もがんばろう

833 名前:132人目の素数さん投稿日:03/01/22 16:27
猿がテキトーにtex文打ってフェルマー予想の証明が出来る確率は?

834 名前:831投稿日:03/01/22 16:59
訂正。
大嘘だった。もうちと考えなおします。


835 名前:132人目の素数さん投稿日:03/01/22 17:10
偶然ハムレットをタイプする確立
(1/44)^100000

猿による総タイプ数
10^10*10^18*10=10^29

明らかに無理。

836 名前:831投稿日:03/01/22 17:17
一匹の猿が打てる確率。
(1/44)^(10^5)*(10^19-10^5+1)>10^(-99980)
猿10^10匹中の期待値
10^(-99970)から、ほぼ0なので無理!


837 名前:132人目の素数さん投稿日:03/01/22 17:19
不等号反対だ。スマソ

838 名前:132人目の素数さん投稿日:03/01/22 17:22
>>836 (・∀・)カエッテイイヨ!

839 名前:132人目の素数さん投稿日:03/01/22 17:30
>>796
どの相手にも平均で負けないために
満たさなくてはならない不等式を係数だけ書くと
(+0,+2,−1,−1,−1,−1,−1,...)≧0。
(−2,+0,+2,−1,−1,−1,−1,...)≧0。
(+1,−2,+0,+2,−1,−1,−1,...)≧0。
(+1,+1,−2,+0,+2,−1,−1,...)≧0。
(+1,+1,+1,−2,+0,+2,−1,...)≧0。
それぞれに1,5,4,5,1をかけて足すと
(+0,+0,+0,+0,+0,−13,−16,...)≧0。
となるので6以上を出す確率は0。
残りは>>821となる。


840 名前:132人目の素数さん投稿日:03/01/22 19:21
そんな事に猿は固執しない。=>無理

841 名前:826投稿日:03/01/22 21:21
答えを言うタイミングとかあるんですか、このスレ?

842 名前:826投稿日:03/01/22 21:26
ちなみに>>835がぼくの持ってる答えに近いです。
10^18秒の間に1冊のハムレットが仕上がる確立は?


843 名前:796の出題者投稿日:03/01/22 21:48
>>839
正解だと思います。
20年以上前の日経サイエンスに載っていた問題でした。

844 名前:132人目の素数さん投稿日:03/01/22 22:01
>>826
そんなにたくさんの猿にたくさんのタイプライターを与えなくとも、
PCで円周率を計算しその値を文字コードとみなせば、その中には
ハムレットもあれば、100年後に発表される世紀の大論文も
含まれることになるよ
円周率解読競争が始まるかもしれない

845 名前:132人目の素数さん投稿日:03/01/22 22:36
ありがちな問題を一ひねりしてみました。

3種類の薬ビンが6個あり1000錠づつ薬が入っている。
しかし外見は同で、区別できるのは錠剤の重さだけである。
錠剤の種類は一番軽いA錠剤、それに比べて10mg重いB錠剤、20mg重いC錠剤
の3種類である。
はかりで1回だけ計って、それぞれのビンの薬の種類を知るためには
どのようなはかり方をしたらよいでしょうか
念のため、一つのビンには1種類の錠剤しか入ってません。

846 名前:132人目の素数さん投稿日:03/01/22 23:16
>>826
「熱物理学」キッテル、丸善
だよね。良い本だった。
この頃読んだ本であれくらいの質の本は、
「論理学をつくる」戸田山 和久、名古屋大学出版会
くらいかなあ。

847 名前:132人目の素数さん投稿日:03/01/22 23:36
数学ってどうしてこんなにおもしろいんですか?

848 名前:132人目の素数さん投稿日:03/01/22 23:59
πとeは簡単な演算で整数に近い値になる事が多い。
π^3=31.00627… や e^3=20.08553… は有名だ。    

問題 □の中に演算子 +-*/^ のどれかを入れて式を完成させよ。
   優先順位は +- < */ < ^ とする。(^を最初に計算)

e □π□π = 9.00…
e □π□π =19.999…

e □π□π□e =29.000…
e □e □π□e =13.99…

π□π□π□e =14.00…
π□π□π□e =45.00…
π□e □π□e =30.99…

π□π□π□e□e□e□e=23.00000…

849 名前:132人目の素数さん投稿日:03/01/23 03:02
>>839
任意の自然数なのに6まででいいんだ!
このスレきっての感動でした

850 名前:132人目の素数さん投稿日:03/01/23 12:42
「わからない問題」のスレに載っけたけど答えてくれないのでこっち。

問題
「12本の金のノベ棒があります。
そのうち1本は他のより重いか軽いです。
天秤を用いて(使用回数は3回以内)、その1本を当ててください。」

この問題全然わからなくて・・・
誰か教えてください。


851 名前:132人目の素数さん投稿日:03/01/23 13:02
>>845
五分ほど考えてみたけどさっぱり分からず。
良問と見た!

852 名前:132人目の素数さん投稿日:03/01/23 13:03
>>845
ん? あれ? なあなあ、はかりってどんなものなの?

853 名前:132人目の素数さん投稿日:03/01/23 13:23
>850
12個の延べ棒を3つずつの4セットA,B,C,Dに分けて
天秤にAとBをかけて、等しかったらAとCを入れ替える。

ここまでくれば分かるでしょ

854 名前:132人目の素数さん投稿日:03/01/23 13:29
>>853
>天秤にAとBをかけて、等しかったらAとCを入れ替える。

そのやりかたで、3回でできるか?
もしAとCを入れ替えて、CとBを天秤にかけたとして、等しかったら?
Dの中にあることが分かるだけで、あと1回では判定できないよ。

855 名前:132人目の素数さん投稿日:03/01/23 13:36
>>845
溶けた。

それぞれの瓶から、一錠、二錠、四錠取って、量る。

a,b,cをそれぞれA,B,Cから取った錠剤の数とすると、(a,b,c) = σ(1,2,4)。
錠剤Aの重さをxとすると、全体の重さは (1+2+4)x + 10b + 20c [mg]。
これを7を法としてみると3b+6cとなるが、これは全てのσで異なった値をとる。

856 名前:132人目の素数さん投稿日:03/01/23 13:36
>>850
http://cheese.2ch.net/math/kako/967/967889985.html
このスレ見れ。

しっかし激しく既出スレ、問題の答えが書いてないんじゃ何の意味もねー

857 名前:132人目の素数さん投稿日:03/01/23 13:40
Dの中にあるなら、Dの中の延べ棒3本(D1,D2,D3)の
D1とD2を天秤にかければいいだけじゃ…

858 名前:132人目の素数さん投稿日:03/01/23 13:49
>>857

>>850
>そのうち1本は他のより重いか軽いです。


859 名前:132人目の素数さん投稿日:03/01/23 15:08
>>855
瓶は6個あるよ?

860 名前:132人目の素数さん投稿日:03/01/23 17:48
>>850
いつぞかのセンター試験(英語)ででてきたやつににてますな。

12本を半分にわける。

6本を半分にわける。

2つの重さを比べる。

これが正解の一部。



861 名前:132人目の素数さん投稿日:03/01/23 18:46
>860
1本他より重いか軽いかわからんのだよ?
それ3回で出来る?

862 名前:不備がったので再掲します投稿日:03/01/23 21:13
>>845
3種類の薬ビンが6個あり1000錠づつ薬が入っている。
しかし外見は同で、区別できるのは錠剤の重さだけである。
錠剤の種類は一番軽いA錠剤、それに比べて10mg重いB錠剤、20mg重いC錠剤
の3種類である。
はかりで1回だけ計って、それぞれのビンの薬の種類を知るためには
どのようなはかり方をしたらよいでしょうか
念のため、一つのビンには1種類の錠剤しか入ってません。
以下追加
はかりは目盛りで表示するはかりで、天秤ではありません。
一番軽い錠剤の重さはわかっているとします。


863 名前:132人目の素数さん投稿日:03/01/23 21:18
これから二人でカードを9枚ずつ使って対戦ゲームをする。
9枚の内訳は、6枚が兵士で残りは姫・王様・闇の男とする。
強さは、闇の男<兵士<姫<王様だが例外として闇の男は王様に勝てるものとする。

二人で同時に一枚のカードを選んで出し、強いカードを出した方がカードを手に入れられる。
同じ強さのカードを出した場合は、次に保留とする。
これを9回繰り返し、カードが多い方を勝ちとする。
このとき、最初は何を出すべきか。また、次は何を出すべきか。



864 名前:132人目の素数さん投稿日:03/01/23 21:35
二次方程式y=x2乗と一次関数y=x+2のグラフがあり、
交点をA、Bとする。
今、y=x2乗上に点Pを置く。(但し、原点を除く)
△AOBと△APBの面積が等しくなる点Pの座標を求めたら神。
(1、1)以外で、x座標が?/?√?になるやつ。
面白くないが


865 名前:132人目の素数さん投稿日:03/01/23 21:45
>>863
http://science.2ch.net/test/read.cgi/math/1043108320/450

866 名前:132人目の素数さん投稿日:03/01/23 22:09
あるヴァカな友達が言いました。
「俺、チョキを2回に1回だすとスゲー勝てるんだよな。」
・・・まあ、こいつは「最初はグー」の次にパーを出しやすいと
読んでるのだろうが。

さて、コイツとじゃんけんをするときの最適な出し方をもとめなさい。
また、その手段で720万回繰り返したとき約何回多く勝つと考えられるか?
有効数字3桁で求めよ。
(ただし、コイツは宣言したことを忠実に守るバカ正直な奴だと仮定する。)


867 名前:132人目の素数さん投稿日:03/01/23 22:46
>>862
3進法を使う!

6個の瓶からそれぞれ1錠・3錠・9錠・27錠・81錠・243錠・729錠ずつ取り出し、それらが混ざらないように秤にかける。

総重量の値から、一番軽い錠剤x1093の値を引く。

その結果をさらに10mgで割る。

あとは3で割って商と余りを出し、その商を3で割り、・・・・・割れなくなるまで続ける。
このとき、余りは左から順に書き出していく。

その余りを左から0=A,1=B,2=Cと置き換えれば、1番目の瓶から6番目の瓶まで
全ての瓶の錠剤が分かる。

855はちょっと惜しい!
aかbならこの方法で1,2,4,8,・・・とすればわかるが
3種類だとうまくいかない。

868 名前:132人目の素数さん投稿日:03/01/23 22:47
>>862
>3種類の薬ビンが6個あり

ある種類は0個であっても構わないということですか?

869 名前:862出題者投稿日:03/01/23 22:56
>>867
そのとおりです。
錠剤が2種類の場合は本や平成教育委員会でもやってました。
ただし、2進数と10進数の関係については触れていませんでした。
この問題は3進数と10進数の関係に問題を拡張したものです。


870 名前:868投稿日:03/01/23 23:00
レス遅かった…

>>848
とりあえず上3つだけ。

e+π+π = 9.001467136…
e ^π-π = 19.99909998…

e ^π+π+e = 29.00056711…

871 名前:お見合い問題投稿日:03/01/23 23:05
お見合いマニアのおばさんから写真が毎日1枚送られてくる。
おばさんによれば、これから毎日1枚づつ100枚送ってくるそうだ。
私は当日中に会うか否かの返事を電話でする必要がある。
私はもちろん100枚の中からできるだけよい相手を選びたいが、
上記のルールの下で私はどのような作戦をたてるのが最もよいだろうか?

872 名前:お見合い問題投稿日:03/01/23 23:08
>>871
追伸
一度選んだら、それ以降の相手を選ぶことはできない。

873 名前:132人目の素数さん投稿日:03/01/23 23:09
>>871
確率論の講義で似たようなのやったなぁ。
逆数の和を使うんだっけ?

874 名前:873投稿日:03/01/23 23:40
>>871
最適停止問題の一つ、最良選択の問題。

全部で100人である。(=N)
また、
1/38 + 1/39 + .... + 1/(N-2) + 1/(N-1) < 1 ≦ 1/37 + 1/38 + 1/39 +....+ 1/(N-2) + 1/(N-1)
の不等式が成り立つ。
よってターニングポイントが37となる。

36人目までは様子を見る。情報集め。
37人目以降は、1人目からそれまでで最もよい相手を選ぶ。
例えば、もし37人目が1〜36人目の誰よりも(・∀・)イイ!!相手だったら
彼女を選ぶ。そうでなければ37人目はパスする。
38人目も同じ。1〜37人目の誰よりも(・∀・)イイ!!相手だったら
彼女を選ぶ。そうでなければ38人目はパスする。
以下繰り返し。(もちろん、もし100人目になったらその100人目の彼女を選ぶ。)

これは計算高い人なら人生の伴侶選び(w にも応用できそうだが、
現実にはNの値が分からない罠。

875 名前:132人目の素数さん投稿日:03/01/23 23:44
>>870
出た正解の範囲から導ける式
e^π-π≒20≒e^3 → e^π≒e^3+π

876 名前:873投稿日:03/01/23 23:57
あ、ターニングポイントが一つずれてるかも…

877 名前:132人目の素数さん投稿日:03/01/24 00:26
>私は当日中に会うか否かの返事を電話でする必要がある。
会うか否かを決定して
>よってターニングポイントが37となる。
37まで選ぶの待てるの?
>一度選んだら、それ以降の相手を選ぶことはできない。
否定しつづけて、やっぱこれ!でいいの?どういうこと?

最適停止問題知らないので解説希望します。 嫌ならいいですけど。

878 名前:132人目の素数さん投稿日:03/01/24 00:58
>>873
一応貴方の考えに基づいて、平均して何番目の美人をgetするかについて
期待値の計算式をつくってみました。

36*5049/9900+Σ[k=36〜99]{R(k)*36*(100-k)*(101-k)/(200-2k)}番目
(ただし、R(k)={(k-1)!64!}/{100!(k-36)!})


879 名前:873投稿日:03/01/24 01:12
>>877
>>872の「一度選んだら」というのは、会う相手を一度選んだら、
ということだと理解しました。
つまり、36人目までは写真だけを見る。会うのは断る。
37人目以降は、>>874の戦略で会うかどうかを決めるということ。
会った後のことについては考えてないと思う。問題文から推察すると。

>>874は、
一人一人が「良い相手度」でスコア付けでき、
相手の100人がそれぞれ 1点,2点,3点,…,99点,100点のスコアを一つづつ付けているものとしたときの、
期待値を最大にする戦略を求めた結果です。(確か)
>>874でカットした議論は結構長いです。
とりあえずキーワードだけ…
確率空間、σ加法族、stopping time、(super)martingale、filtration、F_n-adaptedなど…

もしスコアの分布が違っていたりすれば戦略も変わるかと。
スコアの分布が未知、だともっと難しくなりそうです。
秋山仁あたりが本を書いてるらしい。(自分はそれ読んでません。)
最適停止問題ってのはこの種の問題の総称で、色々研究されていると思います。

戦略を限定した場合の議論はこのへんが参考になるかも。
ttp://web2.incl.ne.jp/yaoki/omiai.htm

880 名前:877投稿日:03/01/24 01:19
かなり複雑なようですね、色々解説ありがとうございます。
勉強してみます。

881 名前:132人目の素数さん投稿日:03/01/24 17:15
2003人で将棋の総当たり戦をする。
このとき、最低何勝すれば単独優勝できるか。
ただし、引き分けはないものとする。

882 名前:132人目の素数さん投稿日:03/01/24 18:59
1000

883 名前:132人目の素数さん投稿日:03/01/24 19:05
>>882
早過ぎるよ〜w

884 名前:132人目の素数さん投稿日:03/01/24 19:56
2003人で将棋の総当たり戦をする。
このとき、最低何勝すれば単独優勝できるか。
ただし、引き分けもあるものとする。


885 名前:132人目の素数さん投稿日:03/01/24 19:58
>>884
881の出題者だけど、1に決まってんじゃン。
とマジレス。

886 名前:132人目の素数さん投稿日:03/01/24 20:00
アハハバレタ
2003人で将棋の総当たり戦をする。
勝ちは勝ち点3 引き分けは勝ち点1とする
このとき、単独優勝できる勝ち点は何点だろうか?




887 名前:132人目の素数さん投稿日:03/01/24 20:03
で、本題
2003人で将棋の総当たり戦をする。
勝ちは勝ち点3 引き分けは勝ち点1とする
このとき、10位までに入賞できる勝ち点は最低何点だろうか?


888 名前:132人目の素数さん投稿日:03/01/24 22:13
e^π と π^e の大小を比較せよ。但し、e、πの近似値を用いてはならない。(30点)

889 名前:132人目の素数さん投稿日:03/01/24 22:17
ごめん、将棋って漏れ詳しくないんだけど、
どうなったら引き分けっていうの?

890 名前:132人目の素数さん投稿日:03/01/24 22:18
千日手など

891 名前:132人目の素数さん投稿日:03/01/24 22:19
>>890
詳しく

892 名前:132人目の素数さん投稿日:03/01/24 23:40
>>890
千日手は打ち直しになるだけで(同じ千日手を繰り返すような手は負けになる)引き分けにならない。

確か王・玉が互いに敵陣に入ったときに対局をうち切って歩以外のコマを点数に換算し、
両方とも規定の点数以上でなかったら(点差が付いていても関係ない)「引き分け」になったかと。



893 名前:お見合い問題投稿日:03/01/24 23:58
>>871
出題した者としてコメントします。
おとといの相手がよかったと言うことはできません、
当日の相手を当日に選ぶか否か判断し、選んだら、明日以降の
写真から選ぶことはできない、と言う条件です。

私はこの問題の回答も正確には知らないし、解法も理解できません。
回答の粗筋としては、最初の何名かはやり過ごしランク付けをする、
次の何人かは歴代で1番ならそれを選択する、それで終わらない場合は、
また何人かについて歴代で2位以上ならそれを選択する、それで終わらない場合は
また何人かについて歴代で3位以上ならそれを選択する…と言ったような
回答です。
始めは理想を高く掲げ、後になるほど現実と妥協せよ、と言うわけです。
若いうちは理想を抱き、年長になると妥協して生きるという、人生
に通じる問題だと思います。

894 名前:132人目の素数さん投稿日:03/01/25 02:56
>>888
両辺対数とって比べりゃいいんじゃないすか?

895 名前:132人目の素数さん投稿日:03/01/25 12:56
log(π)?

896 名前:132人目の素数さん投稿日:03/01/25 14:43
f(x) = (log x)/x 考えると f(pi) と f(e) の大小が分かる
# さすがに 0 < e < pi は使っていいんだよね

897 名前:888投稿日:03/01/25 18:35
>>894
惜しい
>>896
正解!

898 名前:132人目の素数さん投稿日:03/01/25 19:48
>>888,>>896
それは漏れも考えたが、e^eとpi^piの大小は分かるが
e^piとpi^eの大小関係は分からないと思うのだが・・・

そこで、f(x)=xとg(x)=elogxのグラフを考え、f(x)とg(x)は点(e,e)で接し、
x=e以外の点では常にf(x)>g(x)なのでx=πのとき、f(π)>g(π)であり、
指数の形になおしてe^π>π^eというふうに解いた方が確実じゃないのかな?


899 名前:898投稿日:03/01/25 19:49

ゴメソ、訂正。アホだ俺。
移行して十分わかるね。


900 名前:132人目の素数さん投稿日:03/01/25 20:52
正N角形について、重心から各頂点までの長さをR、周の長さをLとする。
Lと2πRが10ケタ目まで一致するNの最小値を求めよ。
但しπの近似値を用いてはならない。(80点)

901 名前:873投稿日:03/01/25 20:57
>>893
分布が最初はわかってないか、あるいは与えられた情報から
分布予測を修正していくってことなのかな。

902 名前:132人目の素数さん投稿日:03/01/26 10:18
>>874
現実問題として、伴侶に選んだ以上、お互いの足りない部分をお互いに助け合う、
だからお互いに相手を受け入れれば、必ずしも最適な組み合わせである必要がない。
そこが愛の素晴らしいところなんだよね。
逆に、相手に足りないところがない完璧人間だと、かえって愛は覚めるぜ。
相手にとって自分は不必要な人間なのかなあって感じるようになるからさ。
特に理系の人にはわかってもらいたい。オレもバリバリの理系なんだけどさ。

>>893
数学問題としても人生問題としても全くその通りだよね。

903 名前:132人目の素数さん投稿日:03/01/26 18:16
純粋な数学問題とは言えないかもしれませんが…
1億円の当たる300円の宝くじ(期待値は50%以下)を買う人は大勢いるが、
金額を1万倍して、1兆円の当たる300万円の宝くじを買う人はめったにいない
と思われるが、その理由を説明してください。できれば、数学的モデルを
使って代数的に証明してください。

904 名前:132人目の素数さん投稿日:03/01/26 19:05
>>903
代数的に両者の当選確率や期待値等は全て等しい。
しかし、大半の人間は「失敗した時のリスクを考える者」である。
300円の宝くじを10口買って、全て外れても3000円の損害で済むが、
300万円の宝くじを10口買って、全て外れてしまうと3000万円もの損害になってしまう。
そもそも宝くじというものは「当てにいくもの」ではなく「当たればいいな」と思って買うものである。
そんな眺望的な宝くじに誰が1口300万円もの金を払うというのか。
>>903の命題は証明された。

905 名前:132人目の素数さん投稿日:03/01/26 19:41
1兆円の使い道って?

906 名前:132人目の素数さん投稿日:03/01/26 20:10
>>904
>大半の人間は「失敗した時のリスクを考える者」である。
この部分を数学的に表現できなければ証明されたとは言えないのではないか?


907 名前:132人目の素数さん投稿日:03/01/26 20:20
毎日1億円で約30年

908 名前:132人目の素数さん投稿日:03/01/26 22:27
なんか経済関連でもらえる金額と満足度は比例しないって聞いたことある。
ちょうどlogのグラフみたいになってて、
一兆円もらうことは一億円もらうことよりも一万倍嬉しい、とはならないらしい。

これ、少ない金額だと逆になるのが面白いとこだな。
一万円もらうのは一円もらうのより限りなく嬉しいもん。

たぶんそれを最適化したのが宝くじなのだろう。

909 名前:132人目の素数さん投稿日:03/01/26 22:44
1000兆円のあたる300億円の宝くじを買う人間は地球にいないだろ?
人間には単なる数字じゃないリアルな金額ってもんがあるんだよ。


910 名前:132人目の素数さん投稿日:03/01/26 22:54
3億円あたって3000万円分また宝くじ買った人はいる罠

911 名前:132人目の素数さん投稿日:03/01/26 23:01
リスクは「所得に対する掛け金の割合」として
考えられない?

どっかの国のように身分階級がはっきりしていて所得格差が
日本とまるっきり違う場合はどうよ?
金持ちは果てしなく金持ちで、貧乏人は果てしなく貧乏人。
そんな所得の分布が日本とは全く異なる社会において、
果たして
>>903の「300万円の宝くじを買う人はめったにいない 」
という話が正しいと思える?俺は思えない。
むしろ逆に300円の宝くじは人気が出ないと思う。

俺は所得分布を調べれば説明できると信じる。

912 名前:132人目の素数さん投稿日:03/01/27 00:00
次スレ立てるときは、次の兄弟スレのリンクもお願いしますだ

問題に答えた人が次の問題を出すスレ
http://science.2ch.net/test/read.cgi/math/1037801891/l50

ほかにもあるけど、どうします?

◆◆◆超難解問題の解答を教えてください◆◆◆
http://science.2ch.net/test/read.cgi/math/1041774035/l50
シンプルで難しい問題
http://science.2ch.net/test/read.cgi/math/1011067036/l50
Qマソが問題を出すスレ
http://science.2ch.net/test/read.cgi/math/1040710744/l50

913 名前:132人目の素数さん投稿日:03/01/27 09:04
「超難解〜」は単発質問スレやん。
だったら
★東大入試作問者になったつもりのスレ★
http://science.2ch.net/test/read.cgi/math/1000592003/
天才中学生の俺を試して下さい。
http://science.2ch.net/test/read.cgi/math/977469232/
の次スレ(こっちの方が先に立つだろうて)をリンクに入れた方が。

914 名前:132人目の素数さん投稿日:03/01/27 09:08
「シンプルで〜」は
もし解いたら数学史に名が残る問題
http://science.2ch.net/test/read.cgi/math/1016031138/
に近い気がするけど、まぁそこら辺はスレ立てる人の自由やな。

915 名前:132人目の素数さん投稿日:03/01/27 09:45
★小学生向け問題募集★
http://science.2ch.net/test/read.cgi/math/1020248263/l50
☆2ちゃんねらーず編・大学入試数学問題集☆
http://science.2ch.net/test/read.cgi/math/988730706/l50
ってスレもあるでよ。

916 名前:132人目の素数さん投稿日:03/01/27 16:13
ここは「その問題面白いっ!」ってレスつくようなスレになればいいな
で、みんなのおもしろかった問題ってどんなの?
おれは>>123>>124>>156>>174>>232>>339>>400>>506>>532>>709>>826

917 名前:132人目の素数さん投稿日:03/01/27 19:57
うーん、いずれもおもしろいね

918 名前:24の男投稿日:03/01/27 20:19
先ほど、違う板に書いたのですが、面白い?命題を昔ひらめいたので、書かせてもらいます。
それは、
    「ルールがあるなら世界ができる。」
です。
たとえば、野球には野球のルールがあるから、野球の世界ができて、
歌い手が歌うことをルールとするなら、歌い手の世界ができる。
などです。


919 名前:132人目の素数さん投稿日:03/01/27 20:31
>>916
826は大した問題では無かったと思うが…
124、232は漏れも推したい。意外と888も好き

920 名前:投稿日:03/01/27 20:34
誰か>>900を解いてくれ…
糸口すら見つからん。

921 名前:132人目の素数さん投稿日:03/01/27 22:11
>>903
手にいれた現金の額に対応する満足度を人は決定できるとする。それを以下のようにあらわす。
a=U(1万円)1万円の満足度はaである。
期待値を計算する関数を以下のように決める。
b=E(−300、1億−300)300円を支出して、一億円が当たる期待値、限りなく−300に近いはず。
宝くじを買わない場合の満足度はx=U(0)と表せる。 普通人ならx=ゼロ?
宝くじを買った場合の満足度の期待値は以下のようになる
y=E(U(−300)、U(1億−300))300円を支出して失う満足度と1億円があたる場合の満足度の期待値、限りなくU(−300)に近いはず。
x<yならそのような賭けは合理的に実行され、
x>yのときそのような賭けは実行しないのが合理的

満足度関数の形状は個人の価値観や経済状態に依存するから、確定的なことは言えないが
この問題の想定では一般的な人々の満足度関数Uの形状は300円の支出で1億円当選する宝くじに関してはx<yの関係をもつが、
300万円の支出で1兆円当選する宝くじに関してはx>yの関係を持つため、そのような宝くじを買うことはない。あるいは、
そのような宝くじを設計し販売することはない。



922 名前:132人目の素数さん投稿日:03/01/28 00:41
やだ



923 名前:132人目の素数さん投稿日:03/01/28 03:26
>>920
近似値を密かに用いて最小値minを出し、
min-1で偽、minで真を示す

924 名前:(^^)投稿日:03/01/29 20:51
(^^)

925 名前:age投稿日:03/01/29 20:51
age

926 名前:132人目の素数さん投稿日:03/01/29 21:02
某スレで書かれてたやつ。面白いかどうかは分からないが…

426 :問題 :03/01/27 18:36
半径1の球面があります。
この球面上に1点をとり、それを中心とした半径r(0<r<1)の球を考えます。
すると、この球によって、半径1の球面の一部が覆われることになります。
さて、半径rの球を最低何個用意すれば半径1の球面を完全に覆うことができるでしょう?
ただし、半径rの球の中心はつねに半径1の球面上に取るものとします

927 名前:yufi投稿日:03/01/29 21:51
昔、本で読んだ問題 スレされていたらスンマソ
3人の兄弟がいて、その親が3人ともに白か黒かの帽子をかぶせました。
≪兄弟は自分の帽子の色が何色か知らない≫≪母親は3人に白の帽子をかぶせた≫
≪兄弟は自分の帽子は見えないが他の二人の帽子の色はわかる≫
だが兄弟のうちの一人が自分のの帽子の色を答えた どのようにしたか?


928 名前:132人目の素数さん投稿日:03/01/29 21:58
>>927
白か黒かの帽子に個数制限はないのかな?
黒は1つしかないとか。

929 名前:yufi投稿日:03/01/29 22:01
ないです 推理?

930 名前:132人目の素数さん投稿日:03/01/29 22:06
≪母親は3人に白の帽子をかぶせた≫ のなら、
とりあえず、兄弟は3人とも白の帽子をかぶってるよね?

で、さらに父親も帽子をかぶせたの?

931 名前:132人目の素数さん投稿日:03/01/29 22:08
なんだ、釣り師か…

         ∧∧
       /⌒ヽ>
       [    _] 真面目なスレで、釣りはやめろよな…
      三___|∪
       (/~∪
      三三
    三三
  三三

932 名前:132人目の素数さん投稿日:03/01/29 22:17
実況板でやれ。


933 名前:928投稿日:03/01/29 22:42
>>931-932


>>929
もし帽子の個数に全く制限が無いなら、残りの2人が被ってるものと
自分が被ってるものの間に関係が無いことになり、
推測のしようがないと思うんだけど。

934 名前:yufi投稿日:03/01/30 21:05
じゃあ答え発表しましょうか?
3人望んだら・・・

935 名前:132人目の素数さん投稿日:03/01/30 21:10
イラネ

936 名前:132人目の素数さん投稿日:03/01/30 21:12
>>934
シネヨ、ボケ!
母親が3人に白い帽子をかぶせたつってんだから、白い帽子なんだよ!

937 名前:132人目の素数さん投稿日:03/01/30 22:24
>>927
一番背の低い奴が自分の帽子の色を兄弟の瞳に写した。
白だったので一目瞭然。背の高い二人は反射角度が負なので写らない。

よって、一番低いやつがわかった。


938 名前:132人目の素数さん投稿日:03/01/31 06:44
なんの話や

939 名前:132人目の素数さん投稿日:03/01/31 17:18
927の問題、
≪母親は3人に白の帽子をかぶせた≫

≪3人全員黒帽子、という事は無い事を母親は3人に知らせた。≫
とすると、一人一人帽子の色が分かるか分からないかを言わせていけば
よくあるパターンによって必ず一人は分かる。

しかし個人的には相手の瞳をよぉくよぉく覗き込んで答える、ってのが
正解のような気がする、とレスしようとしたら937に既に答えられてるし。

940 名前:132人目の素数さん投稿日:03/01/31 21:57
中心角が2αの扇形の重心を求めよ

941 名前:132人目の素数さん投稿日:03/01/31 22:03
Ans.扇形の真ん中

942 名前:132人目の素数さん投稿日:03/01/31 22:13
自スレ
ttp://science.2ch.net/test/read.cgi/math/1026218280/l50

943 名前:132人目の素数さん投稿日:03/01/31 22:37
>>941
ワラタ

944 名前:132人目の素数さん投稿日:03/02/01 01:35
>>927  その問題は、いじわるクイズで無い限り解けない。

俺の知ってるのは、こんな感じ。
 
母親が3人の子に言いました。
 今からあなたたちに、わからないように白か黒の帽子をかぶせます。
次の条件のどちらかを満たしている人に、お菓子をあげます
1 ほかの2人が白い帽子をかぶっているのを見た者 
2 自分の帽子の色を当てた者
そして母親は、3人ともに黒い帽子をかぶせた。

しばらくして、最も賢い次男が自分の帽子が黒であることに気付いた。
次男は、どのように考えたか?

945 名前:132人目の素数さん投稿日:03/02/01 01:43
次男。自分が白だとする。すると、長南はこう考える。
「三男からみて他の二人が白」ではありえない(自明)
よって、自分(長男)の防止は黒田。
ところが、長男が黙っているところを見ると、
長男には自分の帽子の色が分からない。
よって、自分(次男)の帽子は黒なんだーーー

じゃね? 
>>994

946 名前:132人目の素数さん投稿日:03/02/01 01:48
>>944
次男は考えた。

もし自分が白い帽子を被っていたらどうなるか?
兄はおいらの白い帽子と弟の黒い帽子を見ていることになる。
その時兄はこう考えるだろう。
 もし俺が白い帽子を被っているなら、
 真っ先に末っ子が”お兄ちゃんたち白い帽子被ってる”というだろう。
 でも末っ子は何も言わないようだ。だから俺は黒い帽子を被っているんだ!
と。
しかし兄は何も言わない。ということはおいらは黒い帽子を被っているんだな。

947 名前:132人目の素数さん投稿日:03/02/01 01:50
>>945
なるほどっ! それだぁ!
(でも 2ch風の漢字変換は、そろそろ卒業しようよ)

948 名前:946投稿日:03/02/01 01:51
もろ被った

949 名前:132人目の素数さん投稿日:03/02/01 01:55
全員賢かったら、一斉に自分は黒だって言うのだろうか

950 名前:946投稿日:03/02/01 02:02
・「お菓子」なんて別にイラネと思ってる長男が、分かっててもわざと発言しない可能性
・末っ子が反抗期で、母親の言うことを聞かない可能性
とかは考えちゃダメかなやっぱり。もはや論理学の問題ではなくなるが。


951 名前:132人目の素数さん投稿日:03/02/01 04:52
今日のうちに新スレ移行しそうな予感がするので、
次スレのタイトルを考えてみました
>>1を見て知ったけど、現在part3だったんですね

面白い問題おしえて〜な ヽ(∵)メ(∵)メ(∵)メ(∵)ノ

ちょっと横長いか…

952 名前:132人目の素数さん投稿日:03/02/01 04:56
>>1に書くのは、とりあえず、これですかね

---------------------------------------------------------
面白い問題、教えてください

面白い問題おしえてーな
http://science.2ch.net/test/read.cgi/math/1026218280/l50
面白い問題教えて 第2版
http://natto.2ch.net/math/kako/1004/10048/1004839697.html
面白い問題教えて
http://cheese.2ch.net/math/kako/970/970737952.html

問題に答えた人が次の問題を出すスレ
http://science.2ch.net/test/read.cgi/math/1037801891/l50

953 名前:132人目の素数さん投稿日:03/02/01 04:59
>>2にリンクするとしたら、これですか
---------------------------------------------------------------
★東大入試作問者になったつもりのスレ★
http://science.2ch.net/test/read.cgi/math/1000592003/
☆2ちゃんねらーず編・大学入試数学問題集☆
http://science.2ch.net/test/read.cgi/math/988730706/l50
天才中学生の俺を試して下さい。
http://science.2ch.net/test/read.cgi/math/977469232/
★小学生向け問題募集★
http://science.2ch.net/test/read.cgi/math/1020248263/l50

もし解いたら数学史に名が残る問題
http://science.2ch.net/test/read.cgi/math/1016031138/
シンプルで難しい問題
http://science.2ch.net/test/read.cgi/math/1011067036/l50
Qマソが問題を出すスレ
http://science.2ch.net/test/read.cgi/math/1040710744/l50

954 名前:132人目の素数さん投稿日:03/02/01 05:01
今日はバイトで一日中いないから、新スレ立てるときは、テキトーによろしくお願いします

955 名前:132人目の素数さん投稿日:03/02/01 05:24
面白い問題おしえて〜な 四問目

956 名前:132人目の素数さん投稿日:03/02/01 05:44
>>955
4問目 それいいかも

957 名前:132人目の素数さん投稿日:03/02/01 05:46
「おもろい問題をキボンヌ! 4匁」
と書くと頭悪そうですね

958 名前:132人目の素数さん投稿日:03/02/01 06:11
とりあえず、次タイトルは
   「面白い問題おしえて〜な 四問目」
の予定で、おねがいします

959 名前:132人目の素数さん投稿日:03/02/01 07:18
/ヘ;;;;; 
';=r=‐リ 次スレでもぜひ面白い問題を教えてくれたまえ
ヽ二/  馬鹿どもにはちょうど良いめくらましだ


960 名前:132人目の素数さん投稿日:03/02/01 10:41
/ヘ;;;;; 
';=r=‐リ きみたち「面白い四問目おしえて〜な」が良いのではないか
ヽ二/


961 名前:132人目の素数さん投稿日:03/02/01 15:32
問題を解いて金持ちになりたい!

偶数 2n
奇数 2n+1

素数は?

懸賞金がかかってる数学の問題ってこの他にもあるそうでして
http://www.google.co.jp/search?q=%E6%95%B0%E5%AD%A6%E3%80%80%E6%87%B8%E8%B3%9E%E9%87%91&ie=UTF-8&oe=UTF-8&hl=ja&lr=

スレッド立てられないんですけど


962 名前:132人目の素数さん投稿日:03/02/01 21:45
素数の一般項はないということは証明できないのかな。

963 名前:132人目の素数さん投稿日:03/02/01 22:17
19変数の多項式で、式の値が正ならば必ず素数で、
任意の素数がその多項式でもって表せるというのが
なかったっけ?

964 名前:132人目の素数さん投稿日:03/02/01 23:17
マチアセビッチだっけ?
聞いたことはあるけど具体的な式の形は知らん。
ただその式は、素数も生み出すが素数以外も生み出すから
>>961の問題の答えにはなっていないのではないかと。

965 名前:961投稿日:03/02/01 23:37
あのー すいません
自分は2年に上がれそうにない高1です
講師にこんなこと言われて 金欲しさにレスしてしまいまして…
この板初カキコでこんなことスイマセンでした
足し算からやり直します

世界の学者が考え続けてる問題だそうです
何もすることなかったら考えてみて下さい…

966 名前:132人目の素数さん投稿日:03/02/01 23:58
>>965
何もすることない時ってないと思うぞ。
2年にあがれるよーにガンバれ!!
高1数学はまだ簡単な方だから、今からでも大丈夫。

967 名前:961投稿日:03/02/02 00:05
頑張ります…(泣)

968 名前:132人目の素数さん投稿日:03/02/02 00:41
でてきた。Matijasevic's polynomialの25変数26次式。
http://216.239.33.100/search?q=cache:9LwD-sEBWfsC:primes.utm.edu/glossary/page.php%3Frandom%3Dgaps%2520between%2520primes+polynomial+prime&hl=ja&ie=UTF-8
うんごいで。これ。このページによるとただいまの世界記録は
次数の最小5(24変数)
変数数の最小10(1.6x10^45変数(!))



969 名前:132人目の素数さん投稿日:03/02/02 00:44
違った。10変数の16000000000000000000000000000000000000000次式だとよ(w

970 名前:132人目の素数さん投稿日:03/02/02 00:58
ユリと待ち合わせbitch(はぁと

971 名前:132人目の素数さん投稿日:03/02/02 01:09
http://science.2ch.net/test/read.cgi/math/1026218280/l50#http://science.2ch.net/test/read.cgi/math/1026218280/l50.2ch.net/
http://primes.utm.edu/glossary/page.php?sort=MatijasevicPoly

972 名前:関連スレ投稿日:03/02/02 01:15
次スレ立てました。

面白い問題おしえて〜な 四問目
http://science.2ch.net/test/read.cgi/math/1044116042/l50

973 名前:132人目の素数さん投稿日:03/02/02 01:45
>>966
>高1数学はまだ簡単な方だから
フォローになってないような・・・


974 名前:132人目の素数さん投稿日:03/02/02 01:47
高1数学って誰でも解けるよ。だからがんばって

975 名前:132人目の素数さん投稿日:03/02/04 22:49
最近 低調ですね
誰か面白い問題おしえて〜な〜

976 名前:132人目の素数さん投稿日:03/02/04 23:56
>>940
中心から扇形を2等分する直線上にあり、
中心からの距離は扇形の半径の2sinθ/3θ倍の点。


977 名前:132人目の素数さん投稿日:03/02/06 13:19
1000とり合戦は?

978 名前:132人目の素数さん投稿日:03/02/06 18:56
>>977の要望により。

      \∧_ヘ     / ̄ ̄ ̄ ̄ ̄ ̄ ̄ ̄ ̄ ̄ ̄ ̄
 ,,、,、,,, / \〇ノゝ∩ < 1000取り合戦、いくぞゴルァ!!       ,,、,、,,,
    /三√ ゚Д゚) /   \____________  ,,、,、,,,
     /三/| ゚U゚|\      ,,、,、,,,                       ,,、,、,,,
 ,,、,、,,, U (:::::::::::)  ,,、,、,,,         \オーーーーーーーッ!!/
      //三/|三|\     ∧_∧∧_∧ ∧_∧∧_∧∧_∧∧_∧
      ∪  ∪       (    )    (     )   (    )    )
 ,,、,、,,,       ,,、,、,,,  ∧_∧∧_∧∧_∧ ∧_∧∧_∧∧_∧∧_∧
      ,,、,、,,,       (    )    (    )    (    )    (    )


979 名前:132人目の素数さん投稿日:03/02/06 18:57

           ,,.r''"     ,,,.........,,__     ヽ,
         ,r゙'"   ,.、‐''"~  /'ヾ゙、゙゙''‐ 、   ヽ、
         ァ   ,.r'" / /   |ii  ヽ、 _゙'、   ゝ
          /  ,r'/ .,.'   / ,., l l i i ,,r'ヽ~''ヽ .,゙、
        レ  ,イ/  /  ,' .〃 i.i.  i i ゝ-r'゙'''ヽy'ニニニ''''、
         {  ,'lj' //   i .!i ,iリ   i,'i,ヽ.ゝ,,,,,r'{ !,.、、-- ゙、
        } l i〃,'  i ,i,' l ! //   _,,,,',l゙、 i'゙'-‐'゙ヾl, -r,  ゙、
        'l. l',. i! .i , ! イ ,'!l,/'  ´  ヾ゙ヽ ̄'!゙'''i 、''ヽ!゙!  ヽ
        i、l ', ', ! |.l /,'/ 〈     ,.、__` ゙''‐!l. l l l'、 {.    ヽ
      ,、、,,  |. ', ', jiリ'ィァ、, '    'rツr;rヾ;、. リ !l !,ヽ. ',    \
      }゙''‐ソ'| , ', ',.リrr"i゙゙'iァ     '゙ l::;゙''。゙i'}ir! .l.i | l'ヾ'‐ 、   \、
     ,,ツシ'゙,r'| ',. ',.ミil! i;::`o'}         ! :;;;:.リ/'l !,' .| l iヾ-、jヽ    ゙、ヽ
     ヾ´,,,ノ j'| ',. ',゙!ヽ.'〈::. ノ         ゝr,,゙,, .l .,','  .lノ、} }、  〉、  /   \
      `,r',r{ |i. ', .',.! =='''"   '      ` .i .,',' .i lレ'/ヽヽ/.-ヽノ     ヽせいぜい萌えて
      r' /ヾl l  ',. ',l,        rっ       ,イ ,' l l.l'゙  /     、     ', くださいね。
      {,く   l l   ', ',゙ヽ、    ''    ,、‐',,l / i  !|  ヽ,       ゙、ヽ     'ri、
      ヽヽ、i.! l   ', i'ゝ-゙ツ‐ 、,,_ ,,、-'"|-‐','// ,' l l   / ヽ,    ゙、 ヽ    j.j ヽ
        ヽ,!l. |  i ', l   rユ''},、,  r、'''''ツ / . ,'   ! l''i /  .ノ '''‐ 、, ゙、 ヽ, 、.//l ヽ


980 名前:132人目の素数さん投稿日:03/02/06 18:58
              _,..-ー
             (           _
              `ー、_   ,、‐' ´ ̄;;;;;;;;;;;;;` `;;丶 、
                 >'´;;;;;;;;;;;;;;;;;;;;;;;;;;;;;;;;;;;;;;;;;;;;;;;`ヽ.
             _,、−''´;;;/;;;;;;;;;;;;;;;;;;;;;;; ;;;;;;;;;;;;;;;;;;;;;ヽ:.....::\
            /_;;;;;;;;;/;;;,' ;;i!:: ``丶ヽ;;;:゙、  ::::::.....丶::::::i:ヽ
          /V レ': /...;;|: :;;lll: :;;;ヽ...:::;;;;;;;;丶:::;;;;;;;;;::゙、..ヽ:::i::::゙、
         〃'v从ハ:::,';;;::;;;l;;;;;i llヽ:::;ヽ\::....ヽ;;丶:::゙、;;...i;;;;:゙i:l:::::!i      
          /;;;イ;;;;;;イ;;i;;川ハ:::;;゙、ヽ、ヽ;\\;_;;;;i;;;;;;;;;;゙、::::i;;;...ト::::::l:l、    
         イ;;;f l;;;;;;;;;;;;j、;;;lヽヽ;;ヽ、ヽ  '´ ヽ;;ト爪;;;;;i;;:::i;;;::i::::::::i::;l    
         i从{ !,;;;;;;;;リir;;!… ヽi     f´ラTr 、 l;;;;j;;;::j::::j、::::::゙、:|    
          ヽ! i;;;;;;;;;;;i 〃イヽ.      {iii;{ ノ八レ;;;;;イイ;;;;;;;;::::ノ::|   
             ヽ;;;゙iヽ l! {;;( j      ゙0:::iiノ:ト'´jノ  !;;;;;ヽ:〈::::゙i   
               `' ゙、l! i;ii;q      ゙ …'´    /i;;r'´:V::ヽヽ 
                  } 亠' ,          r-'´;;;;ヽ;;;::::::::ト:::ヽ
                   人    ` r =.i      |;;;;;;;;;;;ト、:ヽ:::ノヽ;;::\12人のいもうと達に萌えるゴミども
                /::〃` .、   ヽ.ノ   /  |;;;;;;;;;;;;i:::ヽf:::ヽ::::i;;;:::::\
                 /{:::::;;;;;);;;;;`− 、_ ,  '´  _⊥、;;;;;;;;l::::゙;;ヽ;人リ:::::::::::::\
               /;;∧,‐';〈;;;;;;;;;;;;;;;;;; 〕__,,.-ー'´   ゙、_,..- 、:::/:::ヽハ:::::::::::ヽ

981 名前:132人目の素数さん投稿日:03/02/06 19:00
     o ハァハァ・・・
  o_ /) 
  /<<


982 名前:132人目の素数さん投稿日:03/02/06 19:01
                        _,,.-、
                       /  /ヽ
                 ___∠ヽ___/ //\\
             _,.-‐ニ二,,____ ̄_,.-`゙'ー-、_/ _>
       /_ヽ,.-'"/      `゙'ー-、_ //`<`ー--、
 ____   / //  /:::.           `゙゙ヽ、i |\`ヽ、\
 ヽ 二ニ/` ̄フ´                ヽj  ヽ、}ヽj
 / | ヽ /  /        !            \ヽ.   \
/  ! !/ /      ,ヘ |  .::l   i ヽ;::. ヽ;::.... 、ヽi::.   \
i   l レ'    /  .:/ i  ハ .::::l   l :::ト、:::..ヽ::::::.ヽヽ!::::..  ヽヽ、
{  ! /  / .::/  .:::l :l ! l .::::ヽ ::l\::\\::ヽ;::::::ヽ;}::ヽ;:.   i::.ヽ
\_,.〉' / .::/ ./ .:::!l :::!l.:l ヽ、::ヽ;:ヽ \,,>''´\i:::::::::l:!::::l:::::. .:l:::. }
  〉' / .:::i .:::l .::::ト|_;;;!,!;!,_ ヽ、::ト、ヽ,,.',,.==-、ヽ;:::l:::::::l:::::::.::::l:::.i
  ,'  イ :::::! :::;! '"i´ヽ`:i=,,、゙'ヽヽ;::{ \i'イH' lヽヽ、|:l⌒y::::::::::/∨
  | /l ::::::l /i ::::l 〃ト! `ヾ'i ヽ.ヾ    |:ト-ク::;i!レ´ト/ ハ::::::::::i:::{
  ! / l ::::::!' ヽ;::l〃iー';  oハ  `,     ゞ;;ムi!' ,'__ノ:iヽヾ;::::l:::`ー;'
 ヾ!i ヽ;::| ::iヽヾ!|l! i!o`ーク::i!   i ;.      / \::l::::|:::`ー-‐'ゝ
   ヽ !ヾl ヽ.ヽ、''゙ ヾ;;:ム〃  /r‐‐、    ,.' }   ト、!:::i:::|:::|::::ヽ、
    \{ ヽ  ヽ  ''""´  ,.´ `-‐'   , ' /   l  `ヽ;:l:::ト、:::::}
        \  \,_   /,      /  /    !    `゙'ヾ;::jこの板は「チェキ」に反応するヲタが
        /`゙'ー--`┬´!  `' ーr '´   /      !       \ 20%くらい?
       /      ,.┴┴、_,r--、ト   /     l   ,    /⌒ヽ、

983 名前:132人目の素数さん投稿日:03/02/06 19:02
                    /゙i
             _,..-ー'''''¨フ......ト、,ヘ ‐、
            /. . . . .., -'"..........゙i/.............`'-、
           / . . . . .... ........l..... ...l..∧    `ヽ、
          /. . . ... . ..... .......|:i::: ...|j:::::゙i.........`、  ヽ,
         i ..... ,'  :::..|..::l:::::|:|:::.....::|r…j゙i:|:i,::::ヽ.....、゙iリ
          i .....,イ . i:::::|:::l!::::|::li:::::::::||  ルi:|::l:ヽ::゙iト:: リ
           { ノ i ::|::|:::|ルム、リ{ト、:::リ ⌒j:l::l:::i::::゙iルイ
         _V彡 :l ::|:从_i.=-i、 !.ヽ、i ィク;iヽハノ:::}l:Hヲ、
        _{_ハ、ノ゙i ::iゝ<fノ;;;d}     {s;;;リ'´リノリjノ \
      ,-''"/\Lv'"i,::゙i ヽ;;;シ     `''┘!'f゙::::::::`ヽ、 . \
     /. ./ /:::八、_゙iヽゝ     _,,,`   从!`''-、;;;_:::::ヽ ヽ、
    // :/:::::/    'ト、_    ヽノ  /      `ヽ;::`' 、ヽ
   / /:::::/     ,ィ|ヽ、` 、_   ,/          \:::\`、妹を女と見抜けなければ
   / /::://    _,、-i'{f  `、  }¨〔_             ヽ::::ヽ:ヽ(シスプリに萌えるのは)難しい
  i' /::://  _, -''" ヽ;゙ii l、  ヽ  j j} |;i`''- 、         ゙i:::ノi ゙i
  !/:::/::{ / ゙i,    i;;l\ヽ、__,.'ノ !;|  l ヽ         l/:::|: l
  |: /:::::|´   l    l;;l  `''ーー'´  |;|   l  i        /::::::|: l
  |/:::_ノ    ゙i、   };;|-------ー―┘| ,'  !       /:::::/|:  l
  {_√`''ー−  〉  /;;;;;;;;;;;;;;;;;;;;;;;;;;;;;;;;;;;;;V    }、      /:::::/:::|:: /
 /        |  /;;;;;;;;;;;;;;;;;;;;;;;;;;;;;;;;;;;;;;;;;;゙i   ソ     /:::::/::::ノ./


984 名前:1000投稿日:03/02/06 19:03
うんこ

985 名前:132人目の素数さん投稿日:03/02/06 19:04
 .  / /  / //_,.  -‐- 、 ヽ ヽ i
   ' /  / /r‐' / / / ` i ヽ `、 i  〉
  ' ./   , i/./// ,イノ ! !、 i `i ! /
 . ' / !  , i!/_,/'=/ノ/' i./i ト! ! i、i i
 / i  i  ,  i〃_ノ:::!ヽ   ' リ=!イ ./ !ヾ、
 !.i  i  i  i"i::::::ノ     '_):!ゝ〃'i ヽ
 i .i i ! .i  i  ̄       !:::ノノ ' / i !
 ヽi i ! i i.  !    rー 、' ` '/ i / i i花穂のAAはなぜか少ない
  ``从ヽi 、!` 、  、.ノ  ,  ' i , ー'"'"''ー''"'- 、
    ゝ'i i从-、 ` r ,',..ゝ-‐'"~.."'"ー'- 、:::::::.... ーヽ
      /   ` 、 ,`':::::::::::::::::::::::::.....  ~` 、::::::::.. ヽ
 .    /ー _    ,',::::::::::::::::::::::::::::::::::..   ヽ::::::.. ,
    /~`ヽ、ヽ  、:::::::::::::::::::::::::::::::::::::..   i ':::::: ,

986 名前:132人目の素数さん投稿日:03/02/06 19:05

           ,   ―__ ―-
           /   /((゚_))((⌒))` 、
           '    />‐--‐^ー-‐ヘ
       ,' .  ,'il l |i| |!l !ヽl |l il il   i
       ! i  l l |リ-‐!リノ |!リ l l !|i  }
         |lli|! l|| lノ/,.ィfi⌒   ノノ!il|||l リ
       !il  li| !| !|Jj!     _ 〃ノ '
        、   l. `ー'-   ,'⌒ ,!       / ̄ ̄ ̄ ̄ ̄
        ヽ   `ーiヽ.  「/ /ゝ    < 1000までに12人すべて召喚できるか?
          `ー-‐1l. ト ,. ィ´!        \_____
            / |. [l 「ト、ヽト、
              / .|. レ`ヘ.、 \>
           i'⌒゙l    k 〉
           l/  l   ノ -ヘ
            /  ∧  /i
             〈 _/  `ーイ  ノ
           l/r'ヽ_  /!  '
              〈/l  `7 !l  |
              ヽL__,.イエl]  |
               〃   lZ  |

987 名前:132人目の素数さん投稿日:03/02/06 19:07
     /::/;;/;;;;ノ;;:::/ ;::/ :: |l : ::i ::l:::l::ヽ::\
     / /;;:/ ///;/ /;:/|/|/ll : ::|lN|l : :;:`,:ヽヽ
    〃/〃/,'/i /,'li //;:// / ! | ::l|'|l :|l :ll;:.ヽ;:ヽ\
    ,!〃//,','i|| ll||l/;:lノ_/;:/ ノ./;::/ ||l:}|l;:::l|i :ヽ;:ヽ ヽ
    |〃/i|:l:|i|::i|,||,||!'ニフノ~、` //..._|l! ||l:::|.|l;::`,|::l  !
    |/〃|ll |ll::l|'!|ン'"'ミ'ヽ  ヽ  , ;'ニ、ヽ|l'::l |'|l;::|レl
    |:〃/!|| !|l||.イlゝilシ}   ト//ノ} '} ノソ.ノ |l;:!' '
    '!/〃 'l|::'!l|| ヾ::ノ   ノ l {,;''ノ ノ./:::l|   レ'
     !〃/|;:;::::::'! ヽ、 __ . - ´ ン.`ー  ,' ノ{    / ̄ ̄ ̄ ̄ ̄ ̄ ̄ ̄ ̄ ̄
     !/,'l|||;:;::l :::|、    !_フ   ``'フl´|l::|  < メガネっ娘萌え
  .//;l'iill||!;:;:| :::|i` 、      _. ; '|||ll::|l :l,   \__________
  /: /ll|{ill||li;;:;::l :::|l\._ ` - il|ll:: | .||li: :|;:|'|
 ,'//;il||liソil|lゝ;::| ::!i   ` 、{ヽ!||!' ノ l|ll: ::! | |
 !i!i i|lll!ァ-'´_ `ヽ;||::!i、- ,,フハ、、ヽ、_ ノ|!: i ::l ノ
. |l;!''"‐‐- 、 ヽ、` !|l;:!i《〃|!i| 》、 ヽ.リli::ll!'
.{!´     ヽ ヽ  !、`´ ||.||´ ヽ  ソ!ノ ヽ

988 名前:132人目の素数さん投稿日:03/02/06 19:08
   _,,,,,,_               _,,,,
i'' ̄    .~''''-,,        _,,-'''' ̄  |
ヽ        ~''-,,_   r'~      |
 ヽ          ゝーく       !
  `'-、     _,,w''~ヾヾy^ヾ^i'─vく~
    `''-i _,,,/ヽ ヽ `ソ ノ ノ /ノ/ゝ
     ./~ //''⌒''7v、ヾヽiノ/ノ∠-''ヽ
    .ノ r'~ン'/~//^ソヾ''-,,|~Wヽ,-''⌒'i、
    .ケヽi/爪 .// //   .!/ |i.|iヽ  (
   .// ト./.i/.七二'ヽ   ,ナ-|.!.||.!|⌒ヾr'
  / /  /ヽ!/t'iP'ヽヾ   ノ' ノ'ノ'レ'|ヾ、`i
  ン'~  /(r'd ! .!.!ゞソ.ノ    ノ   r'八ヾ
 ヽ_/ ム''~ヽ!| ` ─ ´   i, ''~''ヽ iソ))'''-ゝ
   ! ( /⌒ソ)\    ┌-r´   .Aン,─、ト、
   !、ゞヽノ.)ノト.丶   !_/ ._,,,-< ゞ(○ )ノ )
   >-,,,,ノム''─<丶  <( ゝ  ゝ 二 _ノコメントのネタ尽きた・・・
  r<\ヘヽ  .!_    ̄('''') ̄~''i─,,,,─'''
  .| ヽ ゝヽv'~ヾゝ//~!∧ヾヽ(ヽ .| ~ゝ
  ソ ヽ \ヾ/ヾ.ム | ∧ゝ ~ !,レ'  \

989 名前:132人目の素数さん投稿日:03/02/06 19:09
)                                 ┌- 、._
                                |    7
                 _,,.. -──- 、.._     |   ./
               ,.‐'"´           `` ‐、|   / /
            /                  ///
      ,,.. -─- 、/                    /
    ,.‐'´ _                         ゙i、
 ∠ ‐ '"´/                          i
     ./                                 |
    ./    ,.イ                        |
   .l    //      ,イ ,1    |ヽ     ト、      !
   i   ./  i  ,   /l / l.l !  、 ト、゙i ,ヘへ、l ヽ.ト、|、 /
   l.  /  .|  /|  /-|←┼‐l、 ヽ ト、!, -─ヽ|─!-l、i /
    ! l   l | ! |、`';:‐-_、._ ヽ、l\l-i'   _,_-='、"~! i"ヽ
.    ヽ!     ヽ|,/l゙、! l(  (80j` l──|. イ80)  )l.  | )|
          l.(l  l ー-‐'   ,!   l、`゙‐--‐' l  /"ノ
          ヽ ヽ、._l_   _,.ノ  〈>  ヽ、._   _.l_//   / ̄ ̄ ̄ ̄ ̄ ̄ ̄ ̄
             ヽ、.__,l  ̄     __     ̄ /、‐'´    | バーロー
               `‐ 、         , ‐'´‐-ヽ    < あんたの自作自演は
                _ ./l` ‐ 、. _,. ‐''"!\         | とっくにバレてんだよ
            _,,.. -‐'ヽ ̄ヽ,-、シ ̄//  |`‐- 、.._      \________
           / i  / /_ 7`‐゙\__.\.  |     i ヽ、
             / l  /   | /||\ / ̄   !   |  !

990 名前:132人目の素数さん投稿日:03/02/06 19:09
          、             _,,_ i,
          ~ ‐-, -‐ ' ~ ,, --"‐、__ ,, 、
         , -‐フ" ~'''''''' " ..,,,,,,,,,,,,''';;ヽ、~'‐、
      , -.~/          '''''''''' ''''''  、ヽ
     ///;;''          .,,,,. ''''::;;;;;;;;;;,,~ゞ、
   / ./∧!;;    ,,  -‐‐‐‐‐--''''';;;;;;,,,,,,;;;;;;;;;,,ヽ,ヽ
     !" j'' ,, -イ / / ;;/ ,,;;/! !'{ ''ヾ 、;;;;;;;;;;;;了'、
     | / ./! ,;/!  ! 〈 ;;/ ,,;;/ リ ! i ! ;;| '‐、;;;;i', j
.     !イ/ 〈 ;;| 'i十! 、、i ,;;;/ ノ j'ナ ノ‐,| ,ヘ,i Y、]
      〉 | ;;;| ;;|、.!,,、-、,.!、;;i ' ノ, -‐' 、リノノ;;; | :
      i { ;;, ;;,、 ;;| ,/,i_j;;;;i 、i / i-j;;;;;,!ト |;;;; !' :::
      { ヽ;, '';;;;,,ゞ 、{、'''_i     i、''''_,ノ };;;;, |:::::
      '! ゞ、;;;;;;;!\' ‐''   ・    ~ " ノ;;;;;;, |: '
      '  | ;;;;;;;;;ヽ、    ー     ',./;;;;; |
        {i‐i ;;;;、,|"'‐ 、.     , イ,,";;;;;;''/^|
        '、^、 ;;r、! '、7‐- '.-‐_" --‐i;;;;;;''イ }衛のAA少ない
        , -ゝヽ ヽ‐''"i, ~||_||   _,j''ノ! リ "キボンヌ
      /  .//ヽ、  ~'‐ノ .i,_, -‐ '"   ~「i‐-、
   , -‐'"    //u     i .j        .|.|  \
  ノ       .|.|ノ     / /        /;;}:}   ヽ

991 名前:132人目の素数さん投稿日:03/02/06 19:10



           r ==- 、    -―-ァー―-y‐-
            l !/ >´    //    l   \
            ヽヽ       / '   i  l\   ヽ
             / / ヽ   〃   /  / | ヽ/7
          /  rイヾ\ / / / /  / //    Y  ヽ ヽ
           /  / ! \\ l / / /'⌒   // ⌒ヽ|| ヽ )ノ
          ノ  / /l     l / / ,ィ:Tヽ レ  ハヽ ! |/ /
        ( ( く_/ ト   ,.-、 ' /1;;|      |j:l |l /
         ヽ   ノ ヽ.  !  l l kクリ    、ヒリ, |/
            )/   〉  ヽ_り ゝ-‐'  rァ  `/
            /    / /   _ヽ      ´_. イ /和服萌え
               /   //|/ l   F、 、  レ
                !/  / l\\  ーヘヽ\
                ヽ /`ヽ \ヽ \  | |V〉ヽ

992 名前:132人目の素数さん投稿日:03/02/06 19:11

              , -ー 、 )ヽ 、
            _)`'⌒ -―-  、 i
              〃, ´           ` 、
          i, '             ヽ
            /       i l i| l i  !
         , !   i  i  | . | l|| ! ! i|l l l i
          l| | ! l| l‐|‐- | l | l|l-|‐! |l l| | l
           l| ! | | l ,ィ'Tilヽノノ 'TTiヽ .!|l リ
          l| l !| ' |Jjl!     |Jj!  !l !'     / ̄ ̄ ̄ ̄ ̄
            l|l ||、 -‐゙   , ー-' /l|      |
.            l| | ゝ.   ー  , イl |!       < 面白い問題を一つ。
            !lL ィ'^i  /,\ !l |         | あと一人の妹は誰でしょう?
       ____  /| | /  |ゝ,. '二` ト !! !        \_____
.      、_    ̄ `ヽ=、/l†/,. - !, |l |ヽ
             ̄ノ i    ヽ.ヾ レァ | |! | 1
            〃 ,.'     ノ ヽil{r -‐1、l| l !
           i/ ,、     |!`ー-‐'| !リl
.          / // ! 、  !|    |   !

993 名前:132人目の素数さん投稿日:03/02/06 19:12
    、-.、._                                .__
    `ヽ、"''ー、._                       _,、-‐'" ̄/
      \   ~''ー--、__           ,,、-―''"~"~    /
        ヽ、   ,    ヾ'ヽ、__    ./'i~ ̄         ./
         \  |!   丶、\"ーェ''7 /    /   _..,,ノ
           `ヽ|`ー-、___"''ゝー<''~ ,,、-'"~i-ー''"~
          、_冫=-  ,, ~"''"⌒ ̄⌒ヽ ̄`ヽ、
          `ー,-''~ /       `ヽ \  丶
           / ノ /   ハ  、     ゝ ヽ  \
      、-ーー''フ / ノl  ./ |l  .|,  l__ .i |) `i    `ヽ、,____,
     、_ニ=‐|/ノl /l |ァ'"メ ||、 .||l!'"|l .`|l. ||  ト、   "''==ニ
.     `ー-/|ノ/| l| lハヾノ|,,..|ハ  l||l|',ェ=ミ,ハ | l | ヽ、  `ヾ、
      // リ | |l||ヾ〉.,ィ⌒i. ヽー' ' .l!。_,〉)リ ノ ,ノ i、ヽ  丶ト、
.     //  / ハ|'|ハi、'、ヾ゚ツ     `ー' ノ /|. ハ丶 、 l )   / ̄ ̄ ̄ ̄ ̄ ̄ ̄
     // /  ハ / | l リ)! 、、、  `  、、、''~-‐'ソ" | 'ノ'ハ ) ル'"リ <  にゃはっ
     リ |  /! |ヽト'^^'リ `‐、,,_  ー'  ._, -''~lハ| ノノ ノ/V |ル'    \_______
       |. 〈 ヽ| ‘  .-ーレルハ''lー-‐'"|リルy’  ~ ''"~  リ
.       ヽ|     /´  // ヽ   ,-|'| ̄⌒ヽ,
            / ._i'"⌒ヽ,、___,、/^ヽ、_  )
           イ⌒  〉  ` ( i i .) ヘ  〉 "''ヽ、
       <ニ==||⊂〜`ヾとニ〜-"ゞ、 /⌒ゝつ 〉-==ニゝ

994 名前:132人目の素数さん投稿日:03/02/06 19:12
"''‐ 、      "''- 、 > -‐'"    "    "''‐ 、__/
   , "''-‐‐-    , -'"       、    \  \ゝ_
‐'' "       , /./   /       i  、   \\ ヽ ) ヽ、 _‐-  .,_
   /   , , '/  /   //    / ト  i 、   ヽ丶ヽ‐ 、. \ "', 、
‐フ"   / /./  /  i. イ { i    i .ハ ii i  、 、、 ',. ',  `  ヽ ヽ "'
   , / /_,{  {   i /i i ii  , /i / .i ./i ト .i  } i i }.    i、. ゝ
 , ' / , {  l  i  i { i .il_ii i  { i { /- i./ 从i_i. i  i ii i       i\ 、
"  /./ i/~"i. i i  i i l''リ_,,.、、 i i レ  ノ,'--' 、 }/i ノi ノ ノ  /⌒i } ヽ、
  / '   .k. .ハヽ、i  トi ,/r_i;;。iヽi'    r_i;;;;。 ゝ i ノ'} ィ/  {.  l./
        i '‐'  ` ヽ 、ゝ i;;;;;;ノ `   ヽ;;;;;;ノ_ノ"/lレ ./   ヽ_,.レ
      i      ヽ ト` ‐ '       "'"  レ''ノ      ノ
       ヽ         、    _`_ .,    , '' "      ノ
        ` , - '" " ' ' 、  ヽ、_ノ   , / )     /
       /         i.i '、 ,   ,  ' '←正解はこれ。1000ゲト。
      /          i i  \. ' \ /
       i,,;''           i i , -‐‐‐-、"',i'‐ 、
     /        ,,;, i iヽ    , ‐"ri'" ̄`、
    ゝ        ,,;;;;;;; i i ゝ '" 7 トiイ./ '!
    ノ       ,;;;;;;;;;;;;;;,,;i i. {,-‐ '// i i }   〉

995 名前:132人目の素数さん投稿日:03/02/06 19:13


是非とも御協力ください。
下のHPをクリックして、現在2位の小玉に票を入れてください。
いずれ大仁田をぬいて1位にしたいのです。
できれば何回もおながいします。
小玉が1位になったらみんなで万歳して一杯飲みましょう。

http://kenji03.hp.infoseek.co.jp/cgi-bin/kage/votec.cgi



996 名前:132人目の素数さん投稿日:03/02/06 19:13
     | ̄\     \          (´´
     |   |: ̄ ̄ ̄ ̄:|       (´⌒(´
     |   |:      .:| ≡≡≡(´⌒;;;≡≡≡
     |   |:      .:|≡≡(´⌒(´⌒;;
              (´⌒(´⌒;;
           ズザーーーーーッ



   _____
 /:\.____\
 |: ̄\ (*゚ー゚)\  <1000GET!
 |:   |: ̄U U ̄:|


        バタン !!
 _______
 | ̄\     \
 |   |: ̄ ̄ ̄ ̄:|
 |   |:      .:|
 |   |:      .:|

997 名前:132人目の素数さん投稿日:03/02/06 19:13
       / ...:/             、 ヽヽ\
       / ...:::::: /           、\  \ヽ
.     /..:::::::::/            i   、 ヽ、 ヽヽ
.    //:::::::::/:.    ./     ヽ   ! i !  ,  i 、`
     /:::::::::/::::  .:::!    i i }:::.  ! ! i、i:: i ! !i i
   /:::::::::::::i::i::::. ..::::i    ! ハ ハ:::.. ii.ハ ト !::::i!:i.ハi
.._, ' -‐ '::::::::::i/!::::...:::::::!  ..:/:/i:!_,i,! i::::..ル'i.リメi::://リノ
 ー'"-‐ '::::::::i::i:::::::::::::::i:: ..:/:/ /,,_ノ !::/ '、:).i/イ
   ,~''":::::::::::!:i::i::::::::::::!i::::i '/、_,}  レ'  、  'i./
  'ー- /::::::::rゝ、:::::::::! !::!ムヽ::::ノ.     ゝノ
    ' '"/::::::{、ヽ\:::i `:!        _,  /
     ' i/!:::::`....、_` 、、`       ̄   /12人のAAを貼った
       ノ, ‐Z - 'ー 、_` ー  _   _, '漏れがヲタだったっていう話。。。
      _ , ‐i      ~ ` ー-'-‐‐‐‐‐‐‐-、
   , '"   〈                /
. //     \ 、              イ

998 名前:132人目の素数さん投稿日:03/02/06 19:14
次スレ

面白い問題おしえて〜な 四問目
http://science.2ch.net/test/read.cgi/math/1044116042/l50

999 名前:132人目の素数さん投稿日:03/02/06 19:14
998

1000 名前:132人目の素数さん投稿日:03/02/06 19:15
1000!!!!!

1001 名前:1001投稿日:Over 1000 Thread
このスレッドは1000を超えました。
もう書けないので、新しいスレッドを立ててくださいです。。。


DAT2HTML 0.26 Converted.